50% found this document useful (2 votes)
1K views188 pages

5c Beastacademymath0000batt 1

Uploaded by

coder.spidey
Copyright
© © All Rights Reserved
We take content rights seriously. If you suspect this is your content, claim it here.
Available Formats
Download as PDF, TXT or read online on Scribd
50% found this document useful (2 votes)
1K views188 pages

5c Beastacademymath0000batt 1

Uploaded by

coder.spidey
Copyright
© © All Rights Reserved
We take content rights seriously. If you suspect this is your content, claim it here.
Available Formats
Download as PDF, TXT or read online on Scribd
You are on page 1/ 188

My,

IArtorProblem{Solving
z
Wee
ae
roblem\Solving

wl?Ltda
© 2016, 2017 AoPS Incorporated. All Rights Reserved.

Reproduction of any portion of this book without the written permission of AoPS Incorporated is
strictly prohibited, except for fair use or other noncommercial uses as defined in sections 107
and 108 of the U.S. Copyright Act.

Published by: AoPS Incorporated


10865 Rancho Bernardo Rd Ste 100
San Diego, CA 92127-2102
info @BeastAcademy.com

a
ISBN: 978-1-934124-65-9

Beast Academy is a registered trademark of AoPS Incorporated.

Written by Jason Batterson, Shannon Rogers, and Kyle Guillet


Book Design by Lisa T. Phan
Illustrations by Erich Owen
Grayscales by Greta Selman

Visit the Beast Academy website at www.BeastAcademy.com. : \ a


Visit the Art of Problem Solving website at www.artofproblemsolving.com. ia
Printed in the United States of America. if
2017 Printing.

2a
« Sequences
« Ratios & Rates
e Decimals
This is
Each
ele ah : chapter of this
MATH N Practice book
\ corresponds to
PRACTICE | | a chapter from
: Beast Academy
Guide 5C.

~You may also read


Vie are page the entire chapter in the
each chapter includes a
recommended sequence
Guide before beginning
for the Guide and the Practice chapter.
Practice books.

Use this Practice book with


Guide 5C from BeastAcademy.com.

Recommended Sequence:
Use this Practice book with
Gulde 5C from BeastAcademy.com.

aaibbck
Book
| Book Pages
crPractice:
| Guide: 12-27
Guide:

ie
You may also read the entire chapter
in the Guide before beginning the
7 Practice: ee.
Sea | Guide: 28-34
Practice: 12-21
Guide: Oe
Practice: 22-35

You may also read the entire chapter


in the Guide before beginning the
Practice chapter.
Every
Yow to Uae. swe iat
Some problems problem marked
in this book are very with a we has a Hints for the
challenging. These starred problems
problems are marked begin on page
with a w. The hardest 108.
problems have two Other problems
stars! are marked with
. For these
Examples are included to help you
‘understand the problems in the section, problems, you should -
* ros write an explanation
for your answer.
een

‘Thetext that goes here usuallyincludesan.


‘explanation
of the solutiontothe problemgiven
in the example.

PRACTICE| instructions to the practice problems go here

Since this problem has a star, it must be really hard) Problems with
stars have hints in the back of the book, starting on page 5.

After problem 54 is problem55. All of the problems in each chapter


are numbered in order, starting with problemnumber 1. Since this
tagblemhas a pencil, your answer shouldinclude an explanation.
_

Somepagesdirectyouto
relatedpagesfromtheGuide.—

None of Solutions
the problems in are in the _Acomplete
this book require back, starting explanation is
the use of a on page given for every
calculator. 112. problem!

Piatend SoletindSeyes 61.You


may
{9010
tobeatrouble
muchsking
yourself,
“Why
wringdid
tox the
that
no
oauthor
ne
can
54,Each
ofthe
solutions
inthe
back
fthabook
includes read
without
super-human
vislon?*
That's
no
complete
explanation
ofhow
tosolve
theproblem,
not ‘900d
anwar,realy,
st hoanswer.
Sometimes,
wowill
provide
asolution62.Sometimes
wogive
twopossible
wayslosolv
‘hat
iscifferent
{11s
usoful than
toreadthe
sway
the you
tosolved
olution the
every problem,
problem
oven
it problem.
‘oraUsually,
‘word nd nego
somo two
linesways
thenare
like separated
es bytho
below:
thalwo
made!
Wouldn't
itbefun
towrite
theeuttors
10 Tho
seoond
salution
would
come
here.
Generally,
tho
Pint
out
amiatako
thoy
made quicker
of
merw
lever
solution
comes
second.
55.
These
arjust
itlookod
tkefako
solutions
thore
was that
test wowrota
hare.
Hwe
put 60
he that
real 69.Hore
ance
trom ikslston
actlrent
book wn
Intho sAcademy
ome
BoastSaga,
serio
solutions
hero,
totho
itwould
give
away
problems
thal
tome
show
ofheanewere Canyou
upinthebook.
Then,
you
ot waa enh tg atho
diagrams’
wouldrt
baable
tohave
fun
trying
foealve
them
on
{your
own!
Here
aresore
fake
Gagrams
togowith
thofake
golutions:
oof
ops geo p
3
sem
ie to Seee
tompting
just
look
In
t bans
heae
sor Taryank the eh
back
of book
for
the oie 8 a
Ippumneregiomes
caper
erring Eeyarta
ee Oe
ey aio
one
57.
Here
are
somo
more
diagrams
tomake
these
fake Can
you
guess
what
the
quesiion
thal
goes
withthe
tok00ktke
real
ones, Gagrams
above
was?
Hero's
ahint
The
answer

58.Hore
insome
mathy
stu: 65,omelet
pergola i
Nie
514524
59=50+50+50+1+2+3=
15046
156, ialolBsepac
bal bahwecian
pasteetBe
rao
oeower
bold.
This aSoe
make De
itgaslertT
toind ee tooach 66,Ie
ho
answer Curabitur
velit
pusia! isl,
Natoanatase
venetian id,
commodo
vec’>guices
problem
quickly ‘elupal
Aenean
quadamulidavolipal
dupe
bad reba sen
nies a aa 67.Makos
wordsno
rosense
doos
Latin 7|think
some
ofrose
are looking for.
68,orlrutrum
consequat
fos
nibh a drum.
dictum Praesent
consequal,
Maecenas
; t99l
volt
magna,
Ouvluipal
auctor
neque
siamat
60.
Did
{ndyou
notice
header
tha thenumber
above?
Tha
problemson
ste
inthe the
pageright
numbor
Practica
book.
Tal can Ne
where
you
way,
when ee
Pebertesque
‘ouipalut
eonvuetusun
taorem,
Moh
thous eeeti
sodaloe
Vdalee
659,
Ready
oget
started
onsome
mathproblems’?
Got
ot

Pretend Solutions
Use this Practice book with
Guide 5C from BeastAcademy.com.

Recommended Sequence:

Book Pages:
Guide: 12-27
Practice: 7-11
Guide: 28-34
Practice: 12-21
Guide: 35-41
Practice: 22-35

You may also read the entire chapter


in the Guide before beginning the
Practice chapter.
Math beasts are
always lookingfor
patterns! —

PRACTICE
Circle the figure that best
completes each pattern below.

Beast Academy Practice 5C


-Some of
these are very get them, you can \
difficult. always move ahead
Ss and come back /
later!

Write the best choice for the next


PRACTICE character in each pattern below.

8. actor 9. parwre

10. acrmquyce | WwW.


ZAXmMVOL

12,OTTFF
SsagNh
- | auitaelEKPW

Wu
MVEMIJS
ict j 15.YMEDACATS
i fal

2
16. LUE a 17. BCDEGPT“
we \P © SF —] *
* *
Beast Academy Practice 5C
Write the next three terms that best
se PRACTICE
fiteachsequence
below.
18. 1, 2¢4 7, 11,16,Ra soi
_ ZL

ioe “AG-
40;G2,22,10). id -

20640, .<320,..160;--80;40,.- OFves.

21.

22.

Beast
Academy
Practice
5C Guide
Pages:
14-19
| 3|
Ifthe pattern below continues, what
~ EXAMPLE ~
be the 50" term in the sequence?
4,8, \2, 16, 20...

_Tofindthe50"term,welookfora lationship
oeibe ct y Thethreeae
(...) at the end of
termanditsposition
inthesequence. a sequence tell us
Position: 1% 2" gd gh ih that it continues
Term: 4, 8, 12, 16, 20 forever.
: Sequences that

_ The 1%!term in the sequence is 1+4=4. continue forever


are called infinite
The 2" term in the sequence is 2+ 4= 8. [
sequences.
The 3 term in the sequence is 3° 4= 12, pe so on.
Eachtermis4timesitsposition
number.
So,theso"terminthesequenceisaC— 200.
Position: 18andgt 4h5, 50"
4
Term: 4, 8, 12, 16, 20,..., 200

In each sequence below, continue the


PRACTICE
pattern to find the missing terms listed.

24. WOyOe, yO; s- 24. 15"term:


at
50° term: 2

25. V5,0, 2272648 1253 F: 25. 9" term:


20" term: DV

26. Bie 8O yo ele phy a. 26. 25" term: | 7


20
100"term: Vidi

27. 99" term: / J


27. O77, Oy 204, OF 2%,

40" term: _\1”


a4
eee Sa
28. 80"term: <i
Lafarge 2] edeer eat Pe ba7 a 1
ff
51° term: iL

Beast Academy Practice 5C


- .___|_ Continuingthe pattern below,findthe 10" and —
_ EXAMPLE| 25" terms inthe sequence. Then, writean
-----'|__ expression for the n" term of the sequence.

3°biae3 5 d 6’ qTbee
hire Z y y
: Each term inthe sequence is a unit fraction with adenominator
that is 2more than the cae S position number.
thfthe 10" 1term is10+2~a
So, es——— the 25" term iso
i 54D
ae
=+,and
the.
nmier isn+2’

Position: {1s ond 3d Ath os . 10° as 2 “ ne


ey ed ee
Jee a. 4’ 5” 6’ Tt 12’ er n+2

PRACTICE For each sequence below, fillin the terms in the positions listed beneath
the blanks, and write an expression for ie n" term of the sequence.

uo, QRSSpa: Fe eee ae


25h 50m

‘bs Wlhe
B09 I 6. Oe 1, Ogee eee
30th 75th

ey
r iyL lzper Y
y,
Rgtee POY.WARES
Wiles
15a y/AAav
eleie rely tele ieee I 2
6th 10%

(ETE aCe er ee i ere enone

af 2,13> Geyare sore | SEN Pome


Beast
Academy
Practice
5C Guide
Pages:
20-26
[11|
-Inanarithmetic
sequence,
thesameamount
issis

‘senuenbes
211)
pipe
og
get from one term to the next.

Theamount
thatisaddedtogetttoeachnext
o termiscalled
commondifference.
-Fillintheblankstocomplete
the Arithmetic
EXAMPLE sequences
arithmetic
sequencebelow. are reallyjust
skip-counting —
ce
Webeginbyfindingthe commondifference.Togetfrom19 to
67 inthis sequence,we addthecommondifference4 times.

ees
_419,
Addingthecommondifference
4 timesaddsatotal of
67-19=48. So, the common difference is 48+4=12.
Weusethistofindtheae terms,asis
HO 412 9 ee ae ee
18, 3) 42, 20, 6

PRACTICE | Find the common difference for each arithmetic sequence below.

35. 7, 16,:25, 34, 43; .% 36. -33, -25, -17, -9, -1, ...

37. 29726725, 20; 1 6, 7x. 36e 5.5 2 loom

39. fae ANYeh 40. 30; WM ee 39.

40.

Beast Academy Practice 5C


43. ©32,304 oa ef

44.

PRACTICE
| Answer
each
question
below.
45. What is the common difference of an arithmetic sequence whose first 45.
term is 25 and whose tenth term is 115?

46. What is the common difference of an arithmetic sequence whose 23” 46.
termis+andwhose
25"termisot

47. An arithmetic sequence has 10" term 4 and 30" term 68. What is the 47.
20" term of the sequence? |

An arithmetic sequence has five terms. The first term is 40, and the
sum of all five terms is 80. What is the common difference?
~

Beast Academy Practice 5C Guide Pages:


ee ‘tAmetic
Ee eeDare
I
an EXAMPLE
What
i sthe50"‘onofthearithmetic
sequen
28 Ba ee
The first term of the arithmetic sequence is 2 — the
common difference is 3. -
+3 43 +3 +3 +3
2.5, i 14
To get to the 2" term, we add 1 tied. to 2.
To get to the 3%term, we add 2 threes to 2.
To get to the 4" term, we add 3 threes to 2.
Togettothe50"term,
weadd49threes
to2.
So,the50"termis
‘a ae 149. :

PRACTICEFindthevalueofthemissingtermlistedfor
i eacharithmetic
sequencebelow.
| 107
A9ee 45 19229,27, 31,042)10" we 50. °=11,-56,
=1,4, °9,=.cere
a0P\/

9} A C(|
oo
5TS. 5, -2880, -16,ie28aacyiwSor a 52. -29, -19, -9, 1, 11,..., Foor
—_

PRACTICE| Answereach question below.

53. What is the 13" term of an arithmetic sequence whose first term is 9 53.
and whose common difference is 8?

54. What is the first term of an arithmetic sequence whose 100" term is
40 and whose common difference is x?

55. The 12" and 15" terms of an arithmetic sequence are 85 and 106. 55.
What is the first term of the sequence?

Guide Pages: 28-34 Beast Academy Practice 5C


Arithmetic
Seguences,
Pare| :
EXAMPLE > Write an expression for the n'" term of the arithmetic
sequence below.
You can check
=|, 4, 9, 14, 19, 24, your answer
by plugging in —
The first term of the sequence is -1 and the common difference is 5. values for n!
For example,
we know the 34
term is 9. So, we can |}
plug in n=3 to get
—§(3)-6=9. v
To get the 2" term, we add 1 five to -1.
To get the 3% term, we add 2 fives to -1.
To get the 4" term, we add 3 fives to -1.

To get the n" term, we add (n-1) fives to -1.

So, the n" term is -1+(n-1)5. Distributing the 5 and ay gives


-1+(n-1)5
=-1+5n- =5
=5n-6.

PRACTICE Write a simplified expression for the n" term


of each arithmetic sequence below.

56. ed24,30,36,HEN 57. 4,19:34,49.....,LE


7 - yr nh
et Ui NB yg
&| re f 9 11
<2 Lc: eH aneene 99. G9 —

PRACTICE | Answer each question below. Simplify all expressions.

60. An arithmetic sequence has first term a and common difference 3. 60.
Write an expression for the 20" term of the sequence.

61. An arithmetic sequence has first term 6 and common difference d. 61.
Write an expression for the 101* term of the sequence.

62. Inan arithmetic sequence, the 1° term is 20, the 2" term is 32, and 62. K= ;
& ~~ the k term is 500. What is k? rs

Beast
Academy
Practice
5C Guide
Pages:
28-34
| 15|
In aSequence Path puzzle, the goal is to draw one or more paths that connect
¢ at

BDUBIIO®S
SY
20g
least three numbers in an order that formsan arithmeticsequence. __ :
Every number must be part of a path, and no two paths can cross the same square.

EXAMPLE | Solve the Sequence Path puzzle to the right.

Writing the numbers in the grid in order io least to greatest,


we have 11, 15, 16, 19, 21, 23.

These six numbers do not form a single arithmetic sequence. Each arithmetic
sequence has 3 or more terms, so we must have two 3-term arithmetic sequences.
Since 11 is the smallest number in the grid, it must be one end of a path. There are
only two arithmetic sequences we can make that begin with 11 and include 3 terms: —
Zo
11, 15, 16, 19, 27, 23

11, 15, 16, 19, 21, 23


Only the sequence 11-16-21 leaves numbers that also form an arithmetic sequence:
15-19-23. So, we connect the numbers in these sequences in order, as shown below.

68.

Beast Academy Practice 5C


69.

71.

Beast Academy Practice 5C


PRACTICE| Answereach question below.

V7. Fill in the missing entries for each of the arithmetic sqquoners below.
¥}5 1H) $9
: DS oo
rons 0 qo 35uh, 43.
d. Lo, 35
78.

b.
Cc. ae 39,
ee dae
d. ee
ee.hy.39,
asf
79. How many common differences are possible for an arithmetic 79.
sequence of integers that starts with 9 and includes 33?

&
80. How m
sequence
any
common
of integers
differences are possiblefor a
that starts with 15 and includes
narithmetic
46?
80. f |

An arithmetic sequence of integers starts with 13, ends with 37, 81.
and includes exactly one other prime. What is the other prime in the
sequence?

Guide Pages: 28-34 Beast Academy Practice 5C


PRACTICE
| Answereach questionbelow.

82. What is the greatest possible common difference of an arithmetic 82.


sequence of integers that includes the terms 27, 34, and 49? | ye

83. What is the largest term that can appear in an arithmetic sequence of 83. |
10 integers that includes 20, 32, and 50? ee

What is the smallest positive integer that could appear in an arithmetic 84.
sequence of non-consecutive integers that includes 23 and 58?

A 7-term arithmetic sequence of positive integers includes the terms 33, 53, and 73.
Circle each number below that could also be a term in the sequence.

3 28 38 53/

VY
What is the common difference of an eight-term arithmetic sequence
of integers with three numbers in the 30’s, two numbers in the 40’s,
and three numbers in the 50’s?

Fill each blank below with a digit to create an arithmetic sequence of 2-digit integers.

Ge se J
aa oo 8t oe

Beast Academy Practice 5C Guide Pages: 28-34 19°


Ina Cross-Sequence puzzle, the goal is to fill every empty square with ©

6389-SS04D
920087
a positive integer so that the numbers in each row and each column can
be arranged in some order to form an arithmetic sequence.

Fill the empty squares to complete the


EXAMPLE Cross-Sequence puzzle to the right.

The center column contains a 6 and a 13. To create an


arithmetic sequence that includes 6 and 13, we have
({[-1],6, 13) or (6, [9.5], 13) or (6, 13, [20]). Since each square
must contain a positive integer, the bottom-center square must
contain 20. We only consider entries that are positive integers
for the remaining squares.
The bottom row is ([16], 18, 20) or (18, [19], 20) or (18, 20, [22]).
Only placing 19 in the bottom-right square also creates an
arithmetic sequence in the right column: 16, 19, 22.

17 or 20
Next, the middle row is ([10], 13, 16) or (13, 16, [19]). If we fill
the middle-left square with a 19, then the top-left square must
be filled with a 17 or a 20 to create an arithmetic sequence
in the left column. Neither of these creates an arithmetic
sequence in the top row. So, the middle-left square is 10.

Finally, only a 14 in the top-left square creates an arithmetic


sequence in both the left column and the top row. ,

PRACTICE
| Filltheemptysquarestocomplete
eachCross-Sequence
puzzlebelow.4)Le
a 73
89. 90.

92. 93.

ae][20
Ky Beast Academy Practice 5C
Some of these >
puzzles are hard.
If you get stumped :
by one, it’s okay to You do not need to create ~
come back to it / [| arithmetic sequences in rows or
later! columns that are interrupted by
black squares. :

of me
|| |ss|4a}ss}6q)
Beast Academy Practice 5C
The number of dots in each figure below represents a triangular number:
The first four triangular numbers are 1, 3, 6, and 10.

st
probe
oueglwiNy
WO EXAMPLE |
@

1 dot
@
ee

3 dots
@
ee
@ @@

6 dots

What is the 50" triangular number?


ee
eee
e000
10 dots

The 50" triangular number is given by the number of dots in the


triangular pattern with 50 rows.

butoo 50rows
oN

This is equal to the sum 1 +2+3+---+48+49+50.


To compute this sum, we add two copies of it, with one copy written in
reverse. 1+2+34++-+48+49+50 :
50+49+48+-ob 34+2+1
51TOPOLte +51 +514+51
This gives 50 pairs of numbers that each sum to 51. So, the sum offall
50 pairs is 50 -51 =2,550. However, this is the total of two copies of the ©
sum we wish to compute. SO, the value of one copy is 2,550+2 = 1,275.

So, the 50" triangular number is 1,275.

PRACTICE| Answereach question below.

105. What is the 40" triangular number?

106.
Use the strategy shown in the example to write a simpler expression
for the n" triangular number.

107. Compute
2+4+6+8+10+12+14+16+18+20+22
107.
a
Guide Pages: 35-41 Beast Academy Practice 5C
PRACTICE
| Compute
each
sumbelow.
108. Ere ett ar oa ea ee ea Ges 108. “2

if

109. 6+12+18+-:-+588+594+600

“6

110. eeu @Lirt:


Polat»
a titt+is+25+ Carga? CR
+115 110:

111. 51+52+53+::-+98+99+100

PRACTICE| Answereach question below.

112. Ariana writes a sequence whose n" term is found by subtracting the
n triangular number from the n" perfect square (where 1 is the first
perfect square). What is the 25" term in Ariana’s sequence?

113. Kropple subtracts the 997" triangular number from the 1,002"¢
‘triangular number. What is Kropple’s result?

114. The k* triangular number is 465. What is k?

Beast Academy Practice 5C


Anaperatieseriesisasumofterms
thatfone:
anniraell

Nala
|
- 3+6+9+12isanarithmetic
serieswithasumof30.Thesumsthatgivetriangular
numbers
a t2+3+--+n)arealsoexamplesofans series. a
EXAMPLE bonne thesumofthefollowing he series
50+52+54+56+58+60+62+64+66+68 —
Weaddtwocopiesofthisseries,withonecopywritten
inreverse.

50
+52
68
+ +54
66
+ +56
64++58
+60
624+ +62
+64+66
+ 68
60+58+56+54+52+50
(Sere
BE

MEBHIGe
TOTHTO+
TOTIEH
TOFS

PRACTICE | Compute the sum of each arithmetic series below.

4+5+64+74+8+94104+11+124+13414+15+16 115. \ ee
115.

116. 20+30+40+50+60+70+80+90 116. /


104+214+32+43+54+65+76+87+98 117. (Ll =
117.

ae
ID)
118. (-6)+(-5)+(-4)+(-3)+(-2)+(-1)+0+14+24+3+44+5+6+7+81185 2c

119. 114+17+23+29+35+41+47+53+59+65 119. SiN


Beast Academy Practice 5C
Review counting
_ the number of ©
terms ina list in the ©
Countingchapter of
Beast Academy 4B.

PRACTICE | Answer each question below about the following sequence.

9, W857 215,25; 22 45,497 53-57, 61:

120. How many terms are in the arithmetic sequence above?


100: ee
)]
Aid| Y
121. What is the median of the sequence above? (ine

122. What is the sum of the terms in the sequence above? 122.

How could you find the average of the terms in the arithmetic sequence above using

onlythefirsttermandthelastterm?Explain. oe ee
‘r| If f ( | y' fh) \ i(| , \ {}yi rx i 44 fh)() | | i }ti L
|A if/4es"
Cj ‘ Pie
|; aVELL ‘
\ ;= LZw;t J wv
SSRI
« ez| ,

PRACTICE| Answereach question below. V

124. The middle term of a 25-term arithmetic sequence is 43. What is the 124. - rn
sum of the first and last terms of the sequence?

125. What is the sum of the terms in a 30-term arithmetic sequence with 125.
first term 19 and last term 81?

(}6
126. Find the sum of all 100 terms in an arithmetic sequence that has first 126 4 ¥ y
term 24 and common difference 4. se
e A f /

127. What is the sum of the arithmetic series 15+21+27+--+207? 127071. “et

Beast
Academy
Practice
5C | 25|
EXAMPLE Groggmakesa patternoutofgumballs,as shownbelow.How oo

jpeg
ou
192207 ‘manygumballsare neededto makethe33"tee in1hispattern? |

@
@
\

Figure
®@

1
@® @ ®@
@ ®@

Figure 2
e@ee
e@ @

Figure
@
®@

The first figure in Grogg’s pattern has 4 gumballs. Then, each figure has 3.
more gumballs than the figure that came before.

] Dee 1 figure 1 Figure 3

The numberof gumballsin each figureofGtogy s pattern makes an


arithmeticsequencewith firstterm 4 and commondifference3.
So, Grogg’s33” figurewillhave 4+32(3)=4+96 = 100 gumballs.

PRACTICE | Answer the following questions about the gumball pattern below.

@
@ @
@ @ @
BOS eeeee@ 0000008
S 3 8
® ®@
®
Figure 1 Figure 2 Figure 3 J

128. How many gumballs are needed to make the 8" figure in this 128.
pattern? :

129. Write a simplified expression for the number of gumballs needed to 129. 2
make the n" figure in this pattern. _

130. There are 221 gumballs in the k* figure of this pattern. What is k? 130. k=

Beast Academy Practice 5C


PRACTICE Answer each question about gumball patterns below.

131. The first four figures in a gumball pattern are shown below. How many
total gumballs are needed to make the 30" figure of this pattern?

Figure 1 Figure 4

132. How many gumballs are needed to make the 10" figure in the pattern
%&~~shown below?

Figure 1 Figure 4

1Me The k**figure of the pattern below uses 420 gumballs. What is k?

Figure 1 Figure 4

134.‘Inthegumball
pattern
below,
somegumballs
are-black
andsome’
G
*' gumballsaregray.Writeanexpressionforthetotalnumber
|of
gumballs in the n" figure of this pattern.

Figure 1 Figure 4

Beast Academy Practice 5C


IntheFibonacci
sequence,
thefirsttermis1andtheseouldfermi
is”Le
Afterthat,everytermis the sum ofthe twopreviousterms.

aIDDOUCIF
|oly
14
/.:Due7b2S EXAMPLE
| ListthefirstfivetermsoftheFibonacci
The first two terms
the 3" term
ae
of the Fibonacci
in the sequence,
sequence are 1 and
we add the 1* and 2" terms:
ce. :

1. To find
1+1 =2.
Squawk.
Fib-uh-
nah-chee.
Then, the 4" term is the sum of the 2" and 3° terms: 1+25 3.
Lobes

Finally, the 5" term is the sum of the 3 and 4" terms: 2+3 =
ag
So, the first five terms of the Fibonacci sequence are ~ 5. oo
1,123.5...

PRACTICE| Answer each question below.

135. Fill in the blanks below to list the first 10 terms of the Fibonacci sequ ence.

136. The 19" and 20" terms of the Fibonacci sequence are 4,181 and 6,765. 136.
What is the 18" term of the Fibonacci sequence?
_

Isthe 100"termofthe Fibonaccisequenceoddor even?


gui

sehie449
Grogg
in which
instead
writes
each
a sequence

of starting
term

fourth term in Grogg’s sequence


of terms
is the sum
with
similar
of the two terms
1, the first term
to the

in Grogg’s
Fibonacci
before
aeBI
sequence,
it. However,
sequence
is 76, what is Grogg’s second term?
is 18. If the

Beast Academy Practice 5C


PRACTICE | Answer each question below about Hoppy the coilbeast.

Hoppy the coilbeast is hopping upa flight of


Stairs. He can hop up 1 or 2 steps at a time.

ee

139. Hoppy can reach the 1° step in 1 way: by taking a 1-step hop from the base. 139. C.
Hoppy can reach the 2™step in 2 ways: by taking a 2-step hop from the
base, or by taking two 1-step hops from the base.
How many ways can Hoppy go from the:base ofthe steps to the 3° step?

140. How many different ways are there for Hoppy to go from the base of 140.

the
steps
tothe
4"step? g
hop As |
To reach the 5" step, 'Hoppy can hop to the 4" step.and take a 1-step 141. we
hop from there, or he can hop to the 3" step and take a 2-step hop.
How many ways can Hoppy go from the base of the steps to the 5" step?

142. Howmany ways are there for Hoppyto get to the 6", 7", 8'",9!" and 10" steps?
The firsttwo entries have been filledforyou, and you can useoo answers above
forthe 3°, 4", and 5" steps,

143. How many ways are there to arrange 10 dominos to form a 2-by-10 143. iE
we rectangle of dominos? Two possible arrangements are shown below.
*

fr Pomaprtpe
pane
foemp
| ef
Beast Academy Practice 5C
For the problems below, use the given rule
PRACTICE
to fill in the blanks of each sequence.

144. Each term is half the product of the two previous terms.

Each term is the opposite of twice the previous term.

AN 0 40HA3.20
146. Eachtermisthereciprocal
ofthesumoftheprevioustwoterms.
\ Wa 61 Mba
(

.
147. Each term is the square of the sum of the previous term’s digits.

«=U % SS\S
-
—————)
ad
ee
,8 . Ves - 7
Brae
*e X »
é3 .oc% bad 2 #. 4
7 , ' ®
° .

148. Each term is the sum of the squares of the previous term’s digits.

ae
149.
*%
Leodore writes a sequence whose
that, each term is the sum of all previous terms. Write an expression
that correctly gives the n" term of Leodore’s sequence for all values
first two terms are O and 1. After
:
of n that are greater than 2.

| 30| Beast
Academy
Practice
5C
PRACTICE | Use the rules given below to answer the questions that follow.

lO get the next term in a Lizzie sequence, reverse the digits of the current term and
atid 1 to the result. For example, the term that comes after 52 is 25+ 1 = 26.
«

aria
150.

» 56OO NWOB89$5.0
To find the next term in an Alex sequence, take the two previous terms 6 subtract the
smaller one from the larger one (order does not matter if the terms are equal).
For example, if the first two terms are 45 and 60, the third term is 60-45 = 15.

151.

Fill
in
t he
missing
a.85, terms
in
e o
Alex
sequence
below.
52,1e oeAN BLaa *
152. What is the 75" term of an Alex sequence whose first two terms are 152.
* 5 and 3?

193 Fill in the blanks


‘term is Ter
below to create
an the one before
a 10-term
my,
Alex sequence in which every

a a eG

154.
BIHAWES
Ralph makes a sequence that begins with a 2-digit number. After the
AAI | 154.
* first term, each term is 9 times the sum of the digits of the term before it.
How many different numbers could be the 4" term in Ralph’s sequence?

Beast
Academy
Practice
5C | 31|
Lookfor patierns ~
that willhelpyousolve
these Problems.

PRACTICE| Answereach question below.

155. What is the units digit of 3°9?

156. Compute the sum of the powers of 2 listed below: . 1c ; 156.

14+24+44+8+164+32+64+128+256+512+1,024+2,048+4,096

he
%

157. Ifthe patterncontinuesbelow,undefwhichletterwillthenumber99. 157.


appear? ¥
AB Cc DEF
ti-2 sae hore

12°141 OnOnBagel
To (aos ont 10

24 2ei22 21 20619

158.Ifthepattern
continues
below,
whatnumber
will
beqecty)‘above
the , Sc Ye
a
number 100? ; : ae:
‘ 21-22-23-24-
---
20 7—8—9—10
1 6 T—2.11
18 Las 1b
17-16-15-14-13

| 32| Beast
Academy
Practice
5C
PRACTICE
| Answereach questionbelow.

159. What is the remainder when 2'” is divided by 7? 159. ve

160. A compubot has’ a button marked *%.When an odd integer is


displayed, pressing * multiplies the number by 3 and adds 1. When
an even integer is displayed, pressing * divides the number by 2. If
the compubot oe 3 now, what will it display after the * button is
p ressed 75ti mess, eer a es
pe a la 4112%s
2|

161. Grogg has two 2-cup juice boxes. The first box is fulland the secondis 161. /
empty. Grogg pours half of the juice in the first box into the second.
Then, he pours one third of the juice in the second box into the first.
Then, he pours one fourth of the juice in the first box into the second.
Then, he pours one fifth of the juice in the second box into the first.
Ifhe continues to pour juice from one juice box to the other this way, how
many cups of juice will be in the first juice box after Grogg’s 99" pour?

Beast
Academy
Practice
5C | 33|
PRACTICE| Answereach questionbelow.

162. What is the smallest positive number that appears in the infinite 162. |
arithmetic sequence that has first term -999 and common difference 8? S:

-999, -991, -983, -975, -967, ...

163. Write the next five numbers in the following sequence:

1,10;
11,*#O0
101
nele10}
A, eS We
get
* = «*G 2

164. The sequence below is the list of positive integers that are not perfect 164.
% squares. What is the 50" term in this sequence?

2; 30) (On04-0. On

= The sequence below is made by adding 1, then 2, then 3, then


repeating this over and over. What is the smallest 7-digit number
ilo
that appears in this sequence?

0,1, 3, 6, 7; 9,12; 4S St SiG yee,

Beast Academy Practice 5C


PRACTICE
| Answereach questionbelow.

166. What is the greatest possible term that can appear in an arithmetic 166.
sequence of 7 positive integers whose sum is 133?

167. In a Tribonacci sequence, each term is the sum of the three


previous terms. Consecutive terms in a Tribonacci Sequence are
shown below. What is x?

xX,y, Z, 44, 81, ...

168. The first three figures in a toothpick pattern are shown below. How
* many toothpicks are needed to make the 20" figure in this pattern?

nay ira aX

169. Bronkle writes the arithmetic sequence 40, 46, 52, ... .
* Gergum writes the arithmetic sequence -50, -41, -32, ....
The k" term of Bronkle’s sequence is equal to the k" term of
Gergum’s sequence. What is the k" term of both sequences?

Beast Academy Practice 5C 3


4—
=
e—
o

oaeSe
te

Use this Practice book with


Guide 5C from BeastAcademy.com.

Recommended Sequence:

Book — Pages:
Guide: 42-51
Practice: 37-47
Guide: 52-57
Practice: 48-57
Guide: 58-63
Practice: 58-63
Guide: 64-69
Practice: | 64-73
You may also read the entire chapter
in the Guide before beginning the
Practice chapter.
: Payattention fo”
theorder o fthe©
ter s ina ratio.

PRACTICE Answer each question about the


collection of shapes below.

1. What is the ratio of black circles to white circles?

2. Whatistheratioofcirclestotriangles? 2.

3. What is the ratioof blackshapes to whiteshapes? 3. 5 to a


*
: >
2

4. What is the ratioof whitetrianglesto blacktriangles? 4. ue

5. How many of the circles below must be shaded so that : SN me :


there are 2 shaded circles for every 1 unshaded circle? 1{

Beast Academy Practice 5C Guide Pages: 44-51 | 37


Katios
Ratios are usually written with the quantities separated by a colon.
For example, a 2-to-3 ratio of apples to oranges is Heal written 2:a

Ratios are almost always written in simplest form, meaning the quantities
in the ratio are integers that do not have any common factors greater than 1.

Mr. Underhill has 6 cats and 8 dogs. What is the


EXAMPLE ratio of cats to dogs, in simplest form?

The ratio of cats to dogs is 6:8. However, since 6 and 8 have a common
factor of 2, this ratio is not in simplest form.
We can split the animals into two groups, each with 6 2=3 cats
and 8+2=4 dogs. So, there are 3 cats for every 4 dogs.
The greatest common factor of 3 and 4 is 1, SOwe cannot
simplify any further. The ratio of cats to dogs iis3:4. -
We can write 6:8 = 3:4.

PRACTICE| Express each ratiobelowin simplestform.

b 197:197=

:gn28I</ 9.ndotwane
e-
ep J ZI"ie
10.
ae ae 1. 2188=__y_g

12.
24:92
=4,27 S 13.a 15
5519,
ye ex VN
3°5

| 38 Guide
Pages:
44-51 Beast
Academy
Practice
5C
PRACTICE | Answer each question below. Write all ratios in simplest form.

a t
16. Ted has 16 blue toy cars and 10 red toy cars. What is the 16. Ops
ratio of blue cars to red cars?

17. Inaclass of 30 students, 26 of the students have brown eyes, and Wiew fi
the rest have green eyes. What is the ratio of brown-eyed students to
green-eyed students?

18. In 100 flips of a coin, 56 of the flips land heads. What is the ratio of
heads flipped to tails flipped?

19. _ The ratio of the side length of a small square to the side length of a 19. 7
larger square is 3:5. What is the ratio of the area of the small square
to the area of the large square? a 5

3
E

. 4 r 2% UE ¢.
20. _ Raquel makes a smoothie with = of a cup of yogurt and s of a cup of 20.
strawberries. What is the ratio of cups of yogurt to cups of
strawberries?

Beast
Academy
Practice
5C Guide
Pages:
44-51
| 39
Theratioofchickene
tocowsonapio}is 2to
EXAMPLEthere are 54 chickens,howmanycows arethere 2

Theratiotellsusthatforevery2 chickensontt farm,there are—


3 cows.So,wecansplitthechickensae COWS intoRie each
with2 chickensand3 cows.
Since there are 54 chickens,wecan make5A
chickenseach. Eachof the 27 groups also ha
So, there are 27 +o=81 cows.

PRACTICE | Answer each question below.

21. The ratio of dragons to yetis in a dance class is 5 to 3. If there 21. |


are 30 dragons, how many yetis are there?

22. Atamath tournament, the ratio of Beast-Academy fans to Orb 22.


Academy fans is 5 to 4. If everyone at the tournament is a fan
of just one school and there are 160 Beast Asam fans, how
many Orb Academy fans are there?
=

one &4,
23. The ratioof grape sodas to orange sodas sold froma soda 23. ; 4
machine last week was 7 to 6. If 84 orange sodas were sold, how
many grape sodas were sold?

Guide Pages: 44-51 Beast Academy Practice 5C


PRACTICE
| Answereach questionbelow.

24. On Priti’s last history test, she answered 2 questions incorrectly 24.
for every 5 questions that she answered correctly. If Priti had 30
correct answers, how many total questions were on the test?

Roland baked sugar cookies and oatmeal-raisin cookies in 25. f ye

25.
a 4-to-3 ratio. If Roland baked 36 oatmeal-raisin cookies,
how many cookies did he bake all together?

Allison makes green paint by mixing blue paint with yellow paint in a 26. |
2:3 ratio. How many pints of blue paint must Allison mix with + pint of
yellow paint to make green paint?

On a recent
traveled
trip,
by bus. He also
Peter traveled
biked
7 miles by train for every
1 mile for every 14 miles he took a
3 miles he
27.
Z
bus or train. If he traveled 210 miles by bus, how far did he bike?

Anna has 40 white beads and 90 black beads. She wants to make
28. Ne
a necklace in which the ratio of white to black beads is 3 to 7. What
is the greatest number of white beads she can use to make the
necklace?

Beast Academy Practice 5C Guide Pages: 44-51 a1


There are many
ways to reason In many situations, it \
through problems helps to consider how the
involving ratios. parts in a ratio relate to
the whole amount.

EXAMPLE The ratioofsoda to juicein Daryl'sfruitpunch recipeis3 to 7. Howmuch


soda and juice does he need to make 50 fluid ounces of fruit punch? |

For every 3 fluid ounces of soda in the fruit


punch, there are 7 fluid ounces of juice. So, in
every 10 fluid ounces of punch, 3 fluid ounces
are soda and 7 fluid ounces are juice.
A 50-fl-oz pitcher holds 50+10=5 servings of 10 fluid ounces, each with 3 fluid :
ounces of soda and 7 fluid ounces of juice. So, Daryl needs 3 -5= 15 fl oz of soda
and 7 -5=35 fl oz of juice.

For every 3 parts soda, there are 7 parts juice, for ia


total of 3+7 = 10 parts.
Ifwedivide50 fluidouncesofpunchinto10equal Vadeedledhedbadfedlel ae
parts, each part is 5 fluid ounces. So, the 3 parts of
soda equal 3 -5=15 fl oz of the punch, and the 7
parts juice equal 7 *5=35 fl oz.
oe = Of —
In a punch that is 3 parts soda and _ parts juice, 3 of the 10 oa are soda and 7 of the
10 parts are juice. In other words, =—~
0 of the punchieesoda, and =>
* of the punchis juice.
aC to make 50 fluidounces of punch, Darylneeds a -50=15 fl oz of soda and
- 50 = 35 fl oz of juice.

3_-50=15 floz
—oO

50 floz
| 7 parts juice 1us e 50 —
= 35 fl oz

PRACTICE
| Answer
thequestion
below.
29. Every blorble has 3 long horns and 4 short horns. There are 29. :
98 horns on all of the blorbles in a field. How many of those
horns are short?

Guide Pages: 44-51 Beast Academy Practice 5C


PRACTICE
| Answereachquestionbelow.

30. Ata basketball game, every player is wearing a white or blue jersey.
The ratio of white jerseys to blue jerseys is 4 to 5. How many of the
27 players at the game are wearing white jerseys?

31. Ina junior Beastball league, each team has 3 defenders


and 5 strikers. How many of the 96 players in the league are
defenders?

32. Edna bakes 240 cupcakes and arranges them onto trays so that 32.
each tray holds 4 chocolate and 6 vanilla cupcakes. How many
vanilla cupcakes did she make?
Ty

33. .*Azoo has an exhibit with spotted stingravens, striped stingravens, and 33.
x. pandakeefts. The ratio of spotted stingravens to striped stingravens is
+ 3:5, and the ratio of stingravens to pandakeets is 2:9. If there are 176
.* animals inthe.exhibit, how many are striped stingravens?

soe en
oo %e a

34. Arectangle has a width-to-height ratio of 5:2. The perimeter of the


* rectangle is 210 inches. What is the area of the rectangle, in square
inches?
at
¢ ¢

Beast Academy Practice 5C


EXAMPLE
| Which
ratiobelow
iisequivalent
tg . |
4:18,22.ep 24:94,ae 98.

Equivalent
ratios
have
thesame
simplest
o. .
Insimplest
form,
wehave .
4:18=2:9,22:88=1:4,
24:94
=12:47,
and21:98
=8:14,
Since9:36=1:4and22:88=14, weeve 9:36=oe oe
So,eet88isequivalent
to=oo

PRACTICE | Answer each question below.

35. Circle the ratio below that is not equivalent to the other four.

21:28 63:84 39:52 51:68

36. Circle the ratios below that are equivalent to 11:17.

cho 44:68 () 55:85 )55:88


“4 66:10

37. Use
the8ratios
below
towrite
four
pairs
ofequivalent
ratios. EN OY
21:35 14:24 32:48 56:96
pre f BD
a
Fy
55:90 33154 “12:20 18:27

38. Use the numbers 6, 9, 28, 30, 42, and 140 to write three equivalent ratios.

6 a aac / ¢h byo
ra
SS |
Guide Pages: 44-51 Beast Academy Practice 5C
PRACTICE
| Answer
each
question
below.
39. Fill the missing entries in the chart below so that the ratio of milk to butter in each

column
is
t he
same.
orn ee
ounces of butter
= Ot ee fr
as

40. There are 18 pigs and 24 goats in a pen. Nine pigs and some goats 40.
enter the pen, but the ratio of pigs to goats does not change. How
many goats entered the pen?

41.. wt Ee
41. Captain Kraken has 36,gold coins and 63 silver coins in a satchel.
After spending some of-the coins, Captain Kraken sees that the ratio ©

of gold coins to silver Coins has not changed. If Kraken spent 8 gold
coins, how many silver coins did he spend? :

4,
Grogg mixes of a cup of cornstarch with “ of a cup of water to make 42.
15 cups of oobleck. If he adds 1 cup of cornstarch to the mixture, how =” : 7
many cups of water shouldthe add to maintain the ratio of cornstarch to
waterinhisoobleck? . j

| .
Beast Academy Practice 5C Guide Pages: 44-51
In a Rectivide puzzle, the goal is to divide a single rectangle into three smaller
» rectangles so that each small rectangle has the same ratio of gray squares to white :
squares as the original rectangle. : | _. fe


Ren EXAMPLE | Solve the Rectivide puzzle below.
Q
0%

The ratio of gray squares to white squares in the original rectangle is 4:8 = 1:2.
So, we can make groups of 3 squares in each of the smaller rectangles, with 1 gray
and 2 white squares. Therefore, the area of each small rectangle is a multiple of 3.

We can split the rectangle into three smaller rectangles as shown below so that the
ratio of gray squares to white squares is 1:2 in each smaller rectangle. :

This is the
only solution.

43.
j KAN
Yt
Aus)
ASRKy

45. 46.

Beast Academy Practice 5C


| 47. 48.

_ 52.

54.

_ Beast Academy Practice 5C 47


Weoften usefractions
whenworking
withratios.
Forexample aboy:oirratio

SUOIQIOD
of2:3canbewrittenas boys_2 :
girls3°
Thismeans thatthenumberofboysdivided
bythenumber offayequals =2
Aproportion isanequation
showingthattworatios
areequal.
Tetearemany
1 ways
tofindthemissingvalue
inaproportion. : _
Tosolveforxintheproportion
4:7=x:21,
wecansolvedaeinihead: as
EXAMPLE
| Whatisthevalueofx inthe equationbelow?
sae
21

Wecanconvert thefraction.
Wecanwrite:44witha denominator
ofot by aS
multiplying
thenumerator
anddenerinatoy
by3. fo yel
4 (12
a== 577 $0 X= 12.
— OT

Wecanisolate
thevariable. 4 | x| |
.Toisolate
thevariable
x,weees: bothsidesof
theequation
by,21. _ |. or =oFo1
Onthe left side, weget 4-a6 12. So, x= 12. ‘s

PRACTICE| Fillin the missingvalue in each equation below.

55. Wy 56.Breei 57.L P_7


32 8/* 58.i ee
PRACTICE| Solveforthe variablein each equation below.

59. aks
4 60.

61. lh
=75 | 62. m_9
57=] 61.wx49

Guide Pages: 52-57 Beast Academy Practice 5C


:oe Whatisthevalue
ofxintheeneiorbelow?

Wecaneliminate
theGenchincics.
We eliminatethe denominatorsof =
> and =~= by
multiplying
bothsides ofthe equeten bya common
multiple of their denominators: 9x.
This gives 5x = 72.
We divide both sides by 5 to get X=

For
anyequation
]=S
wehavead =be.

Solve for the variable in each equation below.


PRACTICE Write your answer in simplest form.

63.S== 64. 63.

64.

65.7 == 66. 65.

66.

67.

68.

69.==4 70. 69. n=

70. 7

Beast Academy Practice 5C Guide Pages:


ym
OO
ei M
Katie makes purple paint by mixing blue paint and red
EXAMPLE | paint in a3:5 ratio. How many ounces of red paint should
she mix with 8 ounces of blue paint to make purple paint?

The ratio of blue paint to red paint is 3:5.


The ratio of 8 ounces of blue paint to ounces of red paint (7) is 8:7.
Since we want 3 parts blue paint for every 5 parts red paint, we have 8:r= 3:5.
2... |. : S_ 3 <— bluepaint
We express 8:7=3:5 withfractions. ; 5 <— redpaint
Multiplying
bothsidesof=
Ae 3 by5rgives 5. b= 3. Br :
40=3r. * oe Manyproblems
Then,ovens bothsidesa theies 7 ae a. _[—involving
ratios
canbeesved6)
by3gives=
ep, SO,foe 133. : : 3s writing
anequation
F and solving for an
So, Katie should mix 13+ ounces of red ae | unknown value!
paint with 8 ounces of blue paint.
Or

We can instead use the ratios of red paint to blue paint: 7:8 = 5:3.
Then, we have
5 <— red paint
fl.
[= 8<—bluepaint
Multiplying
both s idesP y8gives
r=2= =134.
So, Katieshould mix131 ounces of a saint with —
8 ounces of blue paint.

PRACTICE Solve each problem below. Express all non-integer


answers as mixed numbers in simplest form.

71. » Kristen writes a sentence that has 5 consonants for every 4 vowels. If
her sentence has 40 consonants, how many vowels does it have?

pela
72 . | 10 makefudge,Coriuses 9 cups ofsugarforevery2 cups ofcocoa. 49)
If she uses 5 cups of sugar, how many cups of cocoa should she
use? #

73. The ratio of the width of a Blorgbeast’s hoof to the height of the
Blorgbeast’s hoof is 3:8. How many inches wide is a Blorgbeast hoof
~* that is 5 inches tall?

Guide Pages: 52-57 Beast Academy Practice 5C


PRACTICE Solve each problem below. Express all non-integer
answers as mixed numbers in simplest form.

74, Hummingbeast nectar is mixed in a ratio of 2 parts sugar for every 74,
7 parts water. How many ounces of sugar should be added to 16 r
ounces of water to make hummingbeast nectar?

yi Terry hits a home run 2 out of every 7 times that he hits a baseball. If eS: Z
he hit 28 home runs this season, how many times did he hit the ball
but not get a home run?

76. The ratio of the area of isosceles right triangle ABC to the area of right 76.
triangle DEF is 3:5. How many inches long is side ED? V

ae D

B6in C

77. The ratio of the weight of a biffoto the weightofa triffois 2:3. If agroup 77.
of 30 biffos weighs 40 pounds, how many pounds does a group of 30
triffos weigh?

Will writes two numbers, a and b. The ratio of a to bis 3 to 5. The 78. a
sum of a and b is 100. What are a and b? | 7'hy

The ratio of goals scored by Ben to goals scored by Alfie last season 79.
was 5:4. If Ben scored 16 more goals than Alfie last season, how
many goals did Alfie score?

Beast
Academy
Practice
5C Guide
Pages:
52-57
| 51|
Twoshapesaresimilar
ifoneshapecanbeflipped,
rotated,
_
orshrunk
toexactly
match theother.
0 Similarshapes have sides that are proportional. This means that.
. the ratiosofthe lengthsofthe correspondingsides are the same.
%
as The rectangles below are
EXAMPLE — the missing side length.
similar.Find
2
=< ye

20
12

Theratiooftheshortsideofthefirstrectangle
totheshorta e the |
secondrectangle
is1:12.
The ratioof the long side of the first fea to the long side of the -|
second rectangle is 25:20.
The
rectangles
aresimilar,
soh:12=
25: means8
20.This ee
Te forh,wehave
is 15. _ 12.
20

PRACTICE | Find the missing side length(s) in each pair of similar shapes below.

80.

ee | 81. 80.w=pes
8 21 81.&

82. 83. 82.


-@

| 52| Beast
Academy
Practice
5C
PRACTICE
| Answerthe questionsbelow.

84. The ratio of the edge length of a small cube to the edge length of a
large cube is 5:7. What is the ratio of the surface area of the small
cube to the large cube?

85. Chris has a model train set in which all the pieces are built to scale. This
means that the ratios of the sizes of his models to the sizes of the actual
objects are always the same. His 35-inch-tall station models a real
station that is 50 feet tall.

a. One of Chris’s model train cars is 6 inches long. How many


feet long is the actual train car?

b. How many feet long is Chris’s model of an actual track that


the is 12,000 feet long?

86. The map below models the distance between four cities around Route 43 and
Route 2. Yetiville and North Pegasus are 25 miles apart, which is represented
on the map bya distance of 3 inches.

a -Yetiville North
Pegasus
Unicorpia
e

Griffinburg

a. Griffinburg and North Pegasus are 24 inches apart on


* this map. How many miles apart are these two cities?

3 b. The town of West Mermaid is 40 miles from Unicorpia. If West


* Mermaid were included on this map, how many inches away
from Unicorpia would it be?

Beast Academy Practice 5C


A bouquet of flowers contains 4 roses,
EXAMPLE 8 daffodils, and 14 sunflowers. What is the — can.
ratio of roses to daffodils to sunflowers? ‘write ratios —
to describe the
relationships —
The ratio of roses to daffodils to sunflowers is between more
than two
4 to 8 to 14 or 4:8:14. Since 4, 8, and 14 all have quantities.
a common factor of 2, we simplify this ratio:
4:38:14 = 2:4:7.

_ So, we can make groups of 13 flowers, each


with 2 roses, 4 daffodils, and 7 sunflowers.

Remember that a ratio is in simplest form


when the greatest common factor of all terms
in the ratio is 1. :

PRACTICE | Solve each problem below. Write all ratios in simplest form.

87. In a room of 93 students, 18 are wearing red shirts, 30 are wearing


blue shirts, and 45 are wearing yellow shirts. What is the ratio of red
to blue to yellow shirts?

88. Sixty students are chosen to march in the Beast Academy winter
parade. Fifteen of the students are third graders, 14 are fourth
graders, and the rest are fifth graders. What is the ratio of third to
fourth to fifth graders?

Ernie has three types of fish in his tank: guppies, goldfish, and 89.
89.
angelfish. The ratio of guppies to goldfish to angelfish in the tank is
8 to 5 to 3. What fraction of the fish in Ernie’s tank are goldfish?

Rolfe grows peaches, Fuji apples, and Gala apples in his orchard.
90.UZ7g
The ratio of Fuji to Gala trees is 3:5, and the ratio of apple to peach
trees is 4:3. What is the ratio of Fuji to Gala to peach trees?

Beast Academy Practice 5C


PRACTICE | Solve each problem below. Write all ratios in simplest form.

91. A recipe for =e cookies calls for i cups of butter, gt> Cupsof
sugar, and ge3 Cupsof flour.What is the ratio of bitter to sugar
to flour?

92. The ratio of length to width to height of a rectangular prism is


5:6:8. If the prism is 60 cm wide, what is the surface area of the
prism, in square centimeters?

93. The ratio of red to blue to yellow marbles in a bag is 1:2:3. If


there are 16 blue marbles, how many marbles are in the bag?

At Hoppy Pond, the ratio of frogs to toads is 3:5, and the ratio of
green frogs to yellow frogs is 3:2. If every frog is either green or
yellow, what is the ratio of green frogs to yellow frogs to toads?

The ratio of a to b to c is 2:9:3. If the sum a+b+c is 70, what


is the product abc?

The ratio of length to width to height of a rectangular prism is 96.Rec n


HS 2:5:8. If the volume of the prism is 2,160 cubic inches, what
are the length, width, and height of the prism? ‘wiath:“7!
A.
a?
“4 e eign:27A_J

Beast Academy Practice 5C


[55|
InaRatiotile
puzzle,
theSoa)
istofillevery
square
andmendsothat
ek
Gd
ed e each triangle contains a positive integer, and
* each square gives the ratio of the smaller number to the
larger number in the adjacent triangles, in simplest form. — _.

EXAMPLE| Completethe Ratiotilepuzzle to the right.

Se
The ratio of the triangles that touch the right square is 2:5.
“6
If 10 is the larger of the two numbers, we have [4]:10 = 2:5.
If 10 is the smaller of the two numbers, we have
10:[25]= 2:5. So, the value in the center triangle is 4 or 25.

The ratio of the numbers in the triangles that touch the left
square is 2:3. Since 25 is not a multiple of 2 or 3, there ©
is nointegerthathasa 2:3ratiowith25.So,thecenter|
triangleis4.
No integer smaller than 4 has a 2:3 ratiowith 4. However,
if 4is the smaller number in the 2:3 ratio, we have -
4[6]= 2:3. So, the top-lefttriangle is 6.

Wecheckthat4:6=2:3and4:10=2:5.V

PRACTICE | Solve each Ratiotile puzzle below.

97.

100. ©

Beast Academy Practice 5C


Remember that
each ratio describes the
relationship of the smaller
PRACTICE
| Solveeach Ratiotilepuzzlebelow. numbertothelarger
number!

104.

103.
\“iNy
106.

105.
io
108.

The squares below give the ratio of the numbers in the three
PRACTICE
adjacent triangles in order from smallest to largest.

Beast Academy Practice 5C


Arate describeshowmuchofonequantity therei
_ Weoftenusetheword“per”inrates.“Per”means“forevery”
o
_The following are all examples of rates:

65 milesperhour 5 dollarspergallon 90sheetsper|


19pointspergame 57wordsperminute 21studentsperclas:
10dollarsperweek32 milespergallon 12markersperbox

PRACTICE
| Answer
each
question
below.
te

111. Beasty brand paper towels have 85 sheets per roll. How many sheets are in...

2rolls
rolls?
¢ a1 / 5FOS
rolls?
¢ a; 10fous
rolls?
¢ $70 .. rous
lls?7
¢ bl /

112. Bess’s car uses 5 gallons of fuel to drive 160 miles. How many miles will Bess’s car go
*

using...
1gallon? of
; L ah gallons? a
Z 10gallons? / I“Wo
55gallons?

113. Grogg buys 6 pens at the school store for $2.10. At this rate, what is the cost of...

U, ran
LLetS19mall eatee
114. Arthur's Grocery sells a 16-pound bag of potatoes for $12. Betty’s Hf 1 Es a
Grocery sells a 10-pound bag of potatoes for $8. Which store’s " :
potatoes cost less per pound? ae

Guide Pages: 58-63 Beast Academy Practice 5C


@ ) a De

LLLLMBBM LBS DAE EES

PRACTICE
| Answereach questionbelow.

115. |W Z
115. Benji types his 1,500-word book report in 30 minutes. At this rate,
how many minutes will it take for him to type his 5,000-word history
essay?

116. Forty-five baby dragons were born in the last 9 days on Beast Island. 116. ;
At this rate, how many baby dragons will be born in the next 12 days ;/
on Beast Island?

117. Freddy is baking pies for a bake sale. He buys a 15-pound bag of 48 apples for $18.
He uses all of the apples and 24 ounces of sugar to bake 8 pies. Write each rate below.

ee a,
apples
perpie b. ce ofSugarperpie
4 1
Cc. dollars
perpoundofapples d. If pounds
ofapples
perpie
e. ouncesofsugar
perapple f. IMsous perapple
Ww
Use the following information for problems 118-121:
Mr. Jones exchanges 75 Beastbucks for 300 Torts, the currency of Tortuga Island.

118. ‘How many Torts does he receive per Beastbuck? 118. af Torts per Beastbuck

119. How many Torts willMr.Jones receive in exchange 119. _ 7 d


for +00 Beastbucks?
\J
120. At this rate, when Mr. Jones exchanges 120. _ | Beastbucks per Tort
BeastbucksforTorts,howmanyBeastbucks
willhereceive
perTort? 121. IZ sh |
| a2 1. How many Beastbucks will Mr. Jones receive in
exchange for 480 Torts?
||

Beast Academy Practice 5C Guide Pages: 58-63


Speed is a type of rate that describesa relationshipbetweendistance and tim

p22ds We often express speed as the distance traveled for one unit of time. For example, one
common unit of speed is miles per hour (mph), which describes how many miles are
traveled in one hour.

Fiona can sprint 200 meters in 25 seconds.


EXAMPLE | What is Fiona’s speed in meters per second?

_ dist
_ Fiona’s speed is the ratio of the distance she ranto Speed an ee
~ the amount of time it took her to run that distance. _ 200 meters : .
Since Fiona can run 200 meters in 25 seconds, she | 25 seconds
can run 200+25 = 8 meters in one second. So, her = omen
speed is 8 meters per second.
: : 8 meters per second
Just as with other rates, we may instead write this as
8 meters/second, 8 m/sec, meters mM
8 — ong: OF8 se:

Speed problems in this chapter always refer to average speed, the ratio
of total distance to total time. For example, in the problem above, Fiona
probably wasn’t always running the same eee but her average speed
was 8 meters per second. ]

PRACTICE | Answer each question about speed below.

122. The Beast Island activity blimp travels 90 miles in 3 hours. What is 122.
the blimp’s average speed for the 3-hour trip in miles per hour?

123. In one full day, the kudzu vine can grow 15 inches in length. How 123.
* many inches per hour is this?

124. Allison threw a baseball that traveled 145 feet in 2+ seconds. What 24. D
was the average speed of the ball in feet per second?

125. “Robbie ran 15 miles in 12 hours. What was Robbie’s average speed 125.
3/
in miles per hour?

Guide Pages: 58-63 Beast Academy Practice 5C


PRACTICE Answer each question about speed below.

126. Erica runs 4s meters per second. At this speed, how many meters

can she run in 45 seconds?

127. Roland’s remote-control boat has a maximum speed of 25 miles


per hour. How many miles can Roland’s boat can travel in 5 of an
hour?

128. An 8-kilometer hike up Yeti Mountain takes Grogg 3 hours. The hike
back down on the same trail takes him just 2 hours. What is Grogg’s
average speed for his entire trip in kilometers per hour?

129. Allison’s rowing team rowed 5 miles in = of an hour, then rowed 129.

another 3 miles in +5 of an hour. What was their average speed for


the entire trip in miles per hour? ..

130. Annalise can walk 10 miles ir4 hours. At this rate, how many 130.
minutes does it take her to walk one mile?

131. Urlich can paddle a kayak 5 miles per hour. At this speed, how
©”

131.
Gry
many minutes will it take him to travel 4 miles across Beast Bay?

Beast Academy Practice 5C Guide


Pages:
58-63
61
f3 moles
candig10honein20 minutes,
howmany
“EXAMPLE
‘minutes
willittakefor2molestodig5 holes?
a Weaassume, thatallofthemolesworkatthesamefale
/ Webeginbyconsidering howlongitwilltakes1moletodig1hole.
> If 3 moles can dig 10 holes in 20 minutes, itwilltake 1mole three times as longto
| completethe job. So, 1 molecan dig 10 holes in 3 +20=60 minutes. :

ialmolecandig10holesin60minutes,
itwillonlytake<>
9aslongtodig1hole.
Se 1meecandig1holein60a5 6minutes.
Then,
weusethe4-mole-
1“hole
t ime
tofind
howlong
ittakes
2molstoaoeholes.
If1molecandig1holein6minutes,
itwilltakeco}
timesas longtodigSby s
: oe 1molecandig5holes in6 5 -_
- If1molecandig5holes
|in30|mint
— takeoe asjong:
So,oe 2scandi
Rate problems can ~ For these
problems, you
/ be pretty tricky when may assume that
_ we have more than one each worker is
.. worker completing a working at the
: task. | same rate!

PRACTICE| Fillinthe blanksto answer each question below.

132. Five mowers can mow 4 football fields in 60 minutes. Use the reasoning below
to figure out how many minutes it will take 3 mowers to mow 5 fields.

a. Onemower
canmow
4 football
fieldsinad Minutes.
b. Onemower
canmow
1 football
fieldin TW tee
C. Threemowers
canmow
1 football
fieldin Lh Vminutes.
d. Three
mowers
canmow
5football
fields
in ig Soe
meno
133. Six hoses can fill 5 buckets in 15 minutes. Use the reasoning below to
figure out how many buckets can be filled by 3 hoses in 24 minutes.

a. Six hoses can fill 1 bucket in minutes.

b. Six hoses can fill g v buckets in 24 minutes.

CG: Three hoses can fill vi buckets in 24 minutes.

Beast Academy Practice 5C


PRACTICE
| Answereachquestionbelow.

134. Three gazellephants eat 3 buckets of peanuts in 3 minutes.

a. How many minutes would it take 6 pons ta to eat 6


buckets of peanuts?

b. How many buckets of peanuts can 6 gazellephants eat in 6


minutes?

135. Twelve paint-bots can paint 20 houses in 9 hours.

a. How many paint-bots are needed to paint 5 houses in 9 hours?

b. How many houses can 9 paint-bots paint in 6 hours?


*

136. Ten carpenters take 18 days to build 8 sheds.

a. How many days will it take 10 carpenters to build 12 sheds?

b.
*
Howmany sheds can 3 carpenters buildin 15 days? ane

137. Six lumberjacks can chop 5 logs into firewood in 40 minutes.


Dy
a. How many logs can 24 lumberjacks chop in 30 minutes?

How many lumberjacks are needed to chop 6 logs in 72 minutes?


b. tJ
tea
Beast Academy Practice 5C
andfinally pint
to ounces.
gallonis4 quarts. ,

PRACTICE| Answereach question below.

138. A mile is 5,280 feet. How many feet are there in 3 miles? 138.

139. There are 3 feet in one yard. One foot is 12 inches. How many 139.
inches are in 20 yards?

140. There are 100 centimeters in a meter. One kilometer is


1,000 meters. How many centimeters are in 30 kilometers?

141. How many minutes are in 1 week?

142. If you can jog 2 meters per second, how many kilometers
can you jog in one hour?

Guide Pages: 64-69 Beast Academy Practice 5C


: Wecanuse conversion factorsto 7 — Cc, ;
convertbetween
units. - a | oo : eee a>
Aconversion
facior
i| saeaceen
|inwhich
thenumerator
anddenominator
use
sateran
wnstobang:equal amounts.
|Barexample,
Ls isaconversion
-tone factor 1noure:quals
because 16. -
ounces.
Similarly,
since16ounces
equals
1pound,
eure
piisaconversion
factor.
Sincethenumerator anddenominator
ofa conversion
factorareequal,every
conversion
factorisequalto1.So,wecanmultiply
ameasurement bya
conversion
factortome geitsunitswithout
changing
itsvalue. _

EXAMPLE| How
many ounces
is6pone? Howmany pounds
sietnd :2 :
Toconvert
6pounds
intoounces,
weusetheconversion oe
factor
“11Tb
82to. _f/Check
thatyour
pees are 16 Oz

cancelthepounds
units,asshown: _ reasonable.
18.02
3 oe oo Since
6lb=6WoeOF=96cOZ. — 2 1pound is16
Toconvert6ouncesintopounds,
weusetheconversion
factor=Le ounces,
6 pounds
ismorepits 16
to cancel the ounces units, as shown: ounces. Similarly,
6 ounces is less

60z=60z
ae than 1 pound.

Complete each calculation below to convert between


PRACTICE
units using the conversion factors given.

143. Use the equation below to find the number of fluid ounces in 15 cups,
crossing out the units that cancel. C

15cups=15cupses =__7 , / fluidounces

144. Use the equation below to convert 60 ounces to pounds, crossing out the
unitsthatcancel BE 1pound Ai ¢
60ounces=60ounces+=O cs pounds

145. Use the equation below to convert from 150 feet per minute to feet per
second, crossing out the units that cancel. ane
| _ 150feet, 1minute _ LK ie
| 150eetiper minuie’=—-e * seconds
eo /_ feet per second

| Beast Academy Practice 5C Guide Pages: 64-69


:1gallon.
4quale

Below, we
convert between \
some unusual
units! —

PRACTICECircletheconversion
factorontherightyoucouldusetocompleteeach
conversionbelow.Then,fillineachequationwiththecorrectnumber.

W
l G.
i
146. 6 nanoseconds
=05 sakes TOaneceeonde 10
nanoseconds
1shake
\ ys ‘
yy)
147. 30fathoms= iv feet 1fath
eee 6 feet
1 fathom

148. 2 nautical miles =


mys
) meters 1nautical
mile
1,852 meters
1,852 meters
1 nautical mile

149. 90 inches = / cubits / waciet 18 inches


1 cubit
\
aS oh

150. (1itosalr
i \ 4,184 joules
1 kilocalorie

4,184
joules
/
K _) ( 0 Wisse 16 drams
151. 6 dramsperfortnight=_|)~ounces perfortnight (| eqns 1 ounce

152. 200bytesperjiffy=_/-\ulWya bytespersecond so0jiies. >) 1 second


100 jiffies

Guide Pages: 64-69 Beast Academy Practice 5C


PRACTICE Draw a line to connect each rate conversion on the left to the conversion
factor on the right you could multiply by to make the conversion.

153. 4 dollars per meter into cents per meter. \ a:000%a)

5,280 ft
154. 5 calories per gram into calories per kilogram. fore

‘ ; ‘ : 1 mi
155. 66 feet per minute into miles per minute. 5.280 ft

: 100 cents
156. 10 cents per meter into dollars per meter. “7
dollar”

; : ; : 1 dollar
157. 50,000 grains per kilogram into grains per gram. 00:cents
a

158. a of a gallon per foot into gallons per mile. of eels


1,000°Bs
g
KX
Complete each rate conversion below using the
PRACTICE conversion factors from the problems above.

159. 50,000 grains of rice weigh one kilogram. 159. S entice


How many grains per gram is this?

160. It takes + of a gallon of sealant to cover one foot of driveway.


How many gallons per mile of driveway is this?

l
161. A bungee cord costs $4 per meter. 161. ae aT
How many cents per meter is this?

162. Kelly Crab can scamper 66 feet per minute. ; a mi/min


What is Kelly’s speed in miles per minute?

Beast Academy Practice 5C Guide Pages: 64-69


~ Converting
a
Conversions ce | : . rate can require
| convertingmultipleunits©
A vine grows at a rate of 168 inches per a inthe numeratorand /
EXAMPLE denominator. :
week. How many feet per day is this? -
168in
Tobegin,wewrite168inchesperweekas afraction:
1week’
Then, we must convert from inches to feet, and from weeks to days.

to _168in
convertfrominchesto feet,we multiply————
1 week
OV
in|
an Theinches
units
cancel:
:
1689F iff . 14ft
168inches
perweek= 1week 1247 Oe
7 week. =14feet
petthin
Then,
toconvert
from
weeks
todays,
wemultiPly
+Week
byw
17 days*
=eekThe
week
units
scancel 14h.
week

_ - i4tty“wee
a feetpereer =Fuserdae
-or-

We can perform both conversions at once:


AGB
ier| 1ft”tweet _
168inches
perweek
= Tweek
12- Tdays
7 ~
Youprobablycouldhavemadethisconversion
wiltiout
henge
conversion
factors.However,
usingconversion
factorsallows—
ustoorganize
ourworkforcomplicated
conversions.—:

PRACTICE| Answereach question below.

163. A garden hose can fill a 30-gallon fish tank in 8 minutes. Use the equation below to
determine how many fluid ounces flow from the hose every second, crossing out the units
that cancel.

/
-y

30gallons
in8minutes
=oa. aan ;on = /_/ fluid
ounces
persecond
164. Joe uses a total of 3 cups of flour to make 2 batches of cookies. A cup of flour weighs 120
grams, and Joe makes 12 cookies in each batch. Use the equation below to determine how
many grams of flour are in each cookie, crossing out the units that cancel.

3 cups for 2 batches ___3cups


= D batches’ 120Tepe
grams |_1batch_
eean ee _ / grams per cookie:

Guide Pages: 64-69 Beast Academy Practice 5C


PRACTICE
| Answereachquestionbelow.

165. Circle the two conversion factors below that you could multiplyby to
convert 200 feet per houriinte

166. Robbie the roverbot can travel 200 feet per hour. How many inches
3 ie
per minute is this?

167. Circle the two conversion factors below that you could multiply by to
convert 450 gallons per day into fluid ounces perhour.
1 gal
128 fl oz

168. With machines running all day and night, Barry’s Juice Factory
presses 450 gallons of juice per day. How many fluid ounces of juice
are pressed each hour?

169. Circle the two conversion factors below that you could multiply by to
convert 7 pou per cup into ounces per teaspoon (tsp).

48
t sp
7sp Ke 16 1Ib {or
oz <OZ
|
for]
170. Liquid mercury weighs about 7 pounds per cup. Using this estimate,
how many ounces does a teaspoon of liquid mercury weigh?

Beast Academy Practice 5C


PRACTICE| Answereach question below.
4) U .

171. There are 5,280 feet in one mile. Sam Snake slithers at a speed of 171. L¢
3 miles per hour. What is Sam’s speed in feet per minute? V

172. A length of chain costs 15 cents per inch. What is the cost of this
chain in dollars per yard?

173. Emmanuel can jog 3 meters per second. What is his speed in
kilometers per hour?

174. There are 128 fluid ounces in a gallon. A pipe leaks 45 gallons of water 174. ig
every day. How many fluid ounces of water leak from the pipe per
minute?

175. One serving of Beastie Bites cereal has 14 grams of sugar. A box of 175. T
Beastie Bites contains 8 servings. There are about 28 grams in one
ounce. Using this estimate, how many ounces of sugar are in a box
of Beastie Bites?

176. We can use the conversion factor en to convert from inches 176.
to centimeters. A glacier moves 762 cm per day. At this speed,
how many feet per week does the glacier move?

Guide Pages: 64-69 Beast Academy Practice 5C


PRACTICE
| Answereach questionbelow.

ae If two bears can catch 30 salmon in 54 minutes, 177.


how many seconds will it take for one bear to
catch one salmon?

178. Jeremy and Sarah each have a bag of lemons and limes.
The ratio of lemons to limes in Sarah’s bag is 2:3. The ratio of
lemons to limes in Jeremy’s bag is 1:2. Sarah has 4 fruits for
every 5 fruits that Jeremy has. After all 135 fruits are put in the
same basket, what is the ratio of lemons to limes in the basket?

/ Ly)
i) 179. A right triangle with a perimeter of 60 inches has sides whose lengths
are in a 3:4:5 ratio. What is the area of the triangle, in square inches?
179,
a a

180. Two giant tarantulemurs stand 70 meters apart on a huge web. They 180.
begin crawling directly toward each other. One crawls 2 meters per
second, and the other crawls 3 meters per second. After how many
seconds will the giant tarantulemurs meet?

- Beast Academy
Practice5C
PRACTICE| Answereach question below.

181. The ratio of Jorble’s height to Yorble’s height is 5:9. If


% _Jorble is 10 inches shorter than Yorble, how many inches
tall is Yorble?

182. The ratio of the height of a rectangle to its perimeter is 2:19. What is 182.
% the ratio of the height of the rectangle to the width of the rectangle?

183. The ratio of Alyssa’s biking speed to her running speed is 5:2. What
% is the ratio of the time it takes for Alyssa to bike to school to the time
it takes her to run to school?

Beast Academy Practice 5C


PRACTICE
| Answereach questionbelow.

184. The ratio of blue cars to red cars in a parking lot is 2:3. After 15 blue 184. f.
* Cars leave the lot, the ratio of blue cars to red cars is 1:2. How many
red cars are in the parking lot?

| 185. Everyone seated in the Orb Academy auditorium is either a student


* or a teacher. The ratio of students to teachers is 5:1, and the ratio
of filled seats to empty seats is 4:1. What is the ratio of teachers to
empty seats in the auditorium?

186. Two skateboards are 100 feet apart and rolling directly toward one
another, each at speed of 5 feet per second. Brody Bee starts at
the front of one board and flies toward the other board at a speed
of 7 feet per second. Whenever he reaches the front of a board,
he reverses direction, flying back toward the other board at 7 feet
per second. How far does Brody travel before the two skateboards
collide, with Brody narrowly escaping the collision?

Beast Academy Practice 5C


Use this Practice book with
Guide 5C from BeastAcademy.com.

Recommended Sequence: —

Book _ Pages: :
Guide: 70-77 :
Practice: . 75-81
Guide: 78-82
Practice: 82-91
Guide: 83-101
Practice: 92-107
You may also read the entire chapter
in the Guide before beginning the
Practice chapter.
i
= :edie areanotherwaytowritefractions.
Decimal
place
valuesarebasedonSoe of10,justlikeintegers!
Review
St <a “oO SS A eee decimals basics
S»%
nf x ae 5%
x .xoS .XO
S “sei) xe +9 iinChapter 1 1
Practice4D.of |

EXAMPLE
| Write0.125as a fraction.
0.125has a1 inthetenthsplace,a 2 intheBS eat place,and:a
5 inthethousandthsplace.So,
0.125 = 40 * 100°1,000 In this chapter, we
a0 usually write a zero to
. a 20. 5
the left of the decimal
1,000 “1,000 1,000 © point for numbers less
than 1. However, decimals
125
* 7 000° are sometimes written
without the zero. For .
Then, we simplify: a =7 eas -2°0.2.

| — Of;—_
We can write any decimal with three digits to the right of the decimal

point
asanumber
ofthousandaths.
So,0.125=
“2. =4.

Write the following decimals as fractions in simplest form.


PRACTICE
Use mixed numbers for eae greater than 1.

a,CC 2. 0.327=
Lhe
Gse ahaee
Fg
gh,
7
AL / 77 100 505.0505
=!
50)Vea i
5. 27.014= VL 6.

PRACTICE| Writethe followingfractionsand mixednumbers as decimals.

a=aor Ales
8. 53_(1,54af J
sun06 GHOR
10.
34,097]
i aa

Beast Academy Practice 5C


EXAMPLE
| Round
321.456
to

BA A 3
Since
921 456
321.456isSoselto:

PRACTICE
| Round
each
yee below
totheindicated
place
value
yi
| ee F|
13. 1.43 rounds to 14. 0.368 rounds to

~(nearest
tenth)a (nearest
tenth)
19.
77.7777
rounds
to(nearest
ig ee 6.245rounds
hundredth) 10
emaa
Daw,
e antl

17.
0.00317
rounds
todee
! 0? chao
18.8.2497
rounds
toGAY
(nearest
thousandth) (nearest
thousan
PRACTICE Solve each rounding problem below.

19. Insert a decimal point between two digits of each number below so that each resulting
number has hundredths digit 6when rounded to the nearest hundredth.

KO659 9065 4565464


ee gmt
When Grogg rounds the decimal below to the nearest hundredth, then rounds the result
to the nearest tenth, he gets 0.4. Fill in the blanks to make the smallest possible value of

Grogg’s
decimal. 0.3lh.
2.v
Beast Academy Practice 5C
‘theplacevaluesarelinedup,fillinganye
after adecimalpointwith0’s.Then, we comp
_ differencethesame waywedowithwholen
(0984

PRACTICE| Compute each sum or differencebelow.

21. 2.35+6.91
=ils 22.janes
23. 5.9-2.657
=29 Vv 24.10.1-0.008=
Wie
PRACTICE
| Answer
each
question
below.
25. What is the perimeter in centimeters of a triangle with side lengths 25. VA
4.3 cm, 6.45 cm, and 9.9 cm?
=Me
¥p s

26. Alice ran 6.35 miles on a trail, then turned around and ran back to 26. Vi
her starting point. Ben ran 4.4 miles of the trail, then ran back to his
starting point. How many more miles did Alice run than Ben? o7

27. What is the greatest possible difference that can be made by 27.
subtracting one of the numbers below from another?

Oi2Sn "5is44 70065 0.32 0:078 13.1. ~12.85

Beast Academy Practice 5C


EXAMPLE Compute 0.32x10.

buh
Ag
ai2/7W
O1 0.32is3tenthsand2 hundredths, 3 :
or70+100° 0.32x10=(—+
To multiply this sum by 10, we distribute the
10 as shown to the right.

So, 0.3210 =3.2. We use *


for multiplication
in this chapter to
avoid confusing the
multiplication dot with
a decimal point.

For any two adjacent place values, the place : :


value on the left is ten times the place value to its -
right. So, multiplyinga number by 10 moves each
digit to the next-larger place value to its left.

This is the same as moving the decimal point


one place to the right. So, 0.32x 10 = 3.2.

Multiplying
anynumberby10shifts
its
~decimal
point
oneplace
totheright.

PRACTICE
| Compute
each
product
below.
28.0.8x10=
6 iL 29.
0.064x 44
10= ee

31.
0.901x10x10=
Wl ~~
30: AOxS7s

DAYD+ Wy i)
32. 10x21.032x10=%! vk
10x3.001x10=__/V/\

Bg >weve
h/ 1,000 x0.03405 =
Yitp
Os
36. re J 37. 0.040608 x 10° =
;FoR
Guide Pages: 72-77 Beast Academy Practice 5C
PRACTICE | In each problem below, fill in the blank to make a true statement.

38. 1.821 x '=18 21

40.
100x0.085=6541.1Dx30-03
42. 100x } ATA 43. 10,000x,02445
=345

PRACTICE
| Answer
each
question
below.
44. If0.02689x10”=268.9,then what is n? 44. n= ia

45. Compute
0.043+
10. / 45./, 0) es

What is the smallest positive integer that can be multiplied by 0.002 46.
to get an integer result? /

What is the smallest positive integer that can be multiplied by 0.0025 47.
to get an integer result?

Beast Academy Practice 5C Guide Pages: 72-77


EXAMPLE
pee an :
Ko
burhidigyrry
10 0.32
iiS5ae and0.1 iSao
Weuseae fractions
asshown
toSas 0.32x0.
totheright.
L

Foranytwoeen place values,


theaoe vale.:
ontheright
isonetenth
theplace
valuetoitsleft.
So,multiplying
anumberby0.1moves eachact
tothenext-smaller
place
Matas
toitssib :
Thisisthe<sameasmoving
ihedecmalpoint
one
oD tole left.So,0,32x0.1=0.032.

48.

50.

52.
TD)
12,04x<0R= \
chads| uy 53. 6.5x0.1x0.1
Lt)‘ )

34.

56.

— Guide Pages: 72-77 Beast Academy Practice 5C


PRACTICE | Compute each of the following powers of 0.1.

58. (0.1)2=
Od) 59.(0.1)°=
9.00! 60.
PRACTICE
| f .. each
expression
below.
0923
61.

2.3x(0.1)?
= LLY
62. |
63.
rn uel
(0.1)° x404 = — wee 64.
6.7x0.001
=
65. == 5 yoyo66.8,500
(0.1)?
x6.05 x0.0001
0=.29
67. (0.1)®
x4,000,000
=is ee OR
68. 0.0001D,
v
In each problem below, fill in the blank with a decimal
_ PRACTICE
number to make a true statement.

69.
5t.2 ON one 70 Ai | x312.5=0.3125

71.
i.OAL
34,50
LON
= = 0.03456 72. TREheres=0.25
Beast Academy Practice 5C Guide Pages: 72-77
cEXAMPLE
|“Compute
0.9%0.04.
a. riteBach
number
asafraction,
multiply,
‘con
vert
ipa todecimal
form:
o
~7,000
- =0.036.

a: S aor
PRACTICE
Evaluate
— —=0. below.
036. wh

73.
0.2x0.3=
WW 74.0.4x0.8=

75. "+
POOGxO11e
()
7 76.0.05%0.3
=—___
pes;(0 ysCi 0),00qi¢
V7.

U006
X02
>)ie 0.07
x0.8x0.002
=
79.
Whatpositive
number’s
squareis0.0049?
00"
80. A rectangle has side lengths 0.A and 0.B inches, where A and B are -
each digits. If the perimeter. of the rectangle is 1 inch, whatis the
largest possible area of the rectangle in square inches?

(iio
Guide Pages: 78-82 Beast Academy Practice 5C
EXAMPLE
| |Compute
0.90.04. aaa | Decim
Instead of using one of the methods on hes previous page, we consider how
the decimal point moves when we multiply these numbers. —

Since0.9 =9x0. 1,multiplying


by0.9 isthesameas multiplying
by9,then
shifting.
Se decimalpointintheproduct1 placetotheleft.
Similarly,
since0.04=40.01, multiplying
by0.04isthesameas nidipiing
__ by 4, then shifting the decimal point in the product 2 places to the left.

So, to compute 0.9x0.04 we can multiply9x4, then shiftthe decimalpoint


1+2=3 places to the left.
In other words, when multiplying any two numbers, we can just count the
total number of digits to the right of the decimal point in both numbers to
figure out where to place the decimal point in their product!

0.9and0.04havea totalof 1+2=3digitstothe 0.9 x 0.04


rightofthedecimalpoint. t 5
So, we move the decimal point in 9x4 =36 so that 0.036
there are 3 digits to the right of the decimal point.

So,0.9x0.04=0.036.

PRACTICE | Place a decimal pointin each product below to make each equation true.

81. axitetat ; 82.101.001


=101101
Pal
83. 33.34
X333.4
=11115556
84.1,428.57x0.
ikrange
PRACTICE
|Evaluate
each
expression
np a/
85. oy
aeOe)gly 6 6.0.012x9=
"Gor
(03
87.
eneYGAIL.
4 000%
i

1
Beast Academy Practice 5C Guide Pages: 78-82
pea: ngDecimals
Trailing zeros are the zeros at the end of a number that have no nonzero digits
after them. Since trailing zeros after the decimal point do not enanae the value
of a decimal, we usually don’t write them. 2
Forple, weusually
write0.2instead
of0.20,0.2000,0.200000,
and soon.
s

EXAMPLE| Compute0.25x0.04.
Be extra
careful when
We begin by multiplying25x4 = 100. Then, we determine where the product of
to place the decimal point. : two numbers
has trailing
0.25 and 0.04 havea total of 2t2= 4 digits rightof 0925x0.04 | zeros!
the decimal point. 2 2.
So, we move the decimal pointin100 sothat 0
there are 4 digits to the right of the decimal pom,
including the trailing zeros.
Afterwe have placed the decimalpoint, wecan
remove the trailingzeros.
So,0.25x0.
04=0.01.

PRACTICE
| Compute
each
product
below.
89.0.2x0.5=
Q, J 90. 0.06 x0.25 =

Be
0, /
91.0.075x0.8=- 92. 0.00125x0.032 =

93. Not including trailing zeros, how many digits are to the right of the 93.
% ~~ decimal point in the product below?

0.9x0.8x0.7x0.6x0.5x0.4x0.3x0.2x0.1

94. Not including trailing zeros, how many digits are to the right of the 94.
% = decimal point in the product (0.3)'®x(0.07)'5?

95. Not including trailing zeros, how many digits are to the right of the 95.
*% — decimal point in the product (0.6)'®x (0.05)?

Guide Pages: 78-82 Beast Academy Practice 5C


)
PRACTICE For thesieproblems
ra below, fill in each blank with aiedigit so that
the equation is true and no numbers have trailing zeros.

96
(ol(7]+(o)6)-DLsE o7.
[1/0]
(@lx(o][3]=

se, [ol(A)x(o][2]-[0]
99.elleaieIP

101.[olfo|?|x[o][olZ|=[o]{o]o]3]
-100.
Ce OY
VY

102.
[0][7[5]x[0][0|
7=[0][0|3]103.
[0|/3[a}x
lolol=[0|{o]o
/

44. foe 7K]x(o][9|=[0]


7] 1s. [EWI elo
Ve ae

Beast Academy Practice 5C Guide Pages: 78-82


ues ngoe ele
EXAMPLE
Compute
0.0004
x5,000.
Since0.0004=4x0.0001, multiplying by0.0004iis the same as running.
by4, then shiftingthe decimalpointinthe product4 Baer to the left.
Similarly,since 5,000 =5x 1,000,multiplyingby 5,000 isthe same as muteiine:
_ by 5,then shifting the decimal point in the product 3 places to the right. —
So, to compute 0.0004 5,000, we can multiply4x5 = 20, then shift the decimal
point 4 places to the left and 3 places to the right. This is the same as shifting _
the decimal point 1 place to the left: :
The number of
trailing zeros in
a0. an integer tells us
So,0.0004%5,000
=2.0=2. how far to move the
decimal point to
the right.
Since 5,000 has 3 trailing
zeros, multiplying by 5,000
moves the decimal point 3
places fo the right.
PRACTICE
| Evaluate
each
expression
below.
Se,
106. 200x0.009
= Lif y 107. 0.04x13,000
=

108. 0.00025x800=
sed. 109. 300x0.03x0.003 =
idl(})‘
Th, rice
110. 40x0.012x1,000
= 8 111. 0.025x0.04x2,000
=

alegre sesh

112. Not including trailing zeros, how many digits are to the right of the 112.
% —decimal point in the product (300)'°x (0.002)1°?

113. Compute the product (400)'°x(0.0025)'°. 113.

Beast Academy Practice 5C


PRACTICE
| Answereach questionbelow.

114. A toy brick is 0.64 centimeters tall. What is the height in centimeters
of a stack of 40 of these toy bricks?

115. What is the volume in cubic centimeters of a cube whose edges are
0.4 centimeters long?

116. One inch is equal to exactly 2.54 centimeters. What is.the height in
centimeters of a gentlebug who is 0.3 inches tall?

117. Timmy earns $12.25 per hour as a lifeguard. How much money will
Timmy earn after 6 hours of lifeguarding?

118. The product of 0.125 and a is 0.01. What is a?

119. What is the total area in square meters of the two rectangles below?

4.473 m

2.1m S027 m1

2.1m

Beast Academy Practice 5C is


The goal in a Pyramid Descent puzzle is to find a path of touching blocks so that

2UaPDSeg
pivoihg
the product of the numbers in the path equals the number shown above thepyramid.
Each path moves from the top to the bottom of the pyramid, touching :0.07
one block per row.

EXAMPLE | Complete the Pyramid Descent puzzle to the right.

Ignoring the decimal points and trailing zeros in each block, the top
block contains a 2, each block in the middle row contains a 5, and
each block in the bottom row contains a 7. So, ignoring decimal
points and trailing zeros, every path has a product of 2x5x7 = 70.

The product we seek is 0.07. To go from 70 to 0.07, we move the decimal point three
places to the left. The direction in which we move the decimal point and the number of
times we move it is determined by the number of trailing zeros and the number of digits
after the decimal point in each path.
0.07
For example, in the path shown to the right, there are a total of 2
digits after the decimal point, which move the decimal point in the
product 2 places to the left. There is also 1 trailing zero, which moves
the decimal point in the product 1 place to the right. All together, the
decimal point moves 1 place to the left, so this path has a product of 7.

We consider the placement of the decimal point for each 0.2x50x0.7=7,0=7


remaining path. The only path whose product is 0.07 is shown
with the circled numbers below.
0.07 0.07 0.07

0.2x50x0.007=0.070=0.07 0.2x0.5x0.007 = 0.00070= 0.0007 0.2x0.5x7 =0.70=0.7


WY

PRACTICE Complete each Pyramid Descent puzzle below.

120. 0.06 121. 0.008 122.

Beast Academy Practice 5C


124. 0.1001

125. 0.01 126. 0.024

eoefoa
te[=J
|
| 127. 0.012 128.
0.6
|
|

0.1

2.5 | 2.4 | 085 | 0.24

Beast Academy Practice 5C


PRACTICE | Use estimation to answer each question below.
(
129. Is the product 0.0198x407.6 closer to 8 or to 80? 129. A,

130. Is the product 523.9x0.0031 closer to 1.5 or to 15? 130. )

131. Circle the number below that is equal to 0.068 x 297.4 without computing the
exact product. ~

2.2232
202202)
30.2232
180.2232
202.232
132. Circle the product below that is equal to 0.9594 without computing any of the products.

2.46x0.039 :Le
0.8211.7 0.018x5.33
0.234%4.1
)
133. Draw a line to connect each product below to its correct location on the number line.

50.03 x0.0819 0.68x0.392 0.01207x42.9 0.054 x 102.29

Beast Academy Practice 5C


PRACTICEFilleachcirclebelowwithSl to
indicatewhichexpressionisgreater.

134.
0.9914x0.52
©)0.52
af: 135.
3.911x0.049
K)022
VV
136.
9.008
x0.7134
GC)
6.3‘v. 137.
0.852
CO)
98.05
x0.00852
Vv
138.
0.029
x0.38
&)2.05x0.0061139.
3.208x0.768
GC)4.95x0.38
JS :
Y
PRACTICE| Use estimationto answer each questionbelow.

3
140. The product0.0625x48 is equal to what integer? 140. ©

141. Howmany multiplesof 10 are between 4.104x5.724 and 99? 141. / va

142. Circle the two products below that are equal. »

0.028x7.03
an
—0.982
“—
x0.410.621.571
6.76x0.259
143. Ralph uses a calculatorto compute0.0058x 21.3
and gets an answer of 1.2354.
Is his OM ee Explain.
Hoh
AN]
a firlea
a/b): UpAL dee
BeastAcademy
Practice5C
close|.Poy We
;

a
3

150.
sae0.0L
aalse. 151. Soap
a JU|/ i 152.

153. When the fraction sete is written as a decimal, what is the sum
150.|
of all the digits to the right of the decimal point?

Guide Pages: 83-91 Beast Academy Practice 5C


EXAMPLE
| Writethefraction
Zeasadecimal Fractions to De
— Wecoulduseiheveihedontheprevious
page.Or,we __ Reviewlong
can use longdivisionto convertfractionsintodecimals. division in the
Divisionchapter of
: Sma Beast Academy 48.
= 720 using long division is easier to
Check out an
think about thwe write7 as 7.0.Then,we can ignorethe
alternative long
decimal pointfor amomentand thinkof20)7.0 as 20)70. division algorithm
on page 105. —
_ Since20x3=60,weknowthat20goesinto70
3 times with 10 left over. Dividing 20 into 7.0 is similar.
_ Since 20x0.3 =6.0,we know that 20 goes into7.0
0.3 times with 1.0 left over. -

Next,we divide20 into 1.0. This is easier to thinkabout if


we write 1.0 as 1.00 and consider dividing20 into 100.

Since20x5=100, we know20 goes into100 sy


5 timeswith0 leftover.Dividing
20 into1.00is similar.
Since200.05 =1.00,we know20 ae into100 :
0.05timeswith0 ae over.
So, 5=7+
+20=0.3+0.05
=0.35.
Check:20x0.35:
=7.0=/. yo

PRACTICE Use long division to write each fraction below as a decimal.

ae ‘=5 g
154. Fraction: 5) 40 155. Fraction:
—215 4 ) 3.0
Decimal:: i Decimal:
“7

' 1 v§5
156. Fraction:
70. 7 ) 157. Fraction: ant
Decimal:
_*"%/ Decimal:
''(. 7 ~

||
|
|
Print more practice problems at www.BeastAcademy.com
|

| Beast Academy Practice 5C Guide Pages: 83-91


|
Fractions fo Decimals
If a fraction can be written with a denominator that is a power of 10, then it can be written 2as.
a terminating decimal. This means that its decimal digits do not go on and on forever —
(not including Bis zeros).
For example, => can be written as 24372. —0.984375. However, not all decimals terminate.
:= 1,000,000
EXAMPLE| Write the fraction “ as a decimal.

We cannot write + as an ae fraction with a denominator that is a power of 10.

However,
wecanconvert
=7toadecimal
using
long
division.
. 0.0001
11 goes into 9.0 0.008 0.008 —
_ 0.8 times with 0.2 left over. — 8 0.01 0.01
0.8 0.8 0.8 0.8
Then, 11 goes into 0.20 11)9.0 11 19.0 11)9.0 11 9.0
0.01 times with 0.09 left over. -8.8 -8.8 -8.8 -8.8
_ O02 _ 020 0.20 | 020
Then, 11 goes into 0.090 O11 O14 =O 11
0.008 times with 0.002 left over. 0.09 0.090 0.090
: ~ 0.088 - 0.088
Then, 11 goes into 0.0020 7 : 0.002 0.0020
0.0001 times with 0.0009 left over. | | =o

We notice a pattern! The remainders alternate between decimals ending in 2 and 9. When we
eee these remainders by 11, the pceets alternate between decimals ending in 8 and 1.
So,+=0.818181.
Decimals whose digits repeat in a pattern are called repeating decimals. We usually
write repeating decimals by drawing a bar over the digits that repeat. For example, we
write 0.444... as 0.4, we write 0.1555... as 0.15, and we write 0.0454545... as 0.045.
So,2=0.
O41 818181.
.=0.81.

PRACTICE | Use long division to write each fraction below as a repeating decimal.

eters
158. Fraction:

Decimal:
el:/ T°
— )
Cs
3 1.0 159. Fraction:
&
Decimal:
6 ) eit)

Guide Pages: 83-91 Beast Academy Practice 5C


PRACTICE | Write each fraction below as a decimal using long division.

——
160.+=(ia 1612
=: 0.1 162.
9 2
7\
eee

PRACTICE
| Answereach questionbelow.

163. When = is written as a decimal, what is the 100" digit to the right of
* the decimal point?

164. When a is written as a decimal, what is the 99" digit to the right of
*& the decimal point? . .

165. Only fractions that can be written with a denominator that is a power of 10 can be
written as terminating decimals. Circle each fraction below that can be written as a
terminating decimal.

166. If the fraction 5 is in simplest form and can be written as a terminating decimal, then what
must be true about the prime factorization of b?

ie (/aD Jp|if :aa


\A

Beast Academy Practice 5C Guide Pages: 83-91


PRACTICE| Answereach question below.

167. Use the fact that = 0.125 to write x as a decimal.

1 = ae? pis (ke

168. Multiply 3 =0.3by 2 to write = as a decimal. 168. 3 =~

169. Divide
t= 0.3by3towrite
4asadecimal. 169.<anee

170. Use your answer to the previous problem to write each multiple of A below as a decimal.

171. Use the strategies above to help you fill in the blanks in the statements below.

> ialcount! Therefore,


7 VY
1ABrOO
Tait 2 Wo
Beast Academy Practice 5C
5AES
: eoa Conversion
Strategies
a Sele "Use
thefact
that
t3= 0.
3towrite
xasadecimal.
Wenotice
that
ag4xaaMultiplying
by isthesame
asdividing
_ by 11, SOwe divide0.3 by 11,
Thinking
of0.3 as 0.33,withrepeating
blocksof33,makesdividing
by11easier.Weknowthat0.33+11=0. 03,0.3333=11ae ae
and0.333333+11=0.030303.— -
From
this
pattern,
weseethat
(0.333333...)
‘14
ees
= 0.08.
.

PRACTICE| Answereach question below.

172. Divide=
az =0.08 by3 to write o>as a decimal. 172. = OV)

173. Use your answer to the previous problem to write each multiple of =9
3 below as a decimal.

esOU!
1 1.00) 17ae 6. 779 ee 11

174. Write 0.36 as a fraction in simplest form. 174. 0.36 =


HN)

Usethe strategiesaboveto helpcialfillinthe cae inthe statementsbelow.

1a
Be mE Jp
80 ag 4
0002and
<1
999
MLN]
G.Therefore,
>=ae
37 999
02)
Beast Academy Practice 5C
PRACTICE
| Answer
each
question
below.

176. Write ee
— as a decimal. 176. OO = aa 4
/ h
as | ;

177. Convert 7 to 99thsto write 7 asa


decimal. 177.
(Rise 7
why 4
} 7

178.Convert
+5to99ths
towrite—asadecimal.
~ 178.== | 7

ime
179. Usethefactthatfe=0.7towrite
& asadecimal. 179.a= m q

180. Circle the number below that is equal to 4x0.07. q

0.28 0.28 0.29 0.29 0.31

\
1We Convert
+3
=to90ths
towrite+2asadecimal.
> 181.3 = + 4

BeastAcademy
Practice
5C a
EXAMPLE
| Use
thefact
that
Fifi write
Jasadecimal.
Since4=0.5and4=0.3,wetrytosubtract
0.5-
03.
Welineupthedecimal
pointsandwrite0’safterthe5in0.5
To subtract, we need to take a tenth fromthe tenths place in 0.5
to make 10 hundredths.Then, we take a hundredthto make 10 |
thousandths, and so on.Then, we can subtract as shown below. —
Oe ain ee dass
0.5000... O3000.. ¢.BOD... 0.3009...
~USss0. —U3ssos.. 0.B.3998..: —0.3333...
oe 5“on6.- :

PRACTICE
| Answer
eachquestion
below.

182. Usethefactthatact =2towrite


2 asadecimal. 182.
Moe
as peed renWh 7 (/b %:
183. Usethefactthatcee Os
towrite
5 asadecimal. 183.7

oa
184. Usethefactthateas “t towrite
—=
3+asadecimal.
11OF
184.—=

2h ti Dd ff Be
_ 185. Use the fact that 7-7 = 75 to write +5as a decimal. 185. 19 a

| ZNfan
fey(/ ff}€}
ae
| 186. Use the fact that 4-75 =75 to write 75 as a decimal. 186. =
|

Beast Academy Practice 5C


In a Frac-Turn puzzle, each fraction begins a path on a grid. The path is directed byarrows)

[-O
SUu4I7
04
until it leaves the grid or creates a loop. The goal is to fillsome or all of the squares in the
grid with digits so that the path from each fraction traces its decimal form, with no leading
zeros to the left of the decimal point. Below is a solved example, with each path traced.

4 i 4
3 i 3
sl 3 1 : So
Alt | + 2 ee
1 Ali|- 2

EXAMPLE| Solve the Frac-Turn puzzle to the right.

We begin by converting each fraction to a decimal.

2
== .02 tf =o9
21 7-4!
ae 18

The path for 2 = 3.6 has only one empty square after the decimal
point. So, we must place a 6 in the bottom-left square, and a 3 in
one of the other empty squares in the bottom row. The 3 cannot go
in the bottom-right square because this square is in the path
of i= .21. So, we fill in the 3 as shown to the right.

Next, the paths of * = (2 and 4; = .21 both contain a loop.

No square within either loop can contain a digit other than 1 or 2.


So, we must place the 0 in a = .02 as shown to the right. Then,
we place the 2 in the square just above the 0.

Finally, we place a 1 in the only square in the loop that correctly


completes the paths of => = 12 and ae .21. Each fraction’s path
traces its decimal form, so we are done.
writea leading

191. eS 192. 23

Beast Academy Practice 5C


PRACTICE

193. ST Tea al 194.


Pyledel| wlN
r]o
iePTSeole):
Bit.
19 55

195.
* 196.

hoy

197. 198.
y
if

Beast Academy Practice 5C


PRACTICE | Solve each Frac-Turn puzzle below.

199. 200.

rammVaKa
11
aliel
re
KScall
735
S
Ue
201. Complete each of the following to solve the Frac-Turn puzzle below:
ns e Write each fraction on the left in its correct location on the outside of the grid.
e Place all ten digits (from 0-9) once each on the grid so that each fraction’s path
correctly traces its decimal form.

Fractions

29 a
40 6

21 2
4 11

g ail f
4 11

2 aay
10 15

24
5

Beast Academy Practice 5C


Many
decimal
quotients
canb>converted
e into
integer
easier
tocompute. :
EXAMPLE
| Compute4.8+0.16.

We can use long division to divide 0.16 into 4.8.

Or, we can convert this into a quotient of integers. This is easiest to see
when we write the quotient as a fraction:

a - Multiplying
thenumerator
anddenominator
otAB
2by100
gives
an
equivalent
fraction
whose
numerator
anddenominator
areboth
ae
—*
48° 7 480
4
0.16.
> a
- Then,
wecompute
48016>x

PRACTICE Express
eachquotient
belowasa fraction
ormixed
numberinsimplest
form.
ae \ /t}
202.0.05+1.25= UGE: 203.0.4+2.2=

50 Tigi
204.3.03:0.6=_
U7!/ 205.3.5+70.707
= g

PRACTICE| Express each quotientbelowas a whole number or decimal.

p06,
222 7
oO
05
207...
2421040)/ nye)
2032CEG

/ / - / ae

ie ~Tae 210.0.002+0.04=
209.4.9:0.142
7" | 25 he,2
211.0.6+4.5=

104 Guide Pages: 93-94 Beast Academy Practice 5C


Extra! The Traditional Algorithm for Long Division

A
T
_ Many people use an algorithm (a set of steps) for long division that is slightly different
from the one we have used in Beast Academy.

EXAMPLE
| Writethe fraction3 as a decimal.
Traditional Algorithm:
In the traditional algorithm, we find the digit in each place value of the quotient, working
from the largest place value to the smallest until the decimal repeats or terminates.

2 Step 1:
ae We locate the first nonzero digit in our quotient. Since 37 is greater than 15,
” ®te the quotient is less than 1. So, the ones digit of our quotient is 0. We write a
&oL=£.. Qin the ones place of our quotient, aligning the decimal point and all of the
0. place values of our quotient with the place values of the dividend, 15.
37) 15.0 Then, we write 15 as 15.0, which is 150 tenths.

0.4 Step 2:
37) 15.0 To find the tenths digit of our quotient, we divide 150 tenths by 37.
; 37 goes into 150 a total of 4 times, so there are 4 tenths in the quotient. We
- 2 write a 4.in the tenths place of our quotient. 4x37 = 148, so our remainder
is 150-148 =2 tenths (0.2). If we keep our place values and decimal points
aligned, we can think of all the work in terms of whole numbers.

0.40 Step 3:
37) 15.0 Next, we find the hundredths digit of the quotient. We write a 0 in the
148 hundredths place of 0.2 to get 20 hundredths (0.20). Since 37 goes into 20
0.20 0 times, we write a O in the hundredths place of our quotient.

0.405 Step 4:
37) 15.0 Next, we find the thousandths digit of the quotient. We write a 0 in the
148 thousandths place of 0.20 to get 200 thousandths (0.200). Since 37 goes into 200
0.200 5 times, we write a 5 in the thousandths place of our quotient. 37 x5 = 185, so our
ae remainder is 200-185 = 15 thousandths (0.015).

0.405 Steps: o
Next, we find the ten-thousandths digit of the quotient. We write a 0 in the
a ee ten-thousandths place of 0.015 to get 150 ten-thousandths (0.0150). But wait,
“0.200 we’ve done this before! In Step 2, we divided 150 by 37. Now, we are dividing
0.185 150 by 37 again. We get the same quotient (4) and remainder (2) as we did
0.0150 __ before, and our digits begin to repeat.

So,2 =0.405405406...,
or 0.405.
There are many variations on the long division algorithm. We encourage you to find one that works
best for you. However, for many problems that require long division, it’s fine to use a calculator.

Beast Academy Practice 5C


PRACTICE
| Solve
each
problem
below.

212. Notincluding trailing zeros, what is the rightmost digit 212.


4\99. : ‘
when (3) is written as a decimal? \eae

213. When (2)" is written as a decimal, how many zeros are to the right of 213.
the decimal point before the first nonzero digit?

214. Potatoes cost $0.67 per pound. What is the smallest whole number 214.
of dollars you will need to buy 6.375 pounds of potatoes?

215. What is the smallest positive number that 0.008 can be multiplied by 215.
*% to get a decimal product that has exactly 3 digits after the decimal
point, not including trailing zeros?

Beast Academy Practice 5C


PRACTICE
| Solve
each
problem
below.
216. Order 0.05, 0.050, 0.05, and 0.05 Ton least
eaet to sb on the lines below.

009 000 0% Of
eA a

217. Express the sum 0.1+0.01+0.001 as a fraction in simplest form.

218. What is the smallest positive integer you can multiply by 3a SO that 218.
the result can be expressed as a terminating decimal? a

0.03 /
219: Express 5 as a decimal. 219."

220. Grogg adds three copies of 0.3 and1gets 0.9, as shown sciow.
What simpler number is equal to 0.9? ae
Ye
0.3333...
mae
Oberh
)
aa4
DEMS?
05.
e Od 3B
0.3333...
0:9999...

Beast Academy Practice 5C 107


[108
| Hints
toSelected
Problems | Beast
Academy
Practice
5C
Below are hints to every problem marked with a ®.
Work on the problems for a while before looking at the hints.
The hint numbers match the problem numbers.

114. What expression gives the k* triangular number for any


Segue, e number k?
. Where are the endpoints of each new diagonal? 132. Do you see any triangular number patterns in the
. This one is out of this world! gumballs?
. It might help to look at this sequence in the mirror. 133. How many rows and columns are there in the k*"figure?
. What do the letters in the top row have in common? 134. How many black gumballs are in the n" figure? How
- Too good of a hint might give it away. many gray gumballs?
So a cryptic rhyme is all that we'll say. 138. If we call the second term in Grogg's sequence a, what
» It might help to write the whole numbers as fractions. expression could we write for the third term in Grogg's
sequence?
. What is the repeating pattern in this sequence?
141. How could your answers from the previous two problems
. What happens when you raise a negative number to an help you answer this one?
even power? An odd power?
142. To reach each step, Hoppy can hop to the step directly
. It might help to write the whole numbers as fractions. below it and make a 1-step hop, or he can hop to the
. Howcan you use the sum of the terms to find the middle step that is two below it and make a 2-step hop.
term in an arithmetic sequence? 143. Start small. How many arrangements of 1 domino are
- What expression gives the k" term for any number k? possible? 2? 3? 4? How can you use these shorter

. What sequence(s) can you make that include 4? arrangements to count longer ones?
149. List the first nine terms. Notice a pattern?
. The sequence that includes 21 also includes 45.
152. List the first nine terms. Notice a pattern?
. What sequence(s) can you make that include 60?
153. You don't have to pick the first terms first.
. The sequence that includes 17 also includes 65.
154. What is true about the digit sum of any multiple of 9?
. What are the possible common differences?
158. The terms spiral around to form squares. When the first
. What are the possible common differences?
100 terms are written, where will the 100 be?
. What are the possible common differences?
160. What number is displayed after one press? Two
. What is the greatest possible common difference? The
presses? Continue this sequence and look for a pattern.
smallest?
161. Consider the first few pours and look for a pattern. Will
. What do the third and fourth terms tell you about the
this pattern continue?
common difference?
164. How many of the first 50 positive integers are squares?
101. What numbers must fill the empty squares in the left
165. What is the smallest number that has all of its positive
column?
multiples included in this sequence?
102. Start with the bottom row and the left column. What is the
166. What is the smallest integer that can appear in this
common difference of each? What number can fill the
square above the 16?
sequence?
167. How could you find y+z?
103. What numbers are missing in the top row? The bottom
row? How can we arrange these to make each column 168. How many toothpicks are added to each figure to make
an arithmetic sequence? the next one?
104. What numbers are missing in the top row? The bottom 169. What expression could you use to find the k* term of
row? How can we arrange these to make each column each sequence?
an arithmetic sequence?

113. What numbers could you add to the 997" triangular


number to get the 1,002" triangular number?

Beast Academy Practice 5C Hints to Selected Problems


181. Using the given ratio, we can label the heights of Jorble
a aSIWS QaSSeRZ SASS Sele SSS SR and Yorble 5x and 9x. What is the difference in their
14. What could we multiply both quantities in the ratio by to heights?
give a ratio of two integers? 182. If the height of the rectangle is 2x, what is its width?
15. What could we multiply both quantities in the ratio by to 183. If Alyssa bikes 5 mph and runs 2 mph, what is the ratio
give a ratio of two integers? of the time it takes for her to bike to school to the time it
26. We can split Allison's green paint mixture into 5 equal takes for her to run to school?

parts, with some blue and some yellow. If there is i ofa 184. Since the number of red cars doesn't change, how could

pint of yellow paint, how much paint is in each part? you rewrite the ratios using the same number of red cars
in both ratios?
27. How many miles did Peter travel by train?
185. What fraction of the seats are filled? Empty? Filled by
28. Which color of bead will Anna run out of first?
teachers?
33. How many stingravens are there in the exhibit?
186. How long does it take the skateboards to collide?
34. What fraction of the perimeter comes from the sides that
are the rectangle's width?

42. How could you write the cornstarch-to-water ratio as a


ratio of integers? 20. What is the smallest decimal that rounds to 0.4 when
rounded to the nearest tenth?
78. What fraction of 100 is a?

79. 46. What is the smallest integer result you could get?
What fraction of the total number of goals scored by Ben
and Alfie is 16? 47. What is the smallest integer result you could get?
86. a. lf Griffinburg and North Pegasus are actually m miles 57. How are multiplication and division related?
apart, what equation can we write? 93. How many digits are to the right of the decimal point if
b. If Unicorpia and West Mermaid are m inches apart on
you write the trailing zeros in the product?
the map, what equation can we write?
94. How many digits are to the right of the decimal point in
90. What fraction of all the trees are apple trees? Fuji apple (0.3)'5? In (0.07)'5?
trees? Gala apple trees? Peach trees?
95. How many zeros are at the end of the product 68x 5'5?
94. What fraction of all the animals are green frogs? Yellow
104. How many trailing zeros are in the product 0._[ 1 ]xo.LJ
frogs? Toads?
before we remove them to get 0.7?
95. What fraction are a, b, and c of 70?
105. How many trailing zeros are in the product 1.]x0.L] 1]
96. Using the given ratio, we can label the length, width, and before we remove them to get 0.04?
height of the rectangle 2x, 5x, and 8x.
112. What is 300 x0.002?
109. What numbers fill the triangles that touch the 3:5:8
113. What is 400 x0.0025?
square?
118. Try writing these decimals as fractions.
110. What numbers fill the triangles that touch the 2:3:5
square? 119. What could you do with the rectangles to make this
131. How many minutes does it take Urlick to paddle 1 mile? computation easier?
153. How could we write this fraction with a denominator that
135. b. Use your answer to part a. How many houses can 9
is a power of 10?
paint-bots paint in 3 hours?
163. What is the repeating pattern of digits?
136. b. Use your answer to part a. How many days will it take
for 1 carpenter to build 8 sheds? How many days will it 164. What is the repeating pattern of digits?
take 3 carpenters to build 8 sheds? 171. Direliye tee eel Desay
We know that 7 10%o = =2X95) and 15=6X55.
137. a. How long will it take 24 lumberjacks to chop 5 logs? 175. Multiplying by aa is the same as dividing by 111.
b. What fraction is 6 logs of 9 logs? What fraction is 72
Then, we have 55 = 5 = a3.
minutes of 40 minutes?
142. At 2 meters per second, how many meters can you jog in 180. How could your answer from the previous problem help
a minute? here?
175. How many grams of sugar are in the whole box? How 181. We know that + =5x$. Since we know how to write
many grams of sugar are in one ounce? any number of ninths as a decimal, we can write any
176. What conversion factors would you use to convert from number of 90ths as a decimal.
days to weeks, centimeters to inches, and inches to feet?
178. How many of the 135 fruits does Jeremy have?

Hints to Selected Problems Beast Academy Practice 5C


195. How many of the three empty squares in the second
row contain a digit? What digit(s)?
196. How could you fill in digits to complete the path of the “
that begins at the bottom of the grid?
199. How could you complete the path for 2 without blocking
the path for x? Then, how could you complete the path
for xn?

200. What digits must be placed in the loop that is shared


bythepathsofa. a. andn? Howcanthesedigitsbe
placed?
201. For a puzzle like this, knowing where numbers cannot
go is really important. Begin by X'ing any location on the
outside of the grid whose path does not include exactly
one decimal point. As you work through this enormous
Frac-Turn, be sure to place X's in squares that cannot
contain digits.
215. What is the smallest possible product that has exactly
three digits after the decimal point (not including trailing
zeros)?
218. What is always true of the denominator of a fraction that
can be expressed as a terminating decimal? (It may help
to review your answer to Problem 166.)
219. How could you express 0.03 and 0.2 as fractions?
220. What is 0.3333... =0.3 as a fraction?

Beast Academy Practice 5C Hints


toSelected
Problems
[111
|
BeastAcademy
Practice5C
| 112| SequencesChapter7 Solutions
number of segments in the path increases by 1 from
each figure to the next.
1. The pattern begins with an equilateral triangle. Then, 1 line 2lines 3lines 4lines
each figure in the pattern is a regular polygon with one
more side than the figure that came before it.
The last figure we see in the pattern is a regular
IVAN
IAAA
In the next figure, a path of 5 segments will connect the
hexagon. So, the next figure is the regular heptagon. lower-left vertex with the upper-right vertex.

2.
O37.@Cae
The dot’s position alternates between the top and bottom
7 AN
ANCA)
of the square. The small square moves clockwise from corner to corner
within the larger square.

bl ead
ial Ee
So, in the next figure the dot will be at the top of the ir i el
square. So, in the next figure the small square will be in the
The black triangle moves clockwise from corner to corner. upper-right corner of the larger square.
The segment within the smaller square rotates 45

adsl eidkaa degrees counterclockwise from one figure to the next.

So, in the next figure the black triangle will be in the


upper-left corner.
al ga SSac
So, in the next figure the segment will be rotated so that
We circle the only answer choice with the correct
it connects the lower-left and upper-right vertices of the
placement of the dot and the black triangle. smaller square: [/]

NAQOOnM We circle
placement
the only answer
of the smaller

GBF
choice
square
with the correct
and the line within it.

3. The circle’s position moves counterclockwise from vertex


to vertex in the triangle.

AOU AS
The black triangle moves counterclockwise from point to
point within the star.

So, in the next figure the circle will be at the top vertex.
The tick marks move clockwise around the sides of the
WIE
triangle, with each figure having one more tick mark than So, in the next figure the black triangle will be in the
the one before it. rightmost point of the star.
The dot alternates between the right and left sides of the

ae ING black triangle.

So, in the next figure there will be five tick marks on the
right side of the triangle.
We circle
placement
the only answer
of the circle and
choice with the correct
tick marks.
WIT IE
So, in the next figure the dot will be on the right side of
the black triangle.

AALAGA We circle the only answer choice with the correct


placement of the black triangle and dot.

4. In each figure, a path is drawn from the lower-left vertex


of the square to one of the vertices on the right side. The
xen exes
Beast Academy Practice 5C Sequences
Chapter
7Solutions
[113]
7. Each figure is a copy of the figure before it, with one 13. The letters shown are in alphabetical order, with some
extra diagonal drawn. However, it’s difficult to see the letters missing.
pattern the diagonals make as they are being added. Moving from A to B, we do not skip any letters.
Moving from B to D, we skip the letter C.
To make the pattern easier to visualize, we
label the ends of the diagonal in the first nes Moving from D to G, we skip E-F.
Moving from G to K, we skip H-I-J.
figure A and B, as shown to the right.
Moving from K to P, we skip L-M-N-O.
In the second figure, we have the diagonal from the first
Each time we skip a group of letters, we skip one more
figure, plus a new diagonal. We can think of this new
letter than the previous skip. After P, we skip
diagonal as a rotation of the original diagonal, as shown
below. A
Q-R-S-T-U. So, the next letter is V.
-> 14. The letters in this list are the first letters of the planets in
B { B
\ \
A IN \ our solar system! Starting with the planet closest to the
| dae”
B sun and moving outward, we have
The A and B ends each rotate two vertices clockwise. If
Mercury, Venus, Earth, Mars, Jupiter, Saturn, Uranus.
we continue this pattern, then we get the first four figures
The next planet is Neptune, so the next letter in the
that are given.
pattern is N.
B ESN
F> 85 Aees\ ->»B
¢G ; CA
A A) BR 4 15.
ML-= Reading from right to left, these letters spell
(dc?
B
r\
2-78 AN
! EASTACADEMY
These are the letters in “Beast Academy,” but with the B
So, to get the next figure we rotate the A and B ends of
missing! So, the next letter is B.
the newest diagonal two vertices clockwise:
16. The letters are arranged into an “upper row” and a “lower

tm
a
O-s peo
aAY row.” The letters in each row appear in alphabetical
order.
Ae

We circle the correct answer choice


A as shown below.
All of the
curves, and
letters
all of the
in the upper
letters
row are written
in the lower
without
row have at
least one curve.

PAL BE The next letter


it is the first capital
in the pattern
letter after
After J, the letters K, L, M, and N are written without
belongs
J that
in the
has
lower
a curve.
row, so

8. The letters alternate between capitalized and lowercase, curves, but O is curved.
and every other letter in the ordered alphabet is skipped.
17. If we say the letters in this list out loud, they all rhyme!
So, the letter after the lowercase ‘k’will be capitalized,
“Bee, Cee, Dee, Ee, Gee, Pee, Tee, ...”
and it is two letters after ‘k’in the alphabet. This gives the
These letters are also in alphabetical order. So, the next
capital letter M.
letter in the list is the first letter after T that rhymes with
9. Each letter in this list is the first letter of a day of the
the letters in the list: V.
week, starting with Sunday and moving sequentially
through Friday. The day after Friday is Saturday, so the
next letter is S.
10. Moving from A to E, we skip the letters B-C-D. Then,
18. To get from 1 to 2, we add 1.
moving from E to I, we skip the letters F-G-H. Then,
To get from 2 to 4, we add 2.
moving from | to M, we skip the letters J-K-L.
To get from 4 to 7, we add 3.
To get from one letter in the list to the next, we skip the To get from 7 to 11, we add 4.
three letters between them (after Y, we skip Z, then loop To get from 11 to 16, we add 5.
back through the alphabet and skip A and B.)
The number we add to get from one term in the
So, to find the letter after G, we skip H-I-J to get K. sequence to the next increases by 1 each time. So, the
11. The letters in this list are in reverse alphabetical order. number after 16 is 16+6 = 22, followed by 22+7 =29,
Every second letter is also upside down. Since T comes followed by 29+8 = 37.
before U, the letter after the upside-down U is a 1,2, 4, 7, 11, 16, 22, 29, 37
right-side-up T.
19. To get from 46 to 40, we subtract 6.
12. The letters in this list are the first letters of the positive
To get from 40 to 32, we subtract 8.
integers when spelled:
To get from 32 to 22, we subtract 10.
One, Two, Three, Four, Five, Six, Seven, Eight
To get from 22 to 10, we subtract 12.
So, the next letter in the pattern is N for Nine.

Sequences Chapter 7 Solutions Beast Academy Practice 5C


The number we subtract to get from one term in the So, we try writing the integer terms as equivalent
sequence to the next increases by 2 each time. So, the fractions with denominator 2. This gives the following
number after 10 is 10-14 = -4, followed by -4- 16 =-20, sequence:
followed by -20-18 =-38.

46, 40, 32, 22, 10, -4, -20, -38


20. Ignoring signs, each number after 640 is half the number
The numerators of the terms are the perfect squares!
that came before it. Then, every other term is negative.
The fractions with even numerators are simplified in the
So, the next three terms are shown below.
original sequence.
640, -320, 160, -80, 40, -20, 10, -5
Continuing this pattern, the next three fractions are
We can also find each next term in the sequence by
dividing the previous term by -2.
2, &.and©.Simplifying
4 =32,wehave
21. The numbers in this sequence alternate between getting 1 9 25 49 81
ope 8, >?18,9?32;2°
bigger and getting smaller.
Each time a number gets bigger, it is double the number
a . , —
that came before it. For example, we double 10 to get 20
we double 14 to get 28, and we double 22 to get 44. 24. We look for a relationship between each term and its
x2 x2 x2
position in the sequence.
“a aN re The 1s term in the sequence is 1+18= 19.
10, 20, 14, 28, 22, 44, 38, __, The 2™ term in the sequence is 2+18=20.
Each time a number gets smaller, we subtract 6 from the The 3% term in the sequence is 3+18=21, and so on.
number that came before it. For example, we subtract 6
Each term is 18 greater than its position number. So, the
from 20 to get 14, we subtract 6 from 28 to get 22, and
15" term is 15+18=33, and the 50" term is 50+ 18 =68.
we subtract 6 from 44 to get 38.
-6 -6 -6
25. Each term in this sequence is a number raised to the
a a\ a third power.
10,, 20; 14, 28, 22, 44, 38 _ , ___,
The 1* term is 1°= 1, the 2 term is 2°=8,
Continuing this alternating pattern, the next three the\3 term is 3°= 27, the 4" term is 4° =64,
numbers in the sequence are 38°2= 76, then 76-6 = 70, the 5" term is 5*= 125, and so on.
then 70*2= 140.
Each term is equal to its position number raised to the
K2 =6. 52 =O ee Ol oe OL Xe third power. So, the 9" term is 9° = 729, and the 20" term
TR LALA LALA LALALATA
10, 20, 14, 28, 22, 44, 38, 76, 70, 140 is 20%= 8,000.

22. The numbers in this sequence are increasing, which 26. The terms in this sequence alternate between positive
suggests that addition or multiplication may be involved. and negative multiples of 3.

We considera pattern involvingaddition: The 1* term is 3-1 =3, the 2" term is -(3+2) =-6,
the 3% term is 3-3 =9, the 4" term is -(3°4) =-12,
+1 +4 +18 +96
SATATATLA and so on.
1, 2,-6,524, 120, “(I —_—
—) ——
Each term whose position number is odd is 3 times its
There is no obvious pattern that uses addition to get from position in the sequence. Each term whose position
one term to the next. So, we consider multiplication. number is even is the opposite of 3 times its position in
x2 xB x4 x5 the sequence.
TATA TATA
det 2tiO wed, 120, a0asey So, the 25" term in the sequence is 3°25 =75, and the
100" term in the sequence is -(3* 100)=-300.
Here, we see that the number we multiply by to get from
one term to the next increases by 1 each time. So, the 27. The terms in this sequence alternate between 0 and
next number is 120°6=720, then 720° 7 = 5040, then some multiple of 7. The terms in odd-numbered positions

5040-8 = 40320. are equal to 0: the 1* term, 3 term, 5" term, and so on.
So, the 99" term is 0.
KDB KA XS x6 x7 x8
LAX TALALAR LX TR F* Among the terms whose position number is even, we see
1, 2, 6, 24, 120, 720, 5040, 40320
the following pattern:
You may have noticed that the numbers in this sequence
The 2™ term is 1*7=7.
are the factorials: 1!, 2!, 3!, 4!, 5!, 6!, 7!, 8!.
The 4" term is 2°7=14.
23. The terms in this sequence alternate between fractions The 6" term is 3°7=21.
and integers. We notice that each of the fraction terms
Each of these terms is equal to the product of half its
has denominator 2 and a numerator that is a perfect
position number and 7. So, the 40" term in the sequence
square. However, it is difficult to see a pattern among the
is (40+2)*7=20°7=140.
terms that are integers: 2, 8, and 18.

Beast Academy Practice 5C Sequences


Chapter
7Solutions
115|
28. The terms in this sequence form a pattern that repeats 32. This is the sequence of perfect squares. Each term is
every four terms, as shown below: equal to the square of its position number.
141 ligt aA get teatee
Position: 1% Qnd 31 4th 5th
Within this pattern, every second term is 11. So, any term " a iy
with an even position number is 11. Therefore, the 80" Term:
1 4 9 16 29
term in the sequence is 11. So, the 10" term is 10? = 100, the 25" term is 25* = 625,
Since 51 is odd, the 51° term is either 1 or 111. and the n" term is n?.

The terms equal to 1 are the 1°, 5", 9", 13", and so on. 25th ae nih
Positions 1tar20%% seid 4th 5th. 10 ...
These are the terms whose position number is 1 more
Term: A054 39 16: S25 ox 100 -- 625 -- n?
than a multiple of 4.
The terms equal to 111 are the 3%,7", 11", 15", and so on.
33. The terms in this sequence alternate between -1 and 1.
These are the terms whose position number is 3 more
We know that odd powers of -1 are equal to -1, and even
than a multiple of 4.
powers of -1 are equal to 1. So, we can write each term
Since 51 =48+3 is three more than a multiple of 4, the as -1 raised to the power of its position number.
51° term is 111.

29. This is the sequence of the positive even numbers. Each


Position: 1%
5th
en yer yer 1 yew jo mat Ond 3d 4th

term is equal to twice its position number.


Term:
-1 1 -1 -1 os
Position: 1% 2 3% 4 5h So, the 15" term is (-1)'®=-1, the 40" term is (-1)*°=1,
oe de es es es and the 7" term is (-1)”.
' Term:
2 4 6 8 10 Position: 1°. 2 (361.4%) OP) Popa 4oth ... ith
So, the 25" term is 2°25 =50, the 50" term is 2°50 = 100,
and the n™term is 2n. Term: -1 410-1 40-1 es =P oe To (1)

Position=ysane oo 4th 5th. 25th ane 50th aes nh 34. We begin by writing 2 as 2 and 1 as + so that every term
Term: 2-4 6 8 10 -:. 50
in the sequence is a fraction.
2°). 822 SOnehEpe ge

30. This is the sequence of odd numbers, beginning Then, we notice that the numerators in this sequence
with 1. We notice that each term is one less than its alternate between 2 and 1. We could consider the terms
corresponding term in the sequence of even integers with even position numbers separately from the terms
from the previous problem. So, each term is one less with odd position numbers. However, doing so makes
than twice its position number. it difficult to write an expression for the n" term, whose
position number could be even or odd.
Position: 1% on 34 4th
5th
jet deat Jeet Jeet jee a4 We can write each fraction with numerator 1 as an

Term:
1 3 5 TE 9 oo equivalent
fractions
fraction
have the same
with numerator
numerator,
2. This way,
as shown
all
below.
So, the 30" term is 2°30-1 =59, the 75" term is Dis Dict DiesOhi; Oeste?
2°75-1=149, and the n" term is 2n-1.
Now, each term is a fraction with a numerator of 2 anda
Postion Ay &207. Abe Soh e030" ve 75th 2. .yyth denominator equal to that term’s position number.
TET ae rg ee PMO BG 149 -.2n-1 tee We th ta 2
So, the 20" term is 50 10" the 35" term is 35° and the n' h
term is 2
31. Each term in this sequence is equal to 2 raised to the
power of its position number.

Positionscis saan 3” 4th 35.


aa ae) 5th
retin
We add 9 to get from each term to the next.

Term:
2 4 8 16 32 +9
awe +9 +9 +9
Ve Ve \
Ty AG: 255 B45 AB; vers
So, the 6" term is 2°=64, the 10" term is 2'°= 1,024, and
the n" term is 2”. So, the common difference is 9.
36. We add 8 to get from each term to the next.
Positions 1 a2 Se ae BO ce GU es een emennyy

Tem: 2 4 8 16 32+ 64+ 1,024-. 2" +8 +8 +8 +8


a
“BS/ 20 pel Z, 6-9).slgueste

So, the common difference is 8.

Sequences Chapter 7 Solutions Beast Academy Practice 5C


37. We add -3 to get from each term to the next.

+(-3) +(-3)+(-3)+(-3)
PRCA 10, 122, 4 1 3
14%, 174, 193, 22, 2422
are 238 20, 17,3.

So, the common difference is -3. 45. To get from the 1* term to the 10" term in an arithmetic
sequence, we add the common difference 10-1 =9
38. To get from 21 to 35, we add the common difference
times. This adds a total of 115-25 =90. So, the common
twice.
difference is 90+9=10.

LO 46. To get from the 23” term to the 25" term, we add the
aed oh gees
common difference 25-23 = 2 times. This adds a total of
Adding the common difference twice adds a total of 28> 68)
35-21 = 14. So, the common difference is 14+2=7. So, the common difference is 129. Since dividing by
39. To get from 74 to 41, we add the common difference 3 a number is the same as multiplying by that number’s
times.
reciprocal, =t-$5eb
wehave4ee reeTt
47. To get from the 10" ae “ the 30" term, we add the
STR
04, 45,
geeAdy OSaot common difference 30-10=20 times. This adds a total

Adding the common difference 3 times adds a total of of68-4=64.


So,thecommon
difference
isof=g
41-74=-33. So, the common difference is -83+3 =-11. We get the 20" term by adding the common difference
10 times to the 10" term. The 10" term is 4, so the 20"
40. To get from 30 to 323, we add the common difference 5
term is 4+10-2= 4432 = 36.
times.

We add the common difference 10 times to get from


the 10" to the 20" term, and 10 times to get from the
Addingthe common difference5 times addsa total of 20" term to the 30" term. So, the 20" term is halfway
324-39=21=3
2 2 er between the 10" and 30" terms. The number halfway
: xi
So,thecommon
difference
is225=3.Bee? 4168-72_36
between 4 and68 is their average: ———=e :
41. To get from 19 to 64, we add the common difference 3 48. If we let d represent the common fea of the
times. This adds a total of 64-19 = 45. So, the common sequence, then the five terms in the sequence are
difference is 45+3=15.
40, 40+d, 40+2d, 40+3d, 40+4d.
We use the common difference to fill in the blanks as
The sum of these terms is 40(5)+(d+2d+3d+4d), which
shown.
simplifies to 200+ 10d.
+15 “ISR IS to) 1S 415
LR LAR LR LAR LRT We are told that the sum of all five terms is 80. This gives
19, 34, 49, 64, 79, 94,109 the equation 200+ 10d = 80. Subtracting 200 from both
sides, we have 10d =-120. Dividing both sides by 10
42. To get from 98 to 112, we add the common difference gives d=-12.
twice. This adds a total of 112-98=14. So, the common
So, the common difference of the sequence is -12.
difference is 14+2=7.
We use the common difference to fill in the blanks as
shown. Since the difference between consecutive terms in an
arithmetic sequence is always the same, the five terms in
+7 +7 47 +7 +7 +7
PPR ® a this sequence balance around the middle term.
98, 105, 112, 119, 126, 133, 140
-d -d: +d +d
43. Togetfrom322.304,wesubtract
14(oradd-13).
So, it

thetermafter304is304-11=
29.
Working
ea wecanfindnetermsthatcome So, the 3 term is equal to the average of the terms.
before
32bycontinually
adding15,asshownbelow. The sum of the five terms in the sequence is 80, so the

+1"HISGi ~+15 -14 average is 2 = 16. Therefore, the 3 term is 16.


ENah ENes 7
38,363,35,331,32,304,29
44. Togetfrom 10to22weaddthecommon difference
5 We add the common difference twice to get from the
times. This
difference
a ddsa
is 12+5=
eet ofaoe
=28
10=12.
So, t hecommon 1%term of 40 to the 3” term of 16. So, the common
16-40 _-24_ 45
difference is ———
5 3
We use the common yk to fill in the blanks as
shown.

Beast Academy Practice 5C Sequences Chapter 7 Solutions


49. The first term of the sequence is 15 and the common 56. The first term of the sequence is 18 and the common

difference is 4. difference is 6.

+6 +6 +6 +6
+4 +4 +4 +4
LX TARTALA
OO rece VieSiliice 18, 24, 30, 36, 42, ...
To get to the 2" term, we add 1 six to 18.
To get to the 2" term, we add 1 four to 15.
To get to the 3% term, we add 2 sixes to 18.
To get to the 3” term, we add 2 fours to 15.
To get to the 4" term, we add 3 sixes to 18.
To get to the 4" term, we add 3 fours to 15.
To get to the n" term, we add (”-1) sixes to 18.
To get to the 10" term, we add 9 fours to 15.
So, the n"" term is 18+(n-1)6. Distributing the 6 and
So, the 10" term is 15+9(4) = 15+36 = 51.
simplifying gives 18+(n-1)6 = 18+6n-6 =6n+12.
50. The first term of the sequence is -11 and the common
57. The first term of the sequence is 4 and the common
difference is 5.
difference is 15.
+5 +5 +5 +5
a tioeerts £15

et sO, 1574, 9... ra ra


4, 19, 34, 49,...
To get to the 2 term, we add 1 five to -11.
To get to the 2" term, we add 1 fifteen to 4.
To get to the 3% term, we add 2 fives to -11.
To get to the 3 term, we add 2 fifteens to 4.
To get to the 4" term, we add 3 fives to -11.
To get to the 4" term, we add 3 fifteens to 4.
To get to the 40" term, we add 39 fives to -11.
To get to the n" term, we add (n- 1) fifteens to 4.
So, the 40" term is -11+39(5) =-11+195 = 184.
So, the n™term is 4+(n-1)15. Distributing the 15 and
ot. The first term of the sequence is 5 and the common simplifying gives 4+(n-1)15=4+15n-15=15n-11.
difference is -7.
58. The first term of the sequence is -13 and the common
+(-7) +(-7) +(-7) +(-7) difference is 8.
LATA LR TA +8 +8
5, -2, -9, -16, -23, ...
“13-5, 3,'24
To get to the 2™ term, we add one -7 to 5.
To get to the 3 term, we add two -7’s to 5. To get to the n" term, we add (n-1) eights to -13.
To get to the 4" term, we add three -7’s to 5. So, the n" term is -13+(n-1)8. Distributing the 8 and
simplifying gives -13+(n-1)8 = ens 8=8n-21.
To get to the 15" term, we add fourteen -7’s to 5.
So, the 15" term is 5+ 14(-7) =5+(-98) =-93. 59. The first term of the sequence isz 2 and the common

52. The first term of the sequence is -29 and the common differenceiSs5. 9210-9.
a tAeae 41
difference is 10.
+t oT
tOe Ove 10! 10 rare
T&L LATA ORES)
29-19, =O Vi Vl gece 4’ Pay 4:
To get to the 100" term, we add 99 tens to -29. Togettothen"term,
weadd(n-1)one-fourths
to2
So, the 100" term is -29+99(10) = -29+990 = 961. So,then"termis2+(n-1)-4.Distributing
the+and
53. To get to the 13" term, we add the common difference simplifying
gives
to the first term 12 times. The first term is 9, and
9 (n i) ey:a
at “4 aa4
the common difference is 8. So, the 13" term is
pat
=qnt2.
9+12(8) =9+96=105.
54. To go from the 1‘ term to the 100" term, we add the
common difference 99 times. So, to go from the 100" Rewriting
2astheequivalent
fraction
vawehave
term to the 1 term, we subtract the common difference
QeAO2Ad
99 times. PMS
RY
Ms Tse
Each term in the sequence is a fraction with a
The100"termis40,andthecommondifference
isi. denominator of 4 and a numerator that is 8 more than its
So,thefirsttermis40-99-14
=40-33=7. position number in the sequence. For example,
55. To go from the 12" term to the 15" term, we add the
14+8.29
the 1%term is ——
common difference 3 times. The difference between AP WA?
the 12" and 15'" terms is 106-85 = 21. So, the common the 2" term is 2+8
difference is 106-88on = =, 4
the 3 term is aa8 = 4 and so on.
Then, we find the first fe of the sequence by
So, the n" term in the sequence is ue
subtracting the common difference from the 12" term 11
times. So, the first term is 85- 11(7) =85-77 =8. Note that this expression is equivalent to the one we
arrived at in the first solution to this problem. We have
1 wh n
as 8 n+B
gnt2=7t2=7t7= or

Sequences Chapter 7 Solutions Beast Academy Practice 5C


60. To get to the 20" term of the sequence, we add the If the path begins 3-4, then the next number is 5.
common difference to the first term 19 times: The first If the path begins 3-6, then the next number is 9.
term is a, and the common difference is 3. So, the 20" If the path begins 3-10, then the next number is 17.
term is a+19(3) =a+57. If the path begins 3-12, then the next number is 21.
61. To get to the 101° term of the sequence, we add the Only 3-12-21 has three numbers that appear in the grid.
common difference to the first term 100 times. The first The common difference of 3-12-21 is 9. Since 21+9=30
term is 6, and the common difference is d. So, the 101 is not in the grid, there are no other numbers in this path.
term is 6+100d, or 100d +6.
The remaining numbers are 4, 6, 8, and 10, which form
62. The 1* term of the sequence is 20 and the 2™ term is 32, an arithmetic sequence with common difference 2.
so the common difference is 32-20 = 12.
So, we connect 3-12-21 and 4-6-8-10 as shown.
To get the k* term in the sequence, we add the common
difference to the first term (k-1) times. So, the k'" term is
20+(k-1)12 =20+12k-12 = 12k+8.

We want to know the value of k that makes 12k+8 equal


to 500. This gives the equation
12k+8 =500. 66. We write the numbers in the grid from least to greatest:

Subtracting 8 from both sides, we have 12k = 492. 12, 18, 24, 26, 30, 34.
Dividing both sides by 12 gives k= 41. Since all six numbers do not form an arithmetic sequence,

Check: the 41* term is ) =20+480 = 500. W there must be two 3-term arithmetic sequences.

Since 12 is the smallest number in the grid, it is one end


of a path. There is only one arithmetic sequence we can
make that begins with 12.
include at least three numbers. So, all five numbers are VARS
part of the same path, forming an arithmetic sequence 12, 18, 24, 26, 30, 34
with common difference 7. So, one sequence is 12-18-24, and the remaining
The only way to connect all five numbers in an order that numbers form the arithmetic sequence 26-30-34.
forms an arithmetic sequence is shown below. We connect the numbers in these sequences as shown.

64. The 21 in the top-left square is the largest number in the 67. We write the numbers in the grid from least to greatest:
grid. So, it must be one end of a path. Also, the path from
56, 57, 61, 62, 66, 67.
21 must pass through 12 or 15 next.
Since all six numbers do not form an arithmetic sequence,
If the path begins 21-12, then the next number is 3.
there must be two 3-term arithmetic sequences.
However, the four remaining numbers (7, 9, 15, 17) do
not form an arithmetic sequence, and none of them can Since 56 is the smallest number in the grid, it is one end

be part of the 21-12-3 path. of a path. There is only one arithmetic sequence we can
make that begins with 56.
If the path begins 21-15, then the next number is 9. The
four remaining numbers (3, 7, 12, 17) do not form an Lee a
56757, 61, 62, 66, 6/
arithmetic sequence. However, 3 can be included at the
end of the 21-15-9 path, and the remaining numbers So, one sequence is 56-61-66, and the remaining
form an arithmetic sequence: 7-12-17. numbers form the arithmetic sequence 57-62-67.

We connect the numbers in 21-15-9-3 and 7-12-17 as We connect the numbers in these sequences as shown.

shown.

68. We write the numbers in the grid from least to greatest:


The smallest number in the grid is 3, so it is one end of 22, 24, 26, 30, 32, 34, 38, 40.
a path. The only numbers we can draw a path to from 3
The only arithmetic sequences we can make that start
are 4, 6, 10, and 12. We consider each possibility.

Beast Academy Practice 5C Sequences Chapter 7 Solutions


with 22 begin with 22-24-26 or 22-26-30 or 22-30-38. It is So, the two sequences are 74-77-80-83 and 86-89-92.

impossible to draw a path through 22-24-26 or 22-30-38 We connect the numbers in these sequences as shown.

in the grid. So, one path begins with 22-26-30.

FETE SON
22, 24, 26, 30, 32, 34, 38, 40

Then, the only arithmetic sequence we can make with


the smallest remaining number (24) is 24-32-40.
71. We write the numbers in order and consider sequences
Za aN that start with 10. Dashed lines indicate numbers that
22, 24, 26, 30, 32, 34, 38, 40
Pe
oR Se a might be included in the sequence.

Finally, the two remaining numbers (34 and 38) must go TASCS v ~
=-=> =-=>

at the end of the sequence 22-26-30. 10, 22, 28, 34, 40, 46, 52, 58
LON ea Oe
22, 24, 26, 30, 32, 34, 38, 40
res 10, 22, 28, 34, 40, 46, 52, 58
We connect the numbers in 22-26-30-34-38 and 24-32-40 A ea ae oe
10, 22, 28, 34, 40, 46, 52, 58
as shown.
In the second and third cases shown above, there is
no way to make an arithmetic sequence with the five
remaining numbers. So, one path begins with 10-22-34,
as shown in the first case above.
The only arithmetic sequence we can make with the
69. We write the numbers in the grid from least to greatest: smallest remaining number (28) is 28-40-52.

27, 34, 43, 53, 59, 72. The remaining numbers (46 and 58) can only be included
The only arithmetic sequence we can make that begins
at the end of the 10-22-34 sequence.
with 27 is 27-43-59. The remaining three numbers form LW SR ee
the arithmetic sequence 34-53-72.
10, 22, 28, 34, 40, 46, 52, 58
Wee ee
We connect the numbers in these sequences as shown. We connect the numbers in 10-22-34-46-58 and 28-40-52
as shown.

70. We write the numbers in the grid from least to greatest:

74, 77, 80, 83, 86, 89, 92.


72. We write the numbers in the grid from least to greatest:
All seven numbers form an arithmetic sequence with
common difference 3. However, it is not possible to draw 14, 27, 40, 53, 66, 79, 92.
a path through all seven numbers. So, we must have one All seven numbers form an arithmetic sequence with
path with 3 terms and one path with 4 terms. There are common difference 13. However, it is not possible to
only three possibilities: draw a path through all seven numbers. So, we must
have one path with 3 terms and one path with 4 terms.
NaN
74, 77, 80, 83, 86, 89, 92 There are only three possibilities:
ALYY
ToT N
FTN 14, 27, 40, 53, 66, 79, 92
74, 77, 80, 83, 86, 89, 92 LAW,
\LWYY NY ARN ARN
ET ON 14, 27, 40, 53, 66, 79, 92
LAY
74, 77, 80, 83, 86, 89, 92
Nese | ae pe ae ei
14, 27, 40, 53, 66,'79, 92
In the first case, there is no way to connect 74-77-80 Se
without blocking the path for 83-86-89-92. Similarly, In the third case above, if we connect 14-40-66 then
in the third case, there is no way to connect 74-80-86-92 there is no way to connect 27 and 53 in the grid.
without blocking the path for 77-83-89. In each of the first two cases, one path includes
14-27-40 and the other path includes 66-79-92. We must
determine which of these two paths includes 53.

Sequences Chapter 7 Solutions Beast Academy Practice 5C


We temporarily ignore the 53, and draw both paths. We connect these numbers as shown.
Then, the 53 can only be connected to the end of the
14-27-40 path, as shown.

73. Finally, we can connect 39 and 27 to complete the path


The smallest number in the grid is 4. The only arithmetic
that begins 75-63-51.
sequence we can make that begins with 4 is 4-12-20.

tA, ae Pa Be
4,529,122)13,15, 17,20).25
The six remaining numbers do not form a single
arithmetic sequence, so they must form two 3-term
arithmetic sequences. There is only one way we can
make two 3-term arithmetic sequences, as shown below.

pa
Soto. LOO Connecting 9-39-69 leaves a set of numbers that are
oe
impossible to group into paths, and connecting 9-45-81
We connect the numbers in the sequences 4-12-20,
isolates 27. There is no other sequence that includes 9.
5-15-25, and 9-13-17 as shown.
So, this is the only solution.
75. The largest number in the grid is 60, the second-largest
number is 50, and the smallest number is 40. So, the
only arithmetic sequence we can make with 60 is
60-50-40. We connect these numbers as shown below.

74. We write the numbers in order. These are the numbers


we use to skip-count by 6 from 9 to 81, excluding 15:

9,245 27, 33, 39; 45,51, 57; 63,69, 75, 81:


We consider sequences with smallest term 9:

9-21-33 cannot be connected in order on the grid.

Connecting 9-27-45 forces 81 to be connected to


33 or 75. We cannot make an arithmetic sequence that This leaves us with the following numbers:
begins 81-33, and connecting 81-75-69 isolates the 33.
41, 42, 43, 44, 45, 46, 47, 48, 49.
Connecting 9-33-57 as shown below forces 75 to be
connected to 63, creating a sequence whose third term is The sequence that begins with 41 must have a second
term that is 45 or less. However, connecting 41 with 42,
51. So, if 9-33-57 are on the same path, 75-63-51 must
44, or 45 will isolate a group of numbers that do not form
also be on the same path.
an arithmetic sequence. So, one sequence begins with
41-43-45.
Then, the only arithmetic sequence we can make with
first term 42 begins with 42-44-46.
We connect the numbers in 41-43-45 and 42-44-46 in the
only way that does not isolate other numbers in the grid.

Since 81 cannot be part of a sequence that includes any


of the remaining numbers, we must connect 81 to the
end of the 9-33-57 path.
Then, since 69 cannot be on either of the existing paths,
it must be part of a new path. The only sequence that
can be made with the remaining numbers is 69-45-21.

Beast Academy Practice 5C


Sequences
Chapter
7Solutions
[121]
Finally, there is no way to connect the remaining three b. The sequence has 1* term 5 and 3” term 35, so the

numbers
(47,48,49)inanorderthatformsanarithmetic common difference
is2 5=2 =15.Weusethisto
sequence.
So,47and49areincluded attheendofthe fillintheremainingblanksasne
41-43-45
path,and48isincludedattheendofthe ARTIS AishRn ee
42-44-46
; path. 5,LALA LALALALA
20, 35, 50, 65, 80, 95,...

C. The sequence has 1* term 5 and 4" term 35, so the


common difference is 22=2 = 30 —10. We use this to

fill in the remaining blanks as shown.

+10 ©+10... 410) | £109 24100"+10


LALA LALA LALA
5, 15, 25, 35, 45, 55, 65,...
d. The sequence has 1* term 5 and 6" term 35, so the
common difference is 22-2 = 32 = 6. We use this to fill

in the remaining blanks as shown.


76. We can organize the numbers in this grid into a group of
FO) UO +6 +6 +6 +6
“small numbers” and a group of “big numbers.”
LALALALALALA
Small numbers: 1, 5, 9, 13, 17
a 11, 17, 23, 29, 35, 41,
Big numbers: 41, 45, 49, 53, 57, 61, 65
78. a. We add the common difference twice to get from
Since we cannot connect all five small numbers in the 27 to 39. So, the common difference is lel—=—= =6
grid without isolating other numbers, there must be a
We use this to fill in the remaining blanks as shown.
path that begins with a small number and includes a big
number. ENENpee +6 v4 +6
BEN TALATLA
The greatest possible difference between two small gi 15, 221, 27, 33, 39, 45,.
numbers is 17-1 = 16. The least possible difference b
. We add the common difference 3 times to get from
between a big and a small number 41-17 = 24. So, 27 to 39. So, the common difference is SS
———= =4
there cannot be a path with two small numbers and a big
We use this to fill in the remaining blanks as shown.
number.
=4" -~4 +4 +4. +4 #444

Therefore, there must be a path that begins with a small FN EN LIRA TERT
number followed by two or more big numbers. The only
19, 23, 27, 31, 35, 39, 43, ...
arithmetic sequence we can make that begins with a
small number followed by a big number is 17-41-65. Cc.We add the common difference 4 times to get from
27 to 39. So, the common difference is 0-2/2
—— 33
The remaining small numbers must form the arithmetic
We use this to fill in the remaining blanks as shown.
sequence 1-5-9-13, and the five remaining big numbers
-3 +3 +3 +3 +3 +3
must form the arithmetic sequence 45-49-53-57-61. BN TRL LARA
We connect the numbers in these sequences in the only 24, 27, 30, 33, 36, 39, 42,..
way possible, as shown below.
d. We add the common difference 6 times to get from
27 to 39. So, the common difference is 32=27 == =2.
We use this to fill in the remaining blanks as shown.
+2 +2 +2 +2 +2 +2

LL LARLARS ALS
27, 29, 31, 33, 35, 37, 39,...
79. To get from 9 to 33, we add a total of 33-9 = 24. Since
every term is an integer, the common difference must be
a factor of 24.

The factors of 24 are 1, 2, 3, 4, 6, 8, 12, and 24. So,


there are 8 possible common differences.

80. To get from 15 to 46, we add a total of 46-15 =31. Since


or 5 ee peer ae ee : or, Rena " Risu everytermisaninteger,thecommon difference
mustbe
common difference
is35-5=30.Weusethistofillin a factorof31.
theremaining
blanksasshown. 31isprime, soitsonlytwofactorsare1and31.So,
jahcaeyLagae a, +30 thereare2possible common differences.
5 36 6S, ee 125, o\5h 1 Bix. 81. Thedifference between37and13is37-13=24.Since
every term is an integer, the common difference of the

[122 Sequences Chapter 7 Solutions Beast AcademyPractice5C


sequence is some factor of 24.
The factors of 20 are 1, 2, 4, 5, 10, and 20. However, the
We test each possible common difference, stopping if we sequence has seven terms, and we cannot go from 33 to
come across more than one prime between 13 and 37. 73 with common difference 1, 2, 4, or 5 using just seven
terms. For example, if the common difference is 5, then
1: 13,14,15,16,2, 18,@9,... * the sequence must include at least nine terms:
2: 13,15,(, 69.... x 33, 38, 43, 48, 53, 58, 63, 68, 73.
3: 13,16,(9),22,25,28,GB},
... x Therefore, the common difference can only be 10 or 20.
4: 13,(2),21,25,9, ... x If the common difference is 10, then we can have any of
6: 13,(9),25,B}... x the following sequences.

8: 13,21,@9, 37 v 13, 23, 33, 43, 53, 63, 73.


12:13,25,37 * 23, 33, 43, 53, 63, 73, 83.
24:13,37 x 33, 43, 53, 63, 73, 83, 93.
Only the common difference 8 gives a sequence with If the common difference is 20, then we can have either
exactly one prime between 13 and 37. That prime is 29. of the following sequences.
82. The common difference of a sequence of integers that
13,33, 53, 73, 93, 113, 133:
includes 27 and 34 must be a factor of 34-27 =7.
33, 53, 73, 93, 113, 133, 153.
The common difference of a sequence of integers that
We circle the terms that appear in at least one of the
includes 34 and 49 must be a factor of 49-34 =15.
- sequences shown above.
The only number that is a factor of both 7 and 15 is 1.

83.
So, the greatest possible common difference is 1.
The common difference of a sequence of integers that
3 ®) 28 38
63)458)
86. We begin by writing the eight terms, using blanks for the
includes 20 and 32 must be a factor of 32-20 = 12.
missing units digits.
The common difference of a sequence of integers that
includes 32 and 50 must be a factor of 50-32 = 18. Sig Oy a5 AL, ay On, Oa oe
So, the common difference is a factor of both 12 and 18. If the common difference is 5 or greater, then the third
To make the 10" term as large as possible, we use the
term is at least 10 more than the first term. Since the first
greatest common factor of 12 and 18, which is 6. and third terms are both in the 30’s, they must be less
We also make the 10" term as large as possible by than 10 apart. So, the common difference must be less
assuming that the smallest given number (20) is the than 5.
1* term. The 10" term of an arithmetic sequence The 3% term in the sequence is at most 39. If the
with 1s term 20 and common difference 6 is common difference is 3 or less, then the 6" term is at
20+9(6) = 20+54 =74. most 39+3(3) = 48. However, the 6" term has a tens digit
The entire sequence is shown below: of 5. So, the common difference cannot be 3 or less.

20, 26, 32, 38, 44, 50, 56, 62, 68, 74 Therefore, the common difference must be 4.

84. The common difference of a sequence of integers that There are two sequences we can make with a common
includes 23 and 58 must be a factor of 58-23 = 35. Since difference of 4:

the terms are non-consecutive, the common difference 30, 34, 38, 42, 46, 50, 54, 58,
cannot be 1. The remaining factors of 35 are 5, 7, and 35.
31, 35, 39, 43, 47, 51, 55, 59.
Every term in the sequence can be written as 23 plus or 87. The units digits of the middle two terms are 4 and 9. So,
minus some multiple of the common difference. the common difference of this sequence must have units
If the common difference is 5, the smallest positive
integer we can have is 23-4(5) = 23-20 =3.
digit5.Weusethisfacttofillinthemissing
unitsdigits.
If the common difference is 7, the smallest positive oe A! 8 Dy 5tte ONae aoe 9
integer we can have is 23-3(7) = 23-21 =2. Then, only a common difference of 15 takes us from 39
If the common difference is 35, the smallest positive to 84 in three steps.
integer we can have is 23. We use the common difference of 15 to fill in the

So, the smallest possible positive integer is 2. remaining blanks as shown.

. Since the sequence includes the terms 33, 53, and 73,
and all terms are positive integers, its common difference
BA 3.9 8 4s 629" She 29
must be a factor of 53-33 = 20 and a factor of 73-53,
which also is 20.

Beast Academy Practice 5C


Sequences
Chapter
7Solutions
[123]
91. Step 1: Step2: Step 3:

88. The top row contains a 1 and a 4. The only


arithmetic sequence of positive integers
that can be made with 1 and 4 is (1, 4, [7]).
So, the top-right square is 7.

The middle row is (2], 5, 8) or (5, 8, [11).


Only placing 2 in the middle square also
creates an arithmetic sequence in the
middle column: (2, 4, 6).

The bottom row is (8], 6, 9) or (6, 9, [12].


Only placing 3 in the bottom-left square
also creates an arithmetic sequence in the
left column: (1, 3, 5).

89. The top row contains a 2 and a 5. The only


arithmetic sequence of positive integers
that can be made with 2 and 5 is (2, 5, [8]).
So, the top-middle square is 8.

The left column contains a 2 and a 7.


The only arithmetic sequence of positive
ea Te can be made with 2 and . is

The right column is ([2], 5, 8) or (5, 8, [11]).


Only an 11 in the middle-right square also
creates an arithmetic sequence in the
middle row: (7, 9, 11). The left column is
([4], 6, 11) or (6, 11, [16)).
The middlecolumnis ({7],8, 9) or (8,9, [10]). The middle column is
Only a 10 in the bottom-middle square ([15],24, 33) or (24,33, [42)).
also creates an arithmetic sequence in the
bottom row: (8, 10, 12).
Only a 1 in the bottom-left square and a
15 in the bottom-middle square gives an
90. Step 1: Step 2: arithmetic sequence in the bottom row:

Step 3: The left column is (1, [2], 3) or


(1, 3, [5)). If the top-left square is 2, then the
only arithmetic sequence we can make in
the top row is (2, 25, 48), so the top-right
square would be 48. However, (14, 16, 48)
is not an arithmetic sequence.
So, the top-left square is 5.

Step 4: The top row is (5, [15],25) or


(5, 25, [45]). Only a 15 in the top-right
square also gives an arithmetic sequence in
the right column: (14, 15, 16).

Beast Academy Practice 5C


Sequences Chapter 7 Solutions
15 or 22
Step 3:
In the left column, the difference
The middle column is
between 8 and 1 is 7. So, the common
((28],56, 84)or(56,[70],84)or(56,84,[172)).
difference is 7. Therefore, the numbers
The right column is
in the left column are (1, 8, [15],[22]),
([1O),
14,18)or(14,
[16],
18)or(14,18,
[22]
and the top-left square is 15 or 22.
28,70,0r112 10, 16, or 22

In the top row, there is no arithmetic sequence we can


Only a 28 in the middle column and a 16 make with 22, 34, 72, and one other term. So, the top
in the right column make an arithmetic top-left square is 15, and the square below the 8 is 22.
sequence in the row with 22: (16, 22, 28).
Then, we can only place a 53 in the top row to make an
arithmetic sequence: (15, 34, 53, 72).

96. The middle column is (5, [12], 19) or


(5, 19, [33]). Only a 33 also makes an
arithmetic sequence in the row with
11 and 55: (11, 33, 55).

98. The difference between the given terms in the left column
The leftcolumnis is 19-9=10. So, the common difference in the left
((15],35, 55)or (35,[45],55)or (35,55,[75]). column is either 10 or 10+2=5. Therefore, every number
The rightcolumnis in the left column has units digit 9 or 4.
([Z],
9,11)or(9,[1O),
11)or(9,11,
[13}). In the bottom row, we have 23-7 = 16. So, the common
difference is either 16 or 16+2=8. This gives the
15, 45, or 75
following possibilities for the numbers in the bottom row:
(7, 23, [39],[55)) or (7, [15], 23, [31)).
Only a 15 in the left column and a 10 in the
right column makes an arithmetic sequence Of these numbers, only 39 has a units digit of 9 or 4, so
in the row with 5: (5, 10, 15). 39 goes in the bottom-left square, and 55 goes in the
remaining empty square in the bottom row.
Then, the missing term in the left column is 29.

97. In the right column, the common difference is 2 or 1.


So, the smallest term that could be in the right column is
32-2(2) = 28, and the largest term that could be in the
right column is 34+2(2) = 38.
The difference between the given terms in the bottom In the top row, the difference between 46 and 29 is 17.
row is 91-1 =90. So, we have the following possibilities So, the common difference is 17. This gives the following
for the common difference: possibilities for the numbers in the top row:
¢ Common difference 90 gives (1, 91, |181],|271)). ((12],29, 46, [63})or (29, 46, [63],[80)).
* Common difference 90+2 = 45 gives (1, [46], 91, [136]).
So, the number in the top-right square is 12, 63, or 80.
* Common difference 90+3 = 30 gives (1, [31], [61], 91). Of these, only 12 can be used to make an arithmetic
The rightmost square of the bottom row is between 28 sequence in the right column with 23 and 45. So, the
and 38. Only (1, [31], [61], 91) includes a term between 28 top-right square is 12, and the sequence in the right
and 38, so the rightmost square in the bottom row is 31, column is (12, 23, 34, 45).
and the other empty square is 61. Then, the remaining We place a 63 in the top row to complete the puzzle.
empty square in the right column is 33.

|
| Beast Academy Practice 5C
Sequences
Chapter
7Solutions
| 125
|
99. We use the strategies discussed in previous problems to Since 31 cannot be part of the 5-term arithmetic
complete the puzzle as shown. sequence in the right column that includes 6 and 45,
the bottom-right square is 32, and the remaining empty
square in the bottom row is 31.
Then, the common difference in the right column is
45-32 = 13, and the missing numbers are 19 and 58.

100. In the middle column, the difference between 22 and


17 is 5. So, the common difference is 5. Therefore, the
missing numbers in the middle column are 7 and 12.

In the middle row, we can go from 1 to 4 with a common


difference of 1 or 3. However, we cannot go from 1 to 13
in just five terms with a common difference of 1. So, the
Since 58 cannot be part of the 5-term arithmetic
common difference is 3. Therefore, the missing numbers
sequence in the middle row that includes 13 and 22,
in the middle row are 7 and 10.
the rightmost square in the middle row is 19, and the
Since 7 is the only number that can go in both the middle remaining empty square in the middle row is 58.
row and the middle column, we place a 7 in the center
square. Then, we place the 12 and 10 as shown.

We use the previous strategies to complete the puzzle as


Then, we use the strategies discussed in previous shown.
11 and 21
problems to complete the puzzle as shown.

101. In the left column, the common difference is 22-15=7.


So, the missing numbers in this column are 8 and 29.

8 and 29

102. In the bottom row, the common difference is 35-28 = 7.


Since 8 cannot be part of the 5-term arithmetic sequence
Since 28 and 35 are multiples of 7, every number in the
in the bottom row that includes 30 and 33, the bottom-left
bottom row is a multiple of 7.
square is 29, and the remaining empty square in the left
In the left column, the common difference is a factor
column is 8.
of 43-16 =27. We can eliminate 1, 3, and 27. So, the
Then, the common difference in the bottom row is 1, and
common difference in the left column is 9. Therefore, the
the missing numbers in the bottom row are 31 and 32.
missing numbers in the left column are 25, 31, and either
7 or 52. Only 7 is a multiple of 7, so the square in the left
column and bottom row is 7.

Then, the remaining empty squares in the bottom row


are 14 and 21, and the remaining empty squares in the
left column are 25 and 34.

Sequences Chapter 7 Solutions Beast Academy Practice 5C


difference in the bottom row is 5 or 10. If the common
difference is 10, then every number in the bottom row
has units digit 3. However, we cannot make an arithmetic
sequence in the right column with 90, a number with
units digit 3, and one of (20, 26, or 32). So, the common
difference in the bottom row is 5.

Therefore, the missing numbers in the bottom row are


38, 48, and 58.
We consider the empty square above the 11. We cannot 20, 26, and 32

makea5-term
arithmetic
sequence
withthenumbers
11, es ee
17,and21.So,weplace
to the 14above
left of the the11,andplace
14. 21 | ¥/38]8jaa]ia]%|N|
| oe Bo
25 and 34
RINEIEIOP
cs
38, 48, and 58

We fill the blanks shown below in the only way that gives
an arithmetic sequence in the left and right columns.

If we place 34 to the right of the 22, then


e the common difference in the top row is 4, and

¢ one of the remaining empty squares in the top row is


30, and the other is 18 or 38.

However, we cannot place 18, 30, or 38 in the right


column. So, 34 cannot go to the right of 22. So, we place
25 to the right of the 22, and place 34 beneath the 25.
Therefore, the common difference in the top row is 1, and
the missing numbers in the top row are 23 and 24.

104. In the top row, the common difference is 65-54 = 11. So,
the missing numbers in the top row are 21, 32, 43, and 76.
In the bottom row, the common difference is a factor of
87-22 =65. So, the common difference is 1, 5, 13, or
65. We cannot go from 22 to 87 in seven terms using a
common difference of 1 or 5, and we can eliminate 65.
So, the common difference in the bottom row is 13.

We use the previous strategies to complete puzzle as Therefore, the missing numbers in the bottom row are
shown. five of the following: 9, 35, 48, 61, 74, and 100.

21, 32, 43, and 76

Five of 9, 35, 48, 61, 74, and 100


103. In the top row, the common difference is a factor of ee
14-8=6. We cannot go from 8 to 44 in seven terms We fill the blanks shown below in the only way that
using a common difference of 1, 2, or 3. So, the common creates an arithmetic sequence in each column.
difference is 6.

Therefore, the missing numbers in the top row are


20, 26, and 32.
In the bottom row, the common difference is a factor of
43-33 = 10. We cannot go from 33 to 63 in seven terms
using a common difference of 1 or 2. So, the common Teer see

Beast Academy Practice 5C Sequences Chapter 7 Solutions


The remaining empty square can only be 48, so we This gives 12 pairs of numbers that each sum to 26. So,

complete the puzzle as shown. the sum of all 12 pairs is 12°26 = 312. This is the total of

ro 76|o5]
a]=e
two copies of the sum, so one copy is 312+2 = 156.

108. Factoring an 8 from each term in the sum, we have


3
88+80+72+64+56+48+40+32+24+16+8
=8(114+10*9+8+7+6+5+4+3+2 +1).
This is 8 times the sum of the first eleven positive
Triangular Neen : integers, or 8 times the 11" triangular number.
105. The 40" triangular number is given by the number of dots The 11* triangular number is 11-12 So 8 times the 11"
in the triangular pattern with 40 rows. triangularnumber is 8°ue =4°11°-12=528.

ee
ee
@8SO 40 rows We add two copies of the sum, with one copy written in

eee. 000 reverse.


88+80+72+---+24+16+ 8
This is equal to the sum 1+2+3+::-+38+39+40. 8 +16+24+---+72+80+88
To compute this sum, we add two copies of it, with one 96+96+96+---+96+96+96

copy written in reverse.


This gives 11 pairs of numbers that each sum to 96. So,
1+2+3 +---+38+39+40 the sum of all 11 pairs is 11*96 = 1,056. This is the total
AQ+39+38+:-+ 342+ 1
of two copies of the sum, so one copy is 1,056+2 = 528.
41+41+414+--+41+41+41
109. Factoring a 6 from each term in the sum, we have
This gives 40 pairs of numbers that each sum to 41. So,
the sum of all 40 pairs is 40°41 = 1,640. However, this 6+124+18+:-:-+588+594+600
is the total of two copies of the sum we wish to compute. =6(1+ 2 4:34 :4+,98 +209 +100),
So, the value of one copy is 1,640+2 = 820.
This is 6 times the 100" triangular number. So, the sum
So, the 40" triangular number is 820. is equal to 6-4°2-101 —3. 100-101 =30,300.
106. To simplify 1+2+3+---+(n-2)+(n-1)+n, we use our 110. Writing each mixed number as a fraction, then factoring 4
strategy from the previous problem. We add two copies from each term in the sum, we have
of this sum, with one copy written in reverse.
AEN bt Sree 3
gtlotio taste +115
Pee heute (2) bb) tat 4°82) 16 80
Dee esl as Vee)oaisteda
okIno
talsmacees el | Limi mae
(n+1)+(n+1)+(n+1)+°°+(N+1)+(n+1)+(n+1) =4(1Or eernaier 20):
This gives n pairs of numbers that each sum to n+1. So, This is $ times the 20" triangular number. So, the sum is
the sum of all 1 pairs is n(m+1). Since this is the total of equal to
20°21 _ 4 Ba od
two copies of the sum, the value of one copy is ay Seay NI OE 120.
So, a simpler expression for the n" triangular number 1 1
is aol) You may have also distributed the 7 in the
111. We add two copies of this sum, with one copy written in
2 reverse.
numerator
togetza 51+ 52+ 53 +--+ 98 + 99 +100
107. Factoring a 2 from each term in the sum, we have 100+ 99 + 98 +--+ 53+ 52 + 51

2+44+6+8+10412+144+164+18+20+22+24
=2(1+2434+4+5+64+748 +49 +10+11+12). There are 50 terms in 51+52+53+-:-+98+99+100.
So, we have 50 pairs of numbers that each sum to 151.
This is 2 times the sum of the first twelve positive Therefore, the sum of all 50 pairs is 50° 151 = 7,550.
integers, or 2 times the 12" triangular number. This is the total of two copies of the sum, so one copy is
The 12" triangular number is nee So, 2 times the 12" 7,550+2 =3,775.
triangular number is 2 Ete =12°13=156.

We notice that 51+52+53+---+98+99+100 is the sum


We add two copies of the sum, with one copy written in of the first 100 positive integers minus the sum of the first
reverse. 50 positive integers.
2+4+6+4+:+20+22+24
24+22+20+--+6+4+2 (14+24+3+-+48+49+50+51+52+53+--+98+99
100)
26+264+26+::-+26+26+26 —(14+24+3+--+48+49+50)
51+52+53+---+98+99+100

Sequences Chapter 7 Solutions Beast Academy Practice 5C


This is equal to the 100" triangular number minus the Since k is the position number of a term in the sequence,
50" triangular number, which we compute as follows: we know that k is a positive integer. So, we look for two
100-101 50°51 _ 10,100 _2,550 consecutive positive integers, k and k+1, whose product
2 a 2 2 is 930. We know that 900 = 302, so 30 is a reasonable
=5,050-1,275 guess for k.
= 3,775. lf k= 30, then k(k+1) =30(31)=930. W
112. The 25" perfect square is 252. So, k=30. The 30" triangular number is 465.
The 25" triangular number is eonee

So, the 25" term in Ariana’s sequence is

952 725°26 115. We add two copies of this series with one copy written in
aes = 25?-25-13
= 2525-25-13 reverse.
445 +6 ++14+154+16
= 25(25-13)
164+15+14+--+6+5+4
= 25(12)
20+20+20+::-+20+20+20
= 300.
This gives 13 pairs of numbers that each sum to 20.
The sum of all 13 pairs is 13°20 = 260. Since this is
The following diagram shows the first several terms in the sum of two copies of the series, one copy equals
Ariana’s sequence. In each figure, we subtract the n' 260+2=130.
triangular number (given by the dots in gray) from the
n" perfect square (given by the total dots). The number
of remaining black dots gives the 7" term in Ariana’s We find the average of the terms in the series and
multiply by the number of terms.
sequence.
Since the terms in the series form an arithmetic
1 st 2 nd 3 rd 4 th 5 th
sequence, they balance around the median. Therefore,
°@ ©e@0e
their average is equal to the median.
1-1=0 @@ eee
4-3=1 © 6 4, 950.05. 7 2B OF LOT, deed pb Og 10
9-6=3 !

25-15=10

After the 1° term, the numbers in Ariana’s sequence are Since the median is 10, the average is also 10. The sum
the triangular numbers! For example, her 2"? term (1) of 13 terms with an average of 10 is 13°10 = 130.
is the 1* triangular number. Her 3 term (3) is the 2"° 116. We find the average of the terms in the series and
triangular number, and so on. multiply by the number of terms.
Continuing this pattern, Ariana’s 25" term is equal to the Since the terms in the series form an arithmetic
24" triangular number, which is anne = 12°25=300.
sequence, they balance around the median. Therefore,
113. The 997" triangular number is the sum of the first 997 their average is equal to the median.
positive integers. The 1,002" triangular number is the
sum of all of the same integers, plus the integers from 20, 30, 40, 50,! 60, 70, 80, 90
iA
998 to 1,002. So, the difference between the 1002"¢ and
997" triangular numbers is the sum of the integers from
998 to 1002. Since the median is 55, the average is also 55. The sum
of 8 terms with an average of 55 is 8°55 = 440.
14+24+3+---+995+996+997+998+---+1,002
—(14+2+3+---+995+996+997) 117. We find the average of the terms in the series and
multiply by the number of terms.
998+:--+1,002
Since the terms in the series form an arithmetic
The terms in 998+999+1,000+ 1,001 + 1,002 balance
sequence, they balance around the median. Therefore,
around 1,000. So, their average is 1,000, and their sum
their average is equal to the median, which is 54.
is therefore 5° 1,000 = 5,000.
The sum of 9 terms with an average of 54 is 9°54 = 486.
. The k* triangular number is equal to usa We are told
that the k*"triangular number is 465. So, we write an 118. The 15 terms in this series balance around their median,
which is 1. Therefore, the average of the terms is 1.
equation:
aa =465. The sum of 15 terms with an average of 1 is 15°1=15.

We can multiply both sides of the equation by 2 to get rid


of the denominator on the left hand side. This gives Each negative term in the sum cancels its corresponding
k(k+1) =930. positive term. For example, -6+6=0, -5+5=0, and so on.

Beast Academy Practice 5C Sequences Chapter 7 Solutions


After canceling the pairs of terms that sum to zero, we The average of the terms is the same as the average of
are left with O+7+8=15. thefirstandlastterms:
ee 8444=292.So,thesum
119. The 10 terms in this series balance around their median, of all 100 terms inthe sequence is400° 222 =22,200.
which
issent=2 = 38. Therefore, the average of the . To compute the sum of the series, we multiply the average
terms is 38. of the terms by the number of terms.
The sum of 10 terms with an average of 38 is 10°38 = 380. Since the terms in 15+21+27+:-:-+207 form an

120. Converting the terms to a sequence of consecutive arithmetic sequence, the average of the terms is equal to
integers makes them easier to count. the average of the first and last terms: ier e07 = 222 111.

The common difference in the sequence is 4. To find the number of terms, we convert the be

9;°13; 1721257545; 49; 53)'57}61: to a sequence of consecutive integers. The common


difference of the terms is 6.
Subtracting 5 from each term gives the multiples of 4.
DSce lye, 2..20r
4, 8, 12, 16, 20, ..., 40, 44, 48, 52, 56.
Subtracting 9 from each term, we get the multiples of 6.
Then, dividing each term by 4, we have

Ap28, Abo 10, 416127 13514.


6-12; 18;.-.41
98:
These steps did not change the number of terms in the
Dividingeach term by 6 gives
sequence. So, there are 14 terms. i yrsi eco
a
121. Since there are 14 terms in the sequence, the median is So, there are 33 terms in the series. The sum of 33 terms

the number halfway between the 7" and 8" terms. with an average of 111 is 33-111 =3,663.

The 7" term is 9+6(4) =33 and the 8" term is 9+7(4) =
33+37
So, the median is ——— = =70 - 35. INNS WS sa

122. Since the terms form an at a: sequence, their


. Thefirstfigure
inthepattern
has5;gumballs.
TI
Then,each:
figure has 4 more gumballs than the figure that came
average is equal to their median, 35. The sum of 14
before it.
terms with an average of 35 is 14°35 = 490.
@
123. In an arithmetic sequence, the average is equal to the @ ©
@ © ©
median. In this sequence, the median is the number
86@ @)00ce @200°0c08e
halfway between the two middle terms. Since the terms 7] ® e
in an arithmetic sequence balance around the median,
@ @
©
the number halfway between the two middle terms is the Figure 1 Figure 2 Figure 2
same as the number halfway between the first term and
the last term. The number of gumballs in each figure of the pattern
makes an arithmetic sequence with first term 5 and
Oral 2luene eo 30,01 .4 1,45,49, 59,57, 61 common difference 4.

So, to make the 8" figure in the pattern,


5+7(4) =5+28 =33 gumballs are needed.
The number halfway between 9 and 61 is 9+61
——— _70_35.
So, we can compute the average of this arithmetic
Each figure has a gumball in the center, plus four “arms.”
sequence, or of any arithmetic sequence, by taking
the average of the first term and the last term.
The number of gumballs in each arm is equal to the
figure number. For example, the 1* figure has 1 gumball
124. Since the middle term of the sequence is 43 and it is
per arm, the 2™figure has 2 gumbalis per arm, the 3”
an arithmetic sequence, the average of the terms in the
figure has 3 gumballs per arm, and so on.
sequence is also 43.
e °
In Problem 123, we learned that the average of an
arithmetic sequence is equal to the average of the first
® @ @
ee ee @@ 008 e0e@
and last terms. So, the average of the first and last terms @ @ @
ce) @
is 43. Therefore, the sum of the first and last terms is @
43°2=86.
Figure 1 Figure 2 Figure ?
125. The average of the terms in an arithmetic sequence with
So, the 8" figure has four arms with 8 gumballs each,
first term 19 and last term 81 is 19481
_100
_50, plus one gumball in the center. This gives a total of
The sum of 30 terms with an average of 50 is 4(8)+1=33 gumballs.
30°50 = 1,500.
To make the n" figure in the pattern, we begin with the
126. The first term of the sequence is 24 and the common 5 gumballs in the first figure and add 4 gumballs (n- 1)
difference is 4. So, the 100" term in the sequence is times.
24499(4) = 24+396 = 420.

Sequences Chapter 7 Solutions Beast Academy Practice 5C


So, the number of gumballs needed to make the n'
figure is 5+(n-1)4. Distributing the 4 and combining like 132. To get from the 1* figure to the 2"¢, we add 4 gumballs.
terms, this simplifies to 55+4n-4 =4n +1. To get to each figure after that, we add 2 more gumballs
than we added the previous time.

The n" figure in the pattern has 4 arms with n gumballs 1 1+4 4+4+6 14+4+6+8
@o.02 $280.02
each, plus one gumball in the center. So, the n'" figure - s°38sese §=6Seseses
has 4n+1 gumballs. eo %e 520° es
130. From the previous problem, we know that the kt figure Figure 1 Figure 2 Figure ? Figure 4
in the pattern uses 4k+1 gumballs. We are told that 221
So, the number of gumballs in the 10" figure is
gumballs are used in the k* figure, so
1+(4+6+8+10+12+14+16+18+20).
4k+1=221.
The sum in parentheses is an arithmetic series with 9
Subtracting 1 from both sides of the equation gives terms that have an average of ao = 12. So, the sum in
4k = 220. parentheses equals 9°12 = 108.
Dividing both sides of the equation by 4, we have k=55. Therefore, the number of gumballs in the 10" figure is
The 55" figure in the pattern uses 221 gumballs. 1+(44+64+84+10+12+14+16+18+20) =1+108 = 109.
131. The number of gumballs in each figure is given by a sum
of consecutive odd numbers. For example, the number of
If we ignore the gumballs on the right half of each figure,
gumballs in the 4" figure is the sum of the first four odd
the remaining gumballs (shown in black below) make a
numbers: 1+3+5+7= 16.
pattern of triangular numbers.
@1 Gece 20ee%3
0003 @ Se «= a Se®n §=—
So So80?
C0000 5 ene e® %s $e° %e8
O0000007

Figure 4 Figure1 Figure2 Figure2 Figure4


So, the number of gumballs in the 30" figure is the sum
Similarly,ifwe ignorethe gumballsonthe lefthalfof
of the first 30 odd numbers. In Problem 30, we learned
each figure,the remaininggumballsformthe same
that the n" odd number is 27-1. So, the 30" odd number
patternoftriangularnumbers.
is 2(30)-1 =59.
Go ee 28. 908
Therefore, the number of gumballs in the 30" figure is @ SeS §=—s aeS) = So S808
equal to 1+3+5+---+55+57+59. This is an arithmetic
ene @® $e° *e8
series! The 30 terms in this series have an average of
Figure1 Figure2 Figure3 Figure4
ame:
= = 30, so the sum of all of the terms is 30°30 = 900.
So, we can count the total number of gumballs in
each figure by adding the gumballs on the left plus the
We can rearrange the gumballs in each figure to make a gumballs on the right. However, this counts the middle
square, as shown in the diagram below. gumball twice, so we must subtract 1 at the end.
So, the number of gumballs in the 10" figure of the
e _@ e030 \..
ace 2006 | eceee pattern is 2 times the 10" triangular number, minus 1.
@000e ©00000 This gives 2-4214—4 = 10+11-1=110-1 = 109.
SS S6696:
. We begin by finding an expression for the number of
Figure 1 Figure 2 Figure ? Figure 4
gumballs in the k* figure of the pattern.
The 1* figure has 12=1 gumball. Each figure in the pattern forms a rectangular grid of
The 2" figure has 2? = 4 gumballs. gumballs. We can summarize the dimensions of each
The 3%figure has 3? = 9 gumballs. rectangular grid in the following table.
The 4" figure has 4? = 16 gumballs.
Figure # 1 2 3 4
Continuing this pattern, the 30" figure has 30? = 900
gumballs. Dimensions | 1 by2 | 2by3 | 3by4 |] 4by5
You may recall from the Perfect Squares chapter of Following this pattern, the k'" figure in the sequence
Beast Academy 3B that the sum of the first n odd forms a k-by-(k+1) grid, giving k(k+1) total gumballs.
numbers is equal to n?. This is why the number of We want to know when k(k+1) = 420. Since kis a
gumballs in each figure is always a perfect square. positive integer, we look for two consecutive positive
integers, k and k+1, whose product is 420.
1 +30 +5 +7. +9
la. VaVaVaVa.\
a \
420 is close to 400 = 202, so k= 20 is a reasonable
Opty CAP
Oe16 yu2Byas.:
guess. If k= 20, then k(k+1) =20(21) = 420. W

So, k= 20. The 20" figure uses 420 gumballs.

Beast Academy Practice 5C Sequences


Chapter
7Solutions
[131]
134. The number of black gumballs and the number of gray this pattern will continue forever.
gumballs in each figure are always perfect squares. It is So, the terms with a position number of 3, 6, 9, 12,
easier to see why this is true if we rotate each figure 45 15, and so on, are all even. These are the terms
degrees, as shown below. whose position number is a multiple of 3. All of the

@.0 ©,0,° 6.6 ©@


orerete
other terms are odd.

@ ®
ce @.0.e0
eee [email protected]
&e’e'e
Since 100 is not a multiple of 3, the 100" term is odd.

eeceo 138. After the first two terms, each term in Grogg’s sequence
is the sum of the two previous terms. So, if we call the
Figure1 Figure2 Figure3 Figure4 second term of Grogg’s sequence a, then the third term
e Figure 1 has a “1-by-1 square” of black gumballs, for a in his sequence is 18+a.
total of 12= 1 gumball.
18 ONG, 10; ax
e Figure 2 has a 2-by-2 square of gray gumballs and a
The fourth term, 76, is the sum of the two previous terms.
“4-by-1 square” of black gumballs, for a total of
This gives the equation
2°+12=5 gumballs.
a+(18+a)=76.
e Figure 3 has a 3-by-3 square of black gumballs and a
2-by-2 square of gray gumballs, for a total of Combining like terms, we have
3?+2? = 13 gumballs. 2a+18=76.
e Figure 4 has a 4-by-4 square of gray gumballs and a Subtracting 18 from both sides of the equation gives
3-by-3 square of black gumballs, for a total of 2a = 58. Dividing both sides by 2, we have a= 29. So, the
4?+3?=25 gumballs. second term in Grogg’s sequence is 29.
Continuing this pattern, the n" figure has an n-by-n Check: If the first term is 18 and the second term is 29,
square of gray gumballs, and an (n-1)-by-(”-1) then the third term is 18+29 = 47 and the fourth term is
square of black gumballs, for a total of m?+(n-1)? 47+29=76.
gumballs. 18, 29, 47,76 VW
Note: There are several equivalent ways to write this
expression. If you solved this problem a different way, 139. To get to the 3%step, Hoppy must take one of the
you may have gotten 2n?-2n+1 or 2(n?-n)+1. following paths:
¢ Three 1-step hops from the base.
& 14+1+1=3
135. The first five terms of the Fibonacci sequence are given.
eA1-step
hop,
then
a2-step
hop.
roRhige
To find the 6" term, we add the 4" and 5" terms: 3+5=8.
14+2=3 /
¢ A 2-step hop, then a 1-step hop. ed ie
2+1=3 @
NO
MMANG
DsGer yp ee
The 7* term is the sum of the 5" and 6" terms: 5+8 = 13. There is no other way to reach the 3% step. So, Hoppy can
The 8" term is the sum of the 6" and 7" terms: 8+13=21. go from the base of the steps to the 3" step in 3 ways.
The 9" term is the sum of the 7" and 8" terms: 13+21 = 34. 140. Counting the ways that Hoppy can reach the 4" step can
The 10 term is the sum of the 8" and 9" terms: 21+34=55.
get complicated, so we organize our work as shown:
1, 1, 2, 3, 5, 8, 13, 21, 34, 55
e Four 1-step hops from the base.
136. The 20" term of the Fibonacci sequence is the sum of 1414+14+1=4.
the 18" term and the 19" term. The 19" term is 4,181
e Two 1-step hops, then a 2-step hop.
and the 20" term is 6,765. If we call the 18" term n, then
1+1+2=4.
we have the equation 1+4,181 =6,765.
¢ One 1-step hop, then a 2-step hop, then a 1-step hop.
So, n=6,765-4,181 =2,584. The 18" term is 2,584.
14+2+1=4.
137. We list the first few terms of the Fibonacci sequence and
¢ One 2-step hop, then two 1-step hops.
consider whether they are even or odd.
2+14+1=4.
ri ere AoneHOHTS, MOA... ¢ Two 2-step hops.
odd odd even odd odd even odd odd even
2+2=4.
We see that the numbers follow the repeating pattern of There is no other way to reach the 4" step. So, Hoppy
(odd, odd, even). Since can go from the base to the 4" step in 5 ways.
odd+odd = even,
odd+even = odd, and
even+odd = odd,

[132Sequences
Chapter
7Solutions Beast Academy Practice 5C
141. Hoppy can only hop directly to the 5" step with a 1-step There is only 1 way to make a 2-by-1 rectangle:
hop from the 4" step or a 2-step hop from the 3” step:

ae
ols ye -- Se
|
There are 2 ways to make a 2-by-2 rectangle:
1
Base Base

For each of the ways that Hoppy can reach the 4" step,
Ll a
There are 3 ways to make a 2-by-3 rectangle:
there is exactly one way he can hop straight to the 5"
step from there.
For each of the ways that Hoppy can reach the 3" step,
there is exactly one way he can hop straight to the 5%"
Pols.esteste
As the counting gets more difficult, we look for a way to
step from there. count all of the arrangements using our previous work.
Therefore, the number of ways Hoppy can get to the 5" To make a 2-by-4 arrangement, we can add a vertical
step is the sum of the number of ways he can get to the domino to any of our 2-by-3 rectangles:
4" step and the number of ways he can get to the 3" step.
As we learned in the previous problems, Hoppy can get
to the 4" step in 5 ways and to the 3" step in 3 ways.
So, he can get to the 5" step in 5+3 =8 ways. This counts all of the arrangements that have a vertical
domino on the right. However, we’ve missed all of the
The following sums represent the 8 paths Hoppy could
arrangements that have a pair of horizontal dominos on
take to get to the 5” step.
the right. To get the arrangements that end with a pair of
From the 4" step, we have horizontal dominos, we can add two horizontal dominos
(1+1+1+1+1), to the end of either 2-by-2 rectangle:
(14+1+2+1), (1424141), (2+1+1+1), (2+2+1).
From the 37 step, we have
(14+1+142), (14+2+2), (2+1+2).
Every 2-by-4 arrangement is either a 2-by-3 arrangement
142. We use our answers from the previous three problems to
with a vertical domino added at the end, or a 2-by-2
fill in the first three blanks.
arrangement with two horizontal dominos added at the
1anf ais BS. B5 ’ , ysee end. We have counted every arrangement exactly once.
1st ond 34 4th 5th 6th 7h gth gth 1gth So, there are 3+2=5 ways to make a 2-by-4 rectangle.
To find the number of ways Hoppy can reach the 6" step, Similarly, every 2-by-5 arrangement of dominos is either
we use the strategy from the previous problem. Hoppy a 2-by-4 arrangement with a vertical domino added at
can only hop directly to the 6" step by making a 1-step the end, or a 2-by-3 arrangement with two horizontal
hop from the 5" step, or by making a 2-step hop from the dominos added at the end:
4" step. So, the number of ways Hoppy can reach the
6" step is the sum of the number of ways he can reach
the 5" step and the number of ways he can reach the 4"
step. So, the number of 2-by-5 rectangles is the sum of the
numbers of 2-by-4 rectangles and 2-by-3 rectangles.
Hoppy can reach the 5" step in 8 ways and the 4" step in
There are 5+3=8 ways to make a 2-by-5 rectangle.
5 ways. So, he can reach the 6" step in 8+5 = 13 ways.
The number of 2-by-6 rectangles is the sum of the
numbers of 2-by-5 rectangles and 2-by-4 rectangles.
There are 8+5 = 13 ways to make a 2-by-6 rectangle.
This same pattern continues for each next step. The The pattern so far is 1, 2, 3, 5, 8, 13. These are the
number of ways Hoppy can reach Step 7 is the sum of Fibonacci numbers! Each term is the sum of the two
the number of ways he can reach Step n-1 and the previous terms.
number of ways he can reach Step n-2. This is the
The number of 2-by-7 arrangements is 8+13=21.
same rule that is used by the Fibonacci sequence!
The number of 2-by-8 arrangements is 13+21 = 34.
We continue the pattern to fill in the remaining blanks. The number of 2-by-9 arrangements is 21+34=55.
16 2"? BS Be 8. 13.21, 34,90, 89; -.. The number of 2-by-10 arrangements is 34+55 = 89.
qe Bre”
Be’Gr’BR”
ge”TR"BR“ghTOF So, there are 89 arrangements of dominos that form a
2-by-10 rectangle.
143. Counting the number of 2-by-10 arrangements of
dominos is tough. So, we first consider smaller
rectangles.

Beast Academy Practice 5C Sequences


Chapter
7Solutions
| 133]
We have already seen the term 169. So, we know the
next term is 256.
144. The first two terms of the sequence are 8 and 1, and
65, 121, 16, 49, 169, 256, 169, 256
each next term is half the product of the two previous
terms. So, the 3" term is Eas
a2 2 148. The first term is 65. The squares of the digits of 65 are

8, ie 4, oe) ee) eee) eee) 6? = 36 and 5?=25. So, the next term is 36+25=61.

Then, the 4" term is Utsda2.

8, 1, 4, 2 > The squares of the digits of 61 are 6? = 36 and 1?=1. So,


ee — —
the next term is 36+ 1 =37.
We continue this pattern to fill in the remaining blanks as
shown. C5," O15 "G7,2 ees
8, 1, 4, 2, 4, 4, 8, 16
145. The next term is 32+ 72=9+49 = 58.
Each term is the opposite of twice the previous term. So,
the term after 5 is -(2*5) =-10. 65, 61, 37, 58,
iyi) Hhthar le ine i il
The next term is 52+8? = 25+64 = 89.
Then, the term after -10 is -(2+(-10)) =-(-20) = 20.
5, -10, 20 65, 61, 37, 58, 89,__,
_ ) )

We continue this pattern to fill in the remaining blanks as The next term is 8?+9? = 64+81 =145.
shown.
5, -10, 20, -40, 80, -160, 320 65, 61, 37, 58, 89,145,
146. The first two terms are 0 and 1. So, the next term is the The nextterm is 12+4?+5?=1+16+25 =42.
reciprocal of 0+ 1 =1. The reciprocal of 1 is + =1.
65, 61, 37, 58, 89,145, 42,
Osetia Dd)

The next term is 42+2? = 16+4=20.


The next term is the reciprocal of 1+1=2, which is 5.
1 65, 61, 37, 58, 89,145, 42, 20
0, Uy 1, 2——
If we continue this sequence, does it ever repeat?
Thenexttermisthereciprocal
of1+4=3,which
isé. 149. The first two terms of the sequence are 0 and 1. Since
eee each term is the sum of all previous terms, the 3% term is
URE sung yt 5 eae
0+1=1.
The next term is the reciprocal of $+e = Z, which is £.
The 4" term is 0+1+1 =2.
1 26 The 5" term is 0+14+1+2=4.
The 6" term is 0+1+1+2+4=8.
6 _ 32
Thenexttermisthereciprocal
ofo+ 1p Par which
iss The 7" term is 0+1+1+2+4+8= 16.

Ai 22:6, OF After the first two terms, each term in the sequence is a
0, 1, 1, 2 3° 7 32
power of 2:
147. 4st Ond 30 4th 5th 6th 7h Bi
The first term is 65. The sum of the digits of 65 is
6+5=11, and 112=121. So, the next term is 121. OSS hy Se ARBs, 1G...

Useented aris yrs eee cee


The sum of the digits of 121 is 1+2+1=4. So, the next The exponent in the power of 2 is three less than the
term is 42= 16. term’s position number in the sequence. For example,
the 5" term is 25-3 = 22.
65; 21. 16) 5 Ue OR Fea ae
So, the 7" term in the sequence is 2~*) for values of n
The next term is (1+6)? = 7?=49. that are greater than 2 (we make this distinction since
2"-®) works for all terms except the 1%and 2").
651210216 0AOes, ote iid any
The nexttermis (4+9)?= 13?= 169. 150. a. To find the term that comes after 32, we reverse the
digits of 32 and add 1 to the result: 23+1=24.
65,;-121,.16,49) 169455.,.2..,
Then, the next term is 42+1 = 43.
The nexttermis (1+6+9)?=16?=256. Then, the next term is 34+1 = 35.

65, 121,516, 49).169, 256, We continue this pattern to complete the sequence as
The next term is (2+5+6)? = 137 = 169. shown.

65, 121, 16, 49, 169, 256,169, 32, 24, 43, 35, 54, 46, 65, 57, 76, 68

Sequences Chapter 7 Solutions Beast Academy Practice 5C


b. We work backwards. The final term is 100. So, 153. There are many correct answers to this problem. The key
reversing the previous term’s digits and adding 1 to is working backwards! We begin by writing two numbers
the result gives 100. Therefore, the reverse of the for the last two terms, where the next-to-last term is
previous term’s digits is 100-1 =99. Since the reverse greater than the last term. For example, suppose the
of 99’s digits is 99, the term before 100 is 99. last two terms are 2 and 1. We let n represent the term
before 2.

—) ——_)
—) ——)
—_)—__)Nn,Hap1
To find the term before 99, we subtract 99-1 = 98,
then reverse the digits to get 89. There are two possible values for n:
—F. FF ee) ey ey ee 89, 99, 100 * If nis less than 2, then 2-n=1, giving n=1.
¢ If nis greater than 2, then n-2=1, givingn=3.
To find the term before 89, we subtract 89-1 =88,
then reverse the digits to get 88. Since our terms decrease from left to right, 7 must be
greater than 2. So, we compute 7 by adding 1+2=3.
_———
} 2 +— 88, 89, 99, 100

We continue working from right to left to complete the


sequence as shown below. Working from right to left, we continue adding the two
55, 56, 66, 67, 77, 78, 88, 89, 99, 100 largest terms to get the next term. For example, the term
to the left of 3 is 2+3=5, followed by 3+5=8, and so on.
151. @. The term that comes after 85 and 52 is 85-52 = 33. Since the terms increase from right to left, they decrease
from left to right, as shown below.
The term after 52 and 33 is 52-33 = 19.
The term after 33 and 19 is 33-19= 14. 55, 34, 21, 13, 8,5, 3, 2, 1
The term after 19 and 14 is 19-14=5.
Notice that these are the Fibonacci numbers in reverse!
The term after 14 and 5 is 14-5=9.
You can use numbers other than 2 and 1 to end your
The term after 5 and 9 is 9-5 =4.
sequence to get a different answer. You can check your
The term after 9 and 4 is 9-4=5.
work by adding each pair of numbers from right to left!
The term after 4 and 5 is 5-4=1.
154. The sum of the digits of a two-digit number is at least
The completed sequence is shown below.
1+0=1 and at most 9+9= 18. So, the 2™term in Ralph’s
85, 52, 33, 19, 14, 5, 9, 4, 5, 1 sequence is the product of 9and some number between
1 and 18, which gives the following possibilities for the
b. The term that comes after 76 and 47 is 76-47 =29.
2" term:
The term after 47 and 29 is 47-29= 18.
The term after 29 and 18 is 29-18=11. 9, 18, 27, ..., 144, 153, 162.
The term after 18 and 11 is 18-11 =7.
For any number divisible by 9, the sum of that number’s
The term after 11 and 7 is 11-7 =4.
digits is also divisible by 9. Therefore, since the 2™ term
The term after 7 and 4 is 7-4=3.
is a multiple of 9, the sum of the digits of the 2"? term is
The term after 4 and 3 is 4-3=1.
also a multiple of 9.
The term after 3 and 1 is 3-1 =2.
Also, since the 2" term has three or fewer digits, and the
The completed sequence is shown below.
2"¢ term cannot be 999, the sum of the digits of the 2"
76, 47, 29, 18, 11, 7, 4, 3, 1, 2 term is less than 9+9+9 = 27. So, the sum of the digits of
the 2™ term is 9 or 18.
152. The term after 5 and 3 is 5-3 =2, giving 5, 3, 2, ....
Therefore, the 3 term in Ralph’s sequence is 9°9=81
Next, we have 3-2=1, giving 5, 3, 2, 1,.... or 9°18 = 162.
Next, we have 2-1 =1, giving 5, 3,2,1,1,....
The sum of the digits of both 81 and 162 is 9. So, the 4"
Next, we have 1-1 =0, giving 5, 3, 2,1, 1,0
term must be 9°9 =81. Therefore, there is only 1 number
Next, we have 1-0=1, giving 5, 3, 2, 1, 1,0, 1,....
that could be the 4" term in Ralph’s sequence.
Next, we have 1-0=1, giving 5, 3, 2,1,1,0,1,1,....
Next, we have 1-1=0, giving 5, 3, 2,1, 1,0,1,1,0,....
Beginning with the 4" term, we have a repeating cycle of
the terms 1, 1, 0. 155. We look for a pattern in the units digit of the first few
The terms equal to 0 are the 6", 9", 12", 15", and so on. powers of 3. Since we are only concerned with the
These are the terms whose position number is a multiple units digit, we can ignore the other digits in each power.
of 3, starting with the 6" term. (Review units digits in the Multiplication chapter of Beast
Academy 4A.)
Since 75 is a multiple of 3, the 75" term is 0.

Beast Academy Practice 5C


Sequences
Chapter
7Solutions
| 135
|
3'=3. So, to find the column 99 is under, we can continually
3? =9. subtract 12 from 99 until we arrive at a number whose
3° = 27. column we know. Since 99-12(8) = 99-96 = 3, we know
3* = 3°-3 has the same units digit as 7*3 = 21. 99 is under the same column as 3, which is column C.
3° = 34-3 has the same units digit as 1°3=3.
3° = 35-3 has the same units digit as 3°3=9.
The pattern of moving right, then down, then left, then
3’ = 3°-3 has the same units digit as 9°3=2
down repeats every 12 integers. So, every multiple of 12
3° = 3’7+3 has the same units digit as 7-3 = 21.
is in the same column and occurs at the same point in the
The units digits follow the repeating pattern cycle. So, 12°8 =96 is in column A, and the path moves
po c/a io hake
Bar(eeIne to the right after passing through 96.

These digits repeat every 4 terms. The terms in this We continue the path from 96 until we reach 99, which is
pattern equal to 1 are the 4", 8, 12", 16", and so on. in column C.

These are the terms whose position number is a multiple A BC™D- ‘En -F

of 4. Since 96 = 4(24), the 96" term in this pattern is 1.


Therefore, 3° has units digit 1. Continuing the pattern,
3°” has units digit 3, 3° has units digit 9, and 3% has units
digit 7.
156. Rather than computing the entire sum, we consider the
sums of the first few terms and look for a pattern.
The sum of the first 2 terms is 1+2=3.
The sum of the first 3 terms is 1+2+4=7.
158. This spiraling pattern forms a roughly square-shaped
The sum of the first 4 terms is 1+2+4+8= 15.
figure. This suggests that thinking in terms of squares
The sum of the first 5 terms is 1+2+4+8+16=31.
might be useful. We consider the various square pieces
We notice that each result is one less than the next of the pattern, as shown below.
power of 2 in the sum! For example, the sum of the first 21 22-23-24=25
5 terms is 31, which is one less than the next term in the 7-8-9 7-8-9-10 20 7-89-10
sum, 2°= 32.
4_ mee ea, ek ie i ee
14+2+4+8+16+32+--
Ne! 45 &GY WIGSPee te egos 12
31
16-15-1413
17-16-15-14-1
Continuing this pattern, the sum of the first 6 terms is The last number in each figure above is a perfect square.
31+32 =63, which is one less than the next term in the
e 2°=4 is at the lower-left corner of the 2-by-2 square.
sum, 2° = 64.
¢ 3°=Q is at the upper-right corner of the 3-by-3 square.
So, the sum of all of the terms is one less than the power
of two after 2'?= 4,096. The power of two after 4,096 is e 42= 16 is at the lower-left corner of the 4-by-4 square.

213 =2°4,096 =8,192. So, the sum of all the terms in the e 5?= 25 is at the upper-right corner of the 5-by-5 square.
sum is 8,192-1 =8,191.
Each square of an even integer is in a lower-left corner.
In general, the sum of the firstn powers of 2 (starting Each square of an odd integer is in an upper-right corner.
with 2° = 1) is 2”-1.
Since 100 = 10, the number 100 will be in the lower-left
157. In this pattern we move through the positive integers, corner of a 10-by-10 square. To determine what number
starting with 1 in the top-left corner and following the path will be directly above 100, we look at a known example.
as shown.
21-22-23-24--
20 7—8—9—-10
ue : ee i
8 5-4-3 ve
17-46-15-14-13
Here, we see that the number directly above 4? = 16 is
the number to the left of the previous square of an even
integer, 2?=4.
This pattern of moving right, then down, then left, then
down repeats every 12 integers. So, we can add 12 to Similarly, the number directly above 10? = 100 is the
any number and end up in the same column we started in. number to the left of the previous square of an even
For example, 1 is in column A, and 1+12= 13 is also in
integer, 8? = 64.
column A. Similarly, 8 is in column E, and 8+12=20 is The number to the left of 64 is 65. So, the number
also in column E. directly above 100 is 65.

Sequences Chapter 7 Solutions Beast Academy Practice 5C


159. We compute the remainder when dividing the first few 161. We consider how much liquid is in each juice box after
powers of 2 by 7 and look for a pattern. the first several pours. We'll call the first juice box Box A
The remainder of 2'+7 =2+7 is 2. and the second juice box Box B.
The remainder of 2?+7 = 4+7 is 4.
ist Pour
The remainder of 2°+7 =8+7 is 1.
Grogg pours half of the 2 cups of
The remainder of 24+7 = 16=7 is 2.
juice in Box A into Box B. Box A BoxB
The remainder of 25+7 = 32+7 is 4,
Half of 2is$-2=1.
The remainder of 2°+7 =64+7 is 1.

We see a pattern! Since each power of 2 is 2 times So, BoxA is leftwith2-1=1 cup, 1 Pour
the power that came before it, each remainder is the and BoxB is leftwith0+1=1 cup.
remainder of

(2 times the remainder that came before) +7. 2nd Pour


Grogg pours one third of the 1 cup
So, we can be sure that this pattern continues. (Review
of juice in Box B into Box A.
this remainder concept in the Division chapter of Beast
Academy 3C.) Onethirdof1 is$-1 =4.
Box A BoxB
So, the remainders follow the repeating pattern So,BoxB isleftwith 1-+=2cups,
2 Bah rend Alcea,aad ries
andBoxA is leftwith 1+4=4
1 cups. __
istpour
The terms equal to 1 are the 3, 6, 9'", 12! and so on. 3rd Pour
2" pour
These are the terms whose position number is a multiple Grogg pours one fourth of the 4
of 3. Since 99 = 3(33), the 99" term in this pattern is 1. cups of juice in Box A into Box B.
3° pour
So, the remainder of 2°°+7 is 1. Continuing the pattern, Onefourthof4 is¢-4=43°
the remainder of 2'°+7 is 2.
So,BoxA is leftwith4-4=1 cup,
160. The compubot displays the number 3. Since 3 is odd, andBoxB is leftwith£+4=1 cup.
pressing the * button gives the number 3(3)+1 = 10.
4th Pour
10 is even, so pressing * again gives 10+2=5.
Grogg pours one fifth of the 1 cup of
5 is odd, so pressing * again gives 3(5)+1 = 16. Box A BoxB
juice in Box B into Box A.
16 is even, So pressing * again gives 16+2=8.
8 is even, So pressing * again gives 8+2=4. Onefifthof 1is$-1 =H.
5 5 1s pou
4 is even, so pressing * again gives 4+2=2. So, Box Bis left with 1-4=4 cups,
2 is even, so pressing * again gives 2+2=1. andBoxA isleftwith 1+2=£cups. 2 po.
1 is odd, so pressing * again gives 3(1)+1 =4.
We have already seen the number 4, so we know the Sth Pour 3°pour
next number will be 2, followed by 1, then back to 4, and Grogg pours one sixth of the 2 cups
so on forever. of juice in Box A into Box B. 4" pour

One .
sixth of ¢(s Jaren (oye |
is G°==5- 5 pour
We can summarize our findings with the chart below.
Starting number: 3 So, Box A .is left with
. 3Gig!
e aeShecup:

smPresses MlShee"|i 32) 4h (SGN fF-7areer <9") 10 andBoxB is leftwith4+4=1 cup.


Result} 10} 5 |16/ 8/4 ]2)]1]41]24]1 After the first pour, each pair of consecutive pours shown
above moved the same amount of juice out of and then
Starting with the 5" time the * is pressed, the results
back into Box B. This left both boxes with 1 cup of juice
repeat every 3 presses. So, 2 is displayed after the 6"
after the odd-numbered pours. We explore whether or not
press, 9" press, 12" press, 15" press, and so on. In
this pattern will continue.
general, if the number of times the * has been pressed
is a multiple of 3 (and greater than 5), then the number When both boxes have 1 cup of juice, Grogg’s next pour
displayed will be 2. moves some unit fraction of Box B’s juice into Box A.
Since 75 = 3(25), the number displayed after the 75" Ifwecallthatunitfraction
L,thenGrogg
pours
aa =t
press is 2. cupsfrom BoxB intoBoxA.
This question is related to a very famous problem. Many This leaves Box A with 1 +4 cups of juice. Since 1 = . we
mathematicians suspect that no matter what positive
can rewrite 1 +4 as shown below:
integer you start with, you will eventually reach the
number 1 (in this case, we started with 3 and reached (elon bowe
1 after the 7" press). This is known as the Collatz nen
Conjecture, and mathematicians are still trying to prove So, Box A has “et cups of juice.
that it is true!

Beast Academy Practice 5C


Sequences Chapter 7 Solutions
Grogg’s next pour moves a unit fraction of BoxA BoxB 166. The 7 terms in the sequence have sum 133, so their
Box A’s juice into Box B. The denominator average is 133+7 = 19. In an arithmetic sequence, the
of this unit fraction is one greater than on average of the terms is equal to the median. So, the

the previous pour. So, Grogg pours + of median is 19.


the a cups in Box A into Box B, which is —) —) ——!19 _—_—_)———)
——

BP Ia) We also know that the first and last terms in an arithmetic
rH cae sequence balance around the median. So, the first and
This is the same amount that Grogg moved out of Box B
last terms balance around 19.
from the previous pour. So, after both pours, Box B has
the same amount of juice it started with: 1 cup. To make the last term as great as possible, we make
the first term as small as possible. Since every term is a
So, the amount of juice in each box is always 1 cup after
positive integer, the smallest possible value of the first
each odd-numbered pour. Therefore, there is 1 cup of
term is 1.
juice in the first box after Grogg’s 99" pour.
Then, since 1 is 18 less than the median, the last term
162. Since the common difference is 8, we can continually add must be 18 more than the median: 19+18 = 37.
8 to -999 until we get a positive result. 8(125) = 1,000, so
-999+8(125) = -999+ 1,000 = 1 is a term in the sequence.
1 19 37
There is no smaller positive integer than 1, so the >) ———) a) ———)

smallest positive term that appears in the sequence is 1. We are told that each term is a positive integer, so we
must check that there are no non-integer terms in this
163. This is the sequence of positive integers written in base-2.
sequence. The 1* term is 1 and the 4" term is 19, so the
(Review base-2 numbers in the Exponents chapter of common difference is ot = 2 =6. Since the common
Beast Academy 4A.)
difference is an integer, and the first term is an integer,
Alternatively, we can think of it as the sequence of each term in the sequence is also an integer.
positive integers that can be written using only the digits
So, the greatest possible term that can appear in the
0 and 1, ordered from least to greatest.
sequence is 37.
After 111,the next-smallest number is 1000, followed by
We could have also reversed the order of the terms in
1001, then 1010, then 1011, then 1100.
this sequence to arrive at the same final answer.
1, 10, 11, 100, 101, 110, 111, 1000, 1001, 1010, 1011, 1100
167. Since each term is the sum of the three previous terms,
164. In the list of positive integers where the perfect squares we know that x+y+z=44.
are not removed, the 50" term is 50.
We also know that y+z+44=81. We can subtract 44
There are 7 perfect squares less than 50:
from both sides of this equation to get y+z=37.
1, 4, 9, 16, 25, 36, and 49.
In the equation x+y+z=44, we see the expression y+z.
So, in the sequence where the perfect squares are We know from our work above that y+z is 37, so we
removed, all 7 of these terms have been taken out. So, replace y+z with 37 in this equation.
50 appears in position number 50-7 = 43. We count up
X+y+zZ=44,
to the 50" term from here.
xX+37 =44.
Position: 43° 44% 45% 46" 47% 48th 49h 50% Now we have an equation with just one variable! Solving,
erm: 5055515 52; 53) 754,> 65;"'56)°57 we have x=7.
There are no perfect squares from 50 to 57 to remove, As an extra challenge, can you determine possible
so the 50" term is 57. values for y and z? Are these the only possible values?
165. Since we begin with 0, and adding 1 then 2 then 3 is the 168. The 1* figure uses 3 toothpicks.
same as adding 6, we know that every multiple of 6 is a The 2™figure uses 3+6 = 9 toothpicks.
term in this sequence. The 3%figure uses 3+6+9 = 18 toothpicks.
The smallest 7-digit number is 1,000,000. We look for a
multiple of 6 that is close to 1,000,000. Since 1,000,002 LS Pas
is even and the sum of its digits is divisible by 3, it is
divisible by both 2 and by 3, and is therefore a multiple of
2.arcoelarwhdvorw
<tcNfa
6. So, 1,000,002 is a term in the sequence.

Each multiple of 6 in the sequence is reached after


Continuing the pattern, the n" figure uses 3+6+9+---+3n
adding 3 to the previous term. So, the term that came
toothpicks. We can factor a 3 out of each term to get
before 1,000,002 is 1,000,002-3 = 999,999. However,
999,999 is a 6-digit number. 3+64+9+4---+3n
= 3(1+24+3+-+n).
So, the smallest 7-digit number that appears in the Welearnedpreviouslythat 1+2+3+--+n= Hart) So,
sequence is 1,000,002.
3(1+2+3+--+n)=3-2e),
Sequences Chapter 7 Solutions Beast Academy Practice 5C
So, the number of toothpicks needed to make the 20"
figure in the pattern is 3-208) =3°210=630.

It takes 3 toothpicks to make each small triangle.


However, when triangles in a figure touch, some or all
of their toothpicks are shared by another triangle. This
makes counting the number of toothpicks based on the
number of triangles difficult.
However, if we count just the triangles “pointing up,”
we count every toothpick in the figure, and there are no
shared toothpicks. In the figure below, the “up” triangles
are shown with a dot in the center.

/\
/\/\
/\/\/
There are 6 “up” triangles, each with 3 toothpicks, for a
total of 6-3 = 18 toothpicks.
So, we want to know the number of “up” triangles that are
in the 20" figure in this pattern. Looking at the first three
figures, we have 1 “up” triangle, then 3, then 6.

rd NLA. /\
[\/\ /\/\
L\/\/
The “up” triangles make a pattern of triangular numbers!
So, the number of “up” triangles in the 20" figure is the
20" triangular number: eo-et =10°21=210. Each “up”
triangle uses 3 toothpicks, so the number of toothpicks in
the 20" figure of this pattern is 3-210 = 630.
169. Bronkle writes an arithmetic sequence with first term 40
and common difference 6. So, the k* term of Bronkle’s
sequence is 40+(k-1)6 = 40+6k-6 =6k+34.

Gergum writes an arithmetic sequence with first term -50


and common difference 9. So, the k term of Gergum’s
sequence is -50+(k-1)9 =-50+9k-9 =9k-59.

We want to know when the k'" term of Bronkle’s


sequence is equal to the k**term of Gergum’s sequence.
So, we write an equation:
6k+34 =9k-59.
To isolate k, we begin by subtracting 6k from both sides
of the equation. This gives
34 = 3k-59.

Then, adding 59 to both sides gives 93 = 3k. Dividing


both sides by 3, we have 31 =k.
So, the 31% terms of Bronkle’s and Gergum’s sequences
are equal. We can replace k with 31 in either 6k+34 or
9k-59 to find the value of the 31*' term:
6k+34 = 6(31) +34 = 186+34 = 220.
9k-59 = 9(31)-59 = 279-59 = 220.
So, k=31, and the 31* term of each sequence is 220.

Beast Academy Practice 5C Sequences Chapter 7 Solutions


Sequences Chapter 7 Solutions mundeininscmimminin a2
18. There were 56 flips that landed heads, so 100-56 = 44
of the flips landed tails. The ratio of heads flipped to tails
1. There are 3 black circles and 7 white circles, so the ratio flipped is 56:44 = 14:11.
of black circles to white circles is 3 to 7. 19. We can divide the smaller square into 3-3 =9 little
2. There are 10 circles and 7 triangles, so the ratio of circles squares, as shown below. Since the ratio of the side

to triangles is 10 to 7. length of the smaller square to the side length of the


larger square is 3:5, we can place 5 of the same little
3. There are 5 black shapes and 12 white shapes, so the
squares along each side of the larger square. So, we can
ratio of black shapes to white shapes is 5 to 12.
divide the larger square into 5-5 = 25 little squares.
4. There are 5 white triangles and 2 black triangles, so the
ratio of white triangles to black triangles is 5 to 2.
5. We can split these 12 circles into groups of 3 circles, a
then shade 2 circles in each group. One example is
Therefore, the ratio of the area of the smaller square to
shown below.
the area of the larger square is 9:25.

20.
ig ratioofcupsofyogurt
tocupsofstrawberries
is
<.4
>:Toannebi asa ratiooftwointegers,
we
multiply
32and+>bytheleastcommen multiple
oftheir
All together, we must shade 8 circles. denominators,
3.2=6.Thisgives>a4: 3.
6. We divide 5 and 10 by their greatest common factor, 5, to
get 5:10 = 1:2. The greatest common factor of 1 and 2 is
1, so 1:2 is the simplest form of this ratio.
21. We can split the dancers into groups, each with 5
7. We divide 197 and 197 by their greatest common factor, dragons and 3 yetis. Since there are 30 dragons, we can
197, to get 197:197 =1:1. make 30+5=6 groups with 5 dragons in each group.
8. The greatest common factor of 25 and 9 is 1, so 25:9 is Then, each of the 6 groups also has 3 yetis. So, there
the simplest form of this ratio. are 6-3=18 yetis in the class.
9. We divide 40 and 16 by their greatest common factor, 8, 22. We can split fans at the tournament into groups, each
to get 40:16 = 5:2. with 5 Beast Academy fans and 4 Orb Academy fans.
Since there are 160 Beast Academy fans, we can make
10. We divide 42 and 70 by their greatest common factor,
160+5 = 32 groups with 5 Beast Academy fans in each
14, to get 42:70 = 3:5.
group. Each of the 32 groups also has 4 Orb Academy
11. We divide 121 and 88 by their greatest common factor, fans. So, there are 32-4 = 128 Orb Academy fans.
11, to get 121:88 = 11:8.
23. We can split the sodas sold into groups, each with 7
12. We divide 24 and 92 by their greatest common factor, 4, grape and 6 orange sodas. Since 84 orange sodas were
to get 24:92 =6:23. sold, we can make 84+6 = 14 groups with 6 orange
13. We divide 91 and 65 by their greatest common factor, sodas in each group. Then, each of the 14 groups also
13, tove 91:65 = a has 7 grape sodas. So, 14:7 =98 grape sodas were
sold.
14, 2n=k, Towrite24aff=f :7as a ratiooftwointegers,
webeginbya ee bothquantities by3toeliminate 24. We can split Priti's answers into groups, each with 2

thefraction.
Then,wehave3:7=7:21.Wedivideeach incorrect answers and 5 correct answers. Since she

of7 and21bytheirgreatestcommon factor,7,toget answered 30 problems correctly, we can make 30+5 =6


groups with 5 correct answers in each group. Then, each
7:21=1is Therefore,
2 17 =1:3.
15. Towrite=i:2 asa ratiosftwointegers,
wemultiply
3+and of the 6 groups also has 2 incorrect answers. So, Priti
answered 6-2 = 12 questions incorrectly, and there were
“ bythefeastee Us multiple oftheirdenominators,
3-5=15.Thisgives3: 2=5:9. a total of 30+ 12 =42 questions on the test.

16. The ratio of blue cars to red cars is 16:10, which


simplifies to 8:5. As in the previous approach, we can make 30+5=6
groups with 5 correct answers in each group. Each of the
17. The class has 26 students with brown eyes and
6 groups contains a total of 2+5=7 answers. So, there
30-26 =4 students with green eyes. So, the ratio
of brown-eyed students to green-eyed students is were 6-7 = 42 questions on the test.

26:4 = 13:2.

Beast Academy Practice 5C Ratios &Rates Chapter 8 Solutions [141]


25. We can split Roland's cookies into groups, each with For every 4 equal groups of players wearing white, there
4 sugar and 3 oatmeal-raisin. Since Roland baked are 5 equal groups of players wearing blue.
36 oatmeal-raisin cookies, we can make 36+3= 12
We can split 27 players into 4+5 =9 equal groups, each
groups with 3 oatmeal-raisin cookies each. Then, each
with 27+9 =3 players.
of the 12 groups also has 4 sugar cookies. So, Roland
baked 12-4 =48 sugar cookies, which makes a total of So, 3-4=12 players are wearing white jerseys and
36+48 = 84 cookies. 3-5=15 are wearing blue.

As in the previous approach, we can make 36+3 = 12 2 of the jerseys are white, and 2 are blue. So, ata
groups with 3 oatmeal-raisin cookies each. Since each of game with 27 players, 4.97=12 are wearing white and
the groups contains 3+4 =7 cookies total, Roland baked 3.27 = 15 are wearing blue.
7:12 =84 cookies all together.
31. For every 3 defenders, there are 5 strikers. We can split
26. We can split Allison's green paint mix into 5 equal parts: 96 players into 96+8 = 12 teams of 8 players, each with 3
2 parts blue paint and 3 parts yellow paint. Since the 3 defenders and 5 strikers.
ane - yellow paint equal = of a pint, each part is
So, 12°3=36 of the players are defenders and 12-5=60
a°°= tetet of a pint. Therefore, the 2 parts of green
are strikers.
paint
equal
24 “726 2=tofa pint.
Check: The ratio of blue to yellow paint is at.
Multiplying both numbers by the least common multiple 5oftheplayers
aredefenders,
and2arestrikers.
So,in
a league of 96 players, 3.96 = 36 are defenders and
of their denominators, 12, gives i:ts 2:3. YW 2.96 = 60 are strikers.
27 We can split Peter's miles on a bus or train into 10
32. On every tray of 10 cupcakes, 4 are chocolate and 6
equal parts: 7 parts train-miles and 3 parts bus-miles.
are vanilla. We can split 240 cupcakes into 240+10=24
Since the 3 parts of bus-miles equal 210 miles, each
trays of 10 cupcakes, each with 4 chocolate and 6
part is 210+3=70 miles. So, the 7 parts of train-miles
vanilla. So, 24-4 = 96 of the cupcakes are chocolate and
equal 7-70 =490 miles, and Peter traveled a total of
24-6=144 are vanilla.
210+490 = 700 miles by bus or train.
Then, Peter biked 1 mile for every 14 miles that he rode
the bus or train. Since he traveled 700 miles by bus or 5 =2ofthecupcakes
arechocolate,
and= =2
train and 700+14=50, he biked 50 miles. are vanilla. So, of the 240 cupcakes, 2.240 =96 are
chocolate and 2.240 = 144 are vanilla.
28. For every 3 white beads that Anna uses, she'll use 7
black beads. 33. Since the ratio of stingravens to pandakeets is 2:9,
& of the 176 animals in the exhibit are stingravens.
Since 39 = 3-13 is the largest multiple of 3 that is less
than 40, Anna can make up to 13 groups of 3 white <- 176 = 32, so there are 32 stingravens.
beads each (with one left over). Since the ratio of spotted to striped stingravens is 3:5,
Since 84 =7-12 is the largest multiple of 7 that is less 2 of the 32 stingravens are striped.
than 90, Anna can make up to 12 groups of 7 black 2.32 = 20, so there are 20 striped stingravens in the
beads each (with six left over). exhibit.
So, when making groups of 7 black and 3 white beads, 34. The width-to-height ratio of 5:2 tells us that 2 of the
Anna is limited by the black beads she has. So, she
perimeter comes from the width. So, 2.210 = 150 inches
cannot make more than 12 groups of 7 black and 3 white of the perimeter come from the width, and the remaining
beads. Anna can use at most 3: 12 = 36 white beads.
210-150 = 60 inches come from the height.
The perimeter of a rectangle includes two copies each
of the width and height. So, the rectangle is 150+2=75
inches wide and 60+2 = 30 inches tall. The area of a
29. Each blorble has 3+4=7 horns. So, there are 98+7 = 14
75-by-30-inch rectangle is 75-30 = 2,250 square inches.
blorbles on the field. Fourteen blorbles have 14:3 = 42
long horns and 14-4 =56 short horns.

30. For every 4 players wearing white jerseys, there are 5 For every 5 inches in the width of the rectangle, there
players wearing blue jerseys. We can split 27 players are 2 inches in the height. So, for some value of x, the
into 27+9 =3 groups of 9 players, each group having 4 rectangle is 5x inches wide and 2x inches tall. Then,
players wearing white and 5 players wearing blue. the perimeter of the rectangle is 5x+5x+2x+2x=14x
inches.
So, 3-4=12 players are wearing white jerseys and
3-5=15 are wearing blue. Since 14x = 210, we have x=15.

So, the rectangle is 5-15 =75 inches wide and 2-15 =30
inches tall. The area of a 75-by-30-inch rectangle is
75:30 = 2,250 square inches.

Ratios & Rates Chapter 8 Solutions Beast Academy Practice 5C


39. The only column that is completely filled tells us that the
milk-to-butter ratio is 36:56. This ratio simplifies to 9:14,
35. Simplifyingeach ratio, we get which allows us to fill in the left column:

21:28 =3:4, 63:84 = 3:4, 39:52 = 3:4, [ounces


ofmik
| 9[ar]oe]oe]
24:36 = 2:3,
All but 24:36
51:68 = 3:4.
= 2:3 are equivalent to 3:4. So, 24:36 is the
Tounces
ofbuter]
1 | | se|_| 126)
If we triple the amount of milk, then we must also triple
ratio that is not equivalent to the other four.
the amount of butter to keep the same milk-to-butter
21:28 63:84 39:52 51:68 ratio. Similarly, if we double the amount of milk, we must
36. The ratio 11:17 is already in simplest form. A ratio of also double the amount of butter.
integers whose left-side quantity is not a multiple of 11
cannot be simplified to
11:(some integer).
So, we can eliminate the two ratios crossed out below.
22:34 3BO4& 33:51 44:68
55:85 55:88 Bees 66:102 Similarly, if we multiply the amount of butter by 9, we
must also multiply the amount of milk by 9.
Simplifying the remaining ratios, we have
“9
22:34=11:17, 33:51 =11:17, 44:68=11:17,
55:85=11:17, 55:88=5:8, 66:102 = 11:17.
ounces
ofmikan27|ee
| 8| 66|S4|ot
So, the five ratios circled below are equivalent to 11:17. ‘ounces
ofbuter|
14| 42|56|94|126] ee ee ee A
2:34) 30:48 ke
5:85) 55:88 60:93 G6:102> 40. The original ratio of pigs to goats in the pen is
37 Simplifying each ratio, we get 18:24 = 3:4, so for every 3 pigs, there are 4 goats.

2. 30 =320; 14:24 =7:12, SeAo =F After the new animals enter the pen, there are 18+9 =27
56:96 = 7:12," 55:90= 11:18; °%33:54= 11518; pigs, which we can split into 27+3=9 groups of 3 pigs
12:20=3:5; 19227=230. each. Then, each of the 9 groups also has 4 goats, so
So, the pairs of equivalent ratios are there are 9-4=36 goats. Therefore, 36-24 = 12 goats
entered the pen.
21:35 =12:20, 14:24 = 56:96,
32:48 = 18:27, and 55:90 =33:54.
The ratio of pigs to goats is 18:24 = 3:4. Since the
You may have written these equations in a different
entrance of the new animals did not change the
order.
pig-to-goat ratio, for every 3 pigs that entered the pen, 4
38 Since we want three equivalent ratios, each ratio must goats must have also entered.
have the same simplest form.
Since 9=3-3 pigs entered the pen, 3-4=12 goats must
All six numbers are integers, and only two are multiples have also entered the pen.
of 5. So, neither of the quantities in the simplest form of 41. The ratio of gold to silver coins is 36:63 = 4:7, so Kraken
each ratio is a multiple of 5. Therefore, the two choices has 4 gold coins for every 7 silver coins.
which are multiples of 5 must be part of the same ratio so
After Kraken spends some of the coins, he has
that the factors of 5 cancel when we simplify:
36-8 = 28 gold coins remaining, which we can split into
30:140 = 3:14.
28+4=7 groups of 4 gold coins each. Then, each of the
The remaining numbers are 6, 9, 28, and 42. Of these, 7 groups also has 7 silver coins, so there are
only 28 and 42 are multiples of 14, so the other two ratios 7:7=49 silver coins remaining. Therefore, Kraken spent
are__:28 and __:42. 63-49 = 14 silver coins.
Filling these blanks with the two remaining numbers (6 — or—
and 9), we have 6:28 = 3:14 and 9:42 = 3:14. The ratio of gold to silver coins is 36:63 = 4:7. Since
So, the three equivalent ratios are spending some of the coins did not change the

30:140 = 6:28 = 9:42. gold-to-silver ratio, for every 4 gold coins Kraken spent,
he also spent 7 silver coins.
We could write these ratios in any order within the
equation, or flip the order of the quantities in each ratio. Since Kraken spent 8 = 2-4 gold coins, he spent 2-7=14
If you changed the order of the quantities in one ratio, be silver coins.
sure that you changed the order of the quantities in all
the ratios!
28:6 = 42:9 = 140:30.

Beast Academy Practice 5C Ratios & Rates Chapter 8 Solutions


42. ie original ratio of cups of cornstarch to cups of water is 53. 5:15 = 1:3 = 3:9 54. 10:10=1:1=4:4= i
$.3ine 3:2. Since Grogg wants to keep this
cornstarch-to-water ratio, he must add 3 cups of
cornstarch for every 2 cups of water he asi

Since he adds 1 cup of Aa el and 1 =t °3, we know


that he must add 4.2=2= cups of water.

For each of the following problems, there is only one 55. We can write + with a denominator of 15 by multiplying
solution. the numerator and denominator by 5.
43. The ratio of gray squares to white squares in the original
10
rectangle is 3:3 = 1:1.

We can split the rectangle into three smaller rectangles


as shown below so that the ratio of gray squares to white 56. We can write 3 with a numerator of 15 by multiplying the
squares is 1:1 in each smaller rectangle.
numerator and denominator by 3.

44. The ratio of gray squares to white squares in the original


rectangle is 6:6 = 1:1. So, we can make groups of 2 57. We can write é with a denominator of 32 by multiplying
squares in each of the smaller rectangles, each with 1
the numerator and denominator by 4.
gray square and 1 white square. Therefore, the area of
each small rectangle is a multiple of 2.

We can split the rectangle into three smaller rectangles


as shown below so that the ratio of gray squares to white
squares is 1:1 in each smaller rectangle. 58. Wecanwrite7>with
2 a denominator
of88bymultiplying
thenemanibyonddenominator
by8.
peel
19152
1 88
8
We use the same reasoning discussed in the previous 59. We can write - with a denominator of 42 by multiplying
problems to solve each Rectivide puzzle that follows.
the numerator and denominator by 6.
45. 9:3 =3:1 46. 4:8=1:2=2:4
6
ming val.
7 - 42

73 _18
= Gp SOX bs18

To isolate the variable x, we multiply both sides of the


equation by 42.

60. We can write = with a denominator of 45 by multiplying


the numerator and denominator by 5.

eee“5
9ae
heeES
13 65 4
9 45:80a=65

Ratios & Rates Chapter 8 Solutions Beast Academy Practice 5C


To isolate the variable a, we multiply both sides of the 65. We solve for c as shown below.
equation by 45.
13 i) i gepesS)
“9 (45=75°45 f 4 é
G48
13 =F 48 = Ac=a>4
zi ie =
°Cc=
65 =a.
Pay 7c =60
61. We simplify 72 =<. Then, we can write : with a ent see
denominator of 35 by multiplying the numerator and f jf
denominator by 7. ‘7 66. We solve for a as shown below.

494_~7 ees
85 56 nO
Pa yi 14 eA
21_7_49 = ay tag 2
75 = 5 = 357 SOW=49. 14-:9=4-q
— or— 126 =4a
ead 4 : '
We simplify 15 to 5 Then, to isolate the variable w, we 126_
a 74.
multiply both sides of the equation by 35.
So, a=12683441
Wass 635 ? Awe 2
35 5 s 67. We solve for z as shown below.
Ww are
5 95 = B95 2 15
w=49 2 ¢ is
B-BZ=eZn 5Z
62. We cannot multiply the numerator and denominator 0-7=15-5
of = by an integer to get an equivalent fraction with te
denominator 21. So, we isolate the variable. ahs P ,
To isolate the variable we multiply both sides of the Zag =875.
equation
: by, 21. dean a204 68. We solve for v as shown below.
21 4
9-21 3 10
=F 10°
1 = 477i.
= 18 1 37g .4gy =10. 10¥
OV=5,°
3-v=10°10
63. Wecan eliminate the denominators of =.3m = +2 -3n 3y =100
- and 2 by multiplying both sides of mages =100 441
the equation by acommon multiple of RG ae
their denominators: 3m.
69. We solve for 7 as shown below.
This gives 2m = 45. 2m=45 iad
We divide both sides by 2 to get m= = ' (a f
M=-7 =225. n° N=— on
— or— 8-6=11°-n
For any equation ¢ = & we have ad= be. 48 =11n
So, £=12 gives 2-m= 15-3. “ ' 48
Ti 7:

We simplify, then solve for m: So, n=47=447-


2-mM=15°3 70. We solve for r as shown below.
= 10_6
esc aoe O.7r=8.7/

64. We solve for s as shown below. {00ra Ge?


12°25
mine a0 a i ri ea |
Eger Ie
78h = z Bs erat sem
10° 5L aie 5
12-8=5°s
96 =5s
96 _
5 =
So, -%_491
S== =19¢.

Beast Academy Practice 5C Ratios & Rates Chapter 8 Solutions


Then, we multiply both sides by 16 to solve for s as
shown. 5 oe
76 =>:16
In these problems, you may have flipped the order of the
aa
quantities in the ratios to arrive at the same final answer. =5
71. Suppose there are v vowels in the sentence. Then, the So, a = a= ounces of sugar should be added to 16
ratio of 40 consonants to v vowels is 40:v. Since there ounces of water to make hummingbeast nectar.
are 5 consonants for every 4 vowels, we have 40: = 5:4.
75. Two sevenths of Terry's total hits are home runs. Since
Since 40:32 = 5:4, we have v = 32. So, the sentence has two sevenths of his total hits is 28 hits,
32 vowels. one seventh of his total hits is 14 hits.

Therefore, Terry hit the ball 7-14 = 98 times, and he did


not hit a home run 98-28 = 70 times.
We express 40:v = 5:4 with fractions.

40 _5 <— consonants
y 4 <— vowels
Since Terry hits a home run 2 out of every 7 times that
We multiply both sides by 4¥ to solve for v as shown.
he hits the baseball, his ratio of home-run hits to
AO eee
5 - non-home-run hits is 2:5. .
Fae
Suppose Terry made h non-home-run hits. Then, the
40-4=5-v
ratio of 28 home-run hits to A non-home-run hits is 28:h.
160 =5v Since he made 2 home-run hits for every 5
160 _ non-home-run hits, we have 2:5 = 28:h, which gives
z=
160 2 _ 28 <— home-run hits
So, the sentence has —5 = 32 vowels. 5° h << non-home-run hits

72. Suppose Cori needs c cups of cocoa. Then, the ratio of 2


Since ae = we have h=70. Therefore, Terry had 70
5 cups of sugar to c cups of cocoa is 5:c. Since we want
non-home-run hits.
9 parts sugar for every 2 parts cocoa, we have 5:c = 9:2,
76. The area of triangle ABC is ae = 18 square inches.
which gives
5_9 <—@sugar The ratio of the area of triangle ABC to the area of
c 2 <— cocoa triangle DEF is 3:5. Since the area of triangle ABC is
18 =6-3 square inches, the area of triangle DEF is
We multiply both sides by 2c to solve for c as shown.
6:5=80 square inches.

The area of a right triangle is half the product of its leg


lengths, so the product of the lengths of EF and ED is
30-2 =60. Since EF is 6 inches long, ED is 60+6=10
inches long.

So,Corineeds= =1=cupsofcocoa. The area of a triangle is equal to half the product of


73. Suppose the Blorgbeast's hoof is w inches wide. Then, its base and height. So, the ratio of the areas of two
the ratio of w inches of hoof width to 5 inches of hoof triangles that have the same base is the same as the
height is w:5. Since the ratio of hoof width to hoof height ratio of their heights.
is 3:8, we have w:5 = 3:8, which gives
We consider AB and EF as the bases of their triangles,
w_3 << hoof width
5a8
so BC and DE are the heights. Since the ratio of the area
<—_ height
of triangle ABC to the area of triangle DEF is 3:5, and
We multiply both sides by 5 to solve for w as shown. their bases are the same length, the ratio of the height of
Ww aS triangle ABC to the height of triangle DEF 3:5.
gee 9
BAS Since BC is 6 =2:-3inches long, DE is 2-5 = 10 inches
Wee long.
So,thehoofis“ =i- inches
wide. 77. Since the ratio of the weight of a biffo to the weight of a
74. Suppose we need to add s ounces of sugar. Then, the triffo is 2:3, the ratio of the weight of a triffo to the weight
ratio of s ounces of sugar to 16 ounces of water is 5:16. of a biffo is 3:2. This means that the weight of a triffo
Since we want 2 parts sugar for every 7 parts water, we
divided by the weight of a biffo is 3.
have s:16 = 2:7, which gives
So, the weightofa triffois 2 the weightof a biffo.
i=5
S sugar
Since
30biffos
weigh
40ste) 30iis weigh
>
3times
16°7 <— water

asmuch.
Therefore,
30triffos
weigh
53=°40=60
ae

Ratios & Rates Chapter 8 Solutions Beast Academy Practice 5C


Since the ratio of the weight of one biffo to the weight of The ratio of the long side of the first rectangle to the long
one triffo is 2:3, the ratio of the weight of 30 biffos to the side of the second rectangle is 8:w.
weight of 30 triffos is also 2:3.
The rectangles are similar, so 5:15 = 8:w, which gives
Suppose 30 triffos weigh w pounds. Since 30 biffos 3 ==. Solving
forw,wehavew=24.
weigh 40 pounds, the ratio of biffo weight to triffo weight 81 . The ratio of the short side of the first rectangle to the
is 2:3 = 40:w, which gives
short side of the second rectangle is 14:8.
2 40 <— biffo weight
3 w ~<— triffo weight The ratio of the long side of the first rectangle to the long
‘ 2_ 40 ; : side of the second rectangle is 21:r.
Since 3 = Go We have w=60. So, 30 triffos weigh 60
pounds. ihe rectangles are similar, so 14:8 = 21:7, which gives
aot Solving for r, we have r= 12.
78.
Sincea:b=3:5,weknow
thatais =ofa+b,andbis2 82. The ratio of the short leg of the first triangle to the short
ofa+b. leg of the second triangle is 7:6.
Since a+b = 100, we have
The ratio of the long leg of the first triangle to the long leg
>.100== 3-374, of the second triangle is 10:18.
Sate brad.
Similarly, 1 q<1800
-125
b=5100==-=->- _ 901
=62>. The trianglesare similar,so ¢:6=10:18,vas gives
£25ie . Solvingfort, we have Bas30-10-31
The ake ofthe shortestside ofae firsttriangleto the
peeae 83.
The ratio of a to D is 3:5. So, for some value of x, we shortestside ofthe second triangleis 6:9.
have a=3x and b=5x.
The ratio of the longest side (the side opposite the
Since a+b = 100, we have obtuse angle) of the first triangle to the longest side of
3x%+5x =100 the second triangle is 14:x.
The triangles are similar, so 6:9 = 14:x, which gives
= =. Solving
forx,wehavex=21.
The ratio of the remaining side of the first triangle to the
remaining side of the second triangle is 10:y.
So, we have The triangles are similar, so 6:9 = 10:y, which gives
cha Solving for y, we have y= 15.
B=374and
eee =12
=624. = ee We can divide the square faces of the small cube into
5°5=25 little squares, as shown on the left below. Since
4 the ratio of the edge length of the small cube to the edge
Check: 374:62 =3:5,and374+625=100. ¥ length of the large cube is 5:7, we can place 7 of the
79.
Of t hegoalss cored b yBen and
Ben and g4 were scored by Aine
Alfie,
2were
s cored
by same little squares along each side of a face of the large
cube. So, we can divide each face of the large cube into
So, Ben sciteraa Alfie by 2 rio ~ of their total goals. 7°7=49 little squares.
Since Ben scored 16 more goals than Alfie, 16 is one
ninth of the total goals scored by Ben and Alfie.
hogemies, they scored 9-16= est goals. Ben scored
3.444 = 80, and Alfie scored =°144
< =64.Ben scored
in
80-64 = 16 more goals. W Each cube has 6 congruent faces. So, the surface area
of the small cube is 6-25 = 150 of these little squares,
and the surface area of the large cube is 6-49 = 294 of
For every 5 goals Ben scored, Alfie scored 4. So, for these little squares. Therefore, the ratio of the surface
some value of x, Ben scored 5x goals and Alfie scored area of the small cube to the surface area of the large
4x goals. Therefore, Ben outscored Alfie by 5x-4x =x cube is 150:294 = 25:49.
goals. Since Ben scored 16 more goals than Alfie, we
have x = 16. So, Ben scored 5x =5:14=80 goals, and
Alfie scored 4x = 4-16 = 64. Since each cube has 6 congruent faces, the ratio of the
surface area of the small cube to the surface area of the
Check: The ratio of Ben's goals to Alfie's goals is
large cube is equal to the ratio of the area of one face
80:64 = 5:4, and Ben scored 80-64 = 16 more goals. W
of the smaller cube to the area of one face of the large
cube.

Above, we found that when we split the faces of these

80. The ratio of the short side of the first rectangle to the cubes into little squares of the same size, a face of the
short side of the second rectangle is 5:15. small cube has 5-5 = 25 little squares, and a face of the
large cube has 7-7 = 49 little squares.

Beast Academy Practice 5C Ratios & Rates Chapter 8 Solutions


So, the ratio of the surface area of the small cube to the 3
==gO):
Since 35 =75 Wea 4m=75. So, Wa 2 , and the
surface area of the large cube is 25:49.
twocities
ar ee 183miles
apart.
For every 5 units in the edge length of the small cube,
—or—
there are 7 units in the edge length of the large cube. Since3 mapinchesrepresent
25actualmiles,
every==1map-inch represents
2=3=84miles.
So, for some number of units x, the small cube has
5x-unit edges, and the large cube has 7x-unit edges. SinceGriffinburg
andNorthPegasus are2a
Since each cube has 6 congruent faces, the ratio of the
map-inches
apart,thesewecitiesareactually
surface area of the small cube to the surface area of the
=—>= 183milesapart.
large cube is equal to the ratio of the area of one face of
b. The ratio of map inches to actual miles is 3:25.
the small cube to the area of one face of the large cube.
Suppose Unicorpia is 7 map inches away from West
The area of a face of a cube with edge length 5x units is
Mermaid. Since 3 map inches represent 25 miles,
5x-5x = 25x* square units. The area of a face of a cube
and m map inches represent 40 miles, we have
with edge length 7x units is 7x-7x =49x? square units.
3:25 = m:40, which gives
So, the ratio of the surface area of the small cube to the
325 _m
40 <—@mapinches
<— actual miles
surface area of the large cube is 25x?:49x°. We divide
both quantities by x? to simplify this ratio to 25:49. Then, we multiply both sides by 40 to solve for m as
85. a. The ratio of model inches to actual feet is shown.
35:50 ="2:50 = 15:200 = 3:40.

Suppose the length of the original train car is f feet.


Since 3 model inches represent 40 actual feet, and
6 model inches represent f actual feet, we have
3:40 = 6;f, which gives
So, West Mermaid would be =44inches
from
Unicorpia on the map.
3 __6 <— modelinches
40 f <— actual feet
Since 40
&Ser we have f= 80. The actual train car is
80feetlong. 87. The ratio of red to blue to yellow shirts is 18:30:45. We
b. In part (a), we found that the ratio of model inches to simplify the ratio by dividing 18, 30, and 45 by their
greatest common factor, 3. So, 18:30:45 = 6:10:15.
actual feet is 3:40.
. There are 60-15-14=31 fifth graders in the parade.
Suppose the model track is m inches long. Since
The greatest common factor of 15, 14, and 31 is 1, so the
3 model inches represent 40 actual feet, and: m
ratio of third to fourth to fifth graders is 15:14:31.
model inches represent 12,000 feet, we have
3:40 =™m:12,000, which gives From the given ratio, we know that we can oy groups
3 __ Mm <<— model inches of 8+5+3=16 fish, each with 5 goldfish. So 16= of the
40°” 12,000 <«— actual feet fish are goldfish.
From the Fuji-to-Gala sa a we know that g= Of Rolfe's
Multiplying both sides by 12,000, we have
m= + 12,000 = 3-300 = 900. So, Chis's model is 900 apple trees are Fuji and =
3 are Gala.
inches long. From theaeabhieebereratio,
weknowthat4 ofhis
There are 12 inches in a foot, so Chris's model track treesareapple
trees,and=arepeach
trees.
is 900+ 12 =75 feet long. 3.4_
Therefore, =
ara 3 of all Rolfe's trees are Fuji° apple

86. a. Suppose Griffinoburgand North Pegasus are actually eee 2.222 of his trees are Gala apple trees, and
m miles apart. aie 3 map inches represent 25 3
>= 3 of his trees are peach trees.
actual miles, and 254 map inches represent m miles,
we have 3:25 = 2+ om So, we can make groups of 14 trees, each with 3 Fuji,
5 Gala, and 6 peach. The ratio of Fuji to Gala to peach
We eliminate the oes in the ratio on the right side trees is 3:5:6. 3
of the equation by multiplying both quantities by 4:
24:m=9:4m. The ratio of Fuji trees to Gala trees is 3:5. So, we can
make groups of 8 apple trees, each with 3 Fuji and 5
Therefore, 3:25 = 9:4m, which gives
Gala.
3 __9
> <—= map inches
25. 4m <— actual miles The ratio of apple trees to peach trees is 4:3. Since
apple trees are easier to think about in groups of 8 than
in groups of 4, we write the apple-to-peach ratio as
4:3 = 8:6.

Ratios & Rates Chapter 8 Solutions Beast Academy Practice 5C


So, we can make groups of 8+6 = 14 trees, each with 8 95. From the given ratio, we know that a is fat of a+b+c,
apple trees and 6 peach trees. Three of the apple trees bis + of a+b+c, and c is 4 of a+b+tc.
are Fuji, while the other 5 are Gala.
Since pai =70, we have a= 4. 70 = 10.
Similarly, b =<. -70=45 and cas. -70=:15:
Therefore, sips = 10-45-15 =6,750.

rcaeps
The ratio of Fuji to Gala to peach trees is 3:5:6.
The ratio of a to b to c is 2:9:3. So, for some value of x,
91.
Theratioofbutter
toPs toflour is we have a= 2x, b= 9x, and c=3x.
12:ot 22 =9°9°3
=3.5.8 Since a+b+c=70, we have
To write this as a ratio ey integers, we multiply 2x+9x+3x=70
every quantity by the least ae multiple of the 14x=70
denominators, 6. So, 3a=5" S 8.
= 9:15:16.
92. From the given ratio, we know that for every 5 units that
the prism is long, it is 6 units wide and 8 units tall.
So, we have
Since the prism is 10-6 =60 cm wide, it is 10-5=50 cm
long and 10:8=80 cm tall. Gay =2.5=10)

A 50-by-60-by-80-cm prism has two 50-by-60-cm faces, b=9x=9-5=45, and


two 60-by-80-cm faces, and two 50-by-80-cm faces. C=3x=3-5=15.
Therefore, the surface area of the prism is
Therefore, abc = 10-45-15 =6,750.
2-(50°60)+2:(60-80)+ 2:(50-80)
Check: 10:45:15 = 2:9:3, and 10+45+15=70. YW
= 2-(3,000+4,800+4,000)
96. From the given ratio, we know that for every 2 inches that
=2-11,800
the prism is long, it is 5 inches wide and 8 inches tall.
= 23,600 square centimeters.
So, for some number of inches x, the prism is 2x inches
93 From the given ratio, we know that 7= =4 of the marbles in long, 5x inches wide, and 8x inches tall, and its volume is
the bag are blue. Since the 16 blue marbles are one third
2x-5xX+8x = (2°5+8)+(x-x+x) = 80X° cubic inches.
of the marbles in the bag, the bag holds a total of
16-3=48 marbles. The volume of the prism is 2,160 cubic inches, so we
have 80x? = 2,160. Dividing both sides of this equation by
94 From the green-to-yellow frog ratio, we know that 2 of the
80 gives x* = 27. Since 3° = 27, we have x=3.
frogs are green and “ are yellow.
So, the prism is 2x = 2:3 =6 inches long, 5x=5-3=15
From the frog-to-toad ratio, we know that s of these
inches wide, and 8x = 8-3 = 24 inches tall.
animals are frogs and “ are toads.
Check: The volume of a 6-by-15-by-24-inch rectangular
3.3
5
Beers 5"B=
23
20 =. of the animals are green frogs, prism is 6-15-24 =2,160 cubic inches.

5"oe20 20
=oftheanimals
areyellow
frogs,
and
“og
5 2=8oftheanimals
aretoads.
So, we can make groups of 40 animals, each with 9 97. The ratio of the triangles that touch

green frogs, 6 yellow frogs, and 25 toads. The ratio of the left square is 6:15 = 2:5, and the

green frogs to yellow frogs to toads is 9:6:25. ratio of the triangles that touch the
right square is 6:16 =3:8.

The ratio of green frogs to yellow frogs is 3:2. So, we can 98. Every triangle contains an integer, so the ratio in the left
make groups of 5 frogs, each with 3 green and 2 yellow. square tells us that the smaller number in the triangles
The ratio of frogs to toads is 3:5. Since frogs are easier that touch it is a multiple of 2.
to think about in groups of 5 than in groups of 3, we write Since 9 is not a multiple of 2, it must be the larger
the frog-to-toad ratio as 3:5 = 15:25. number in the triangles that touch this square.
So, we can make groups of 15+25 = 40 animals, each [69 = 2:3, so the empty triangle contains 6.
with 15 frogs and 25 toads. In each group of 15 frogs,
Then, the ratio of the triangles that touch the right square
3-3=9 are green and 3-2 =6 are yellow.
is 6:8 = 3:4.

Green |Yellow| Toads

The ratio of green frogs to yellow frogs to toads is 9:6:25.

Beast Academy Practice 5C Ratios &Rates Chapter 8 Solutions [149]


99. The ratio of the triangles that touch the left square is 2:9. So, the top-center triangle is 18.
Since 4 is a multiple of 2 but not 9, it must be the smaller
The ratio of the triangles that touch
number in the triangles that touch this square.
the top-left square is 3:4. Since 18 is a
4[18]= 2:9, so the empty triangle contains 18. multiple of 3 but not 4, it is the smaller
number, and we have 18:[24]= 3:4.
Then, the ratio of the triangles that touch the right square
is 6:18 = 1:3.
The ratio of the triangles around the
bottom-left square is 5:8. Since 24 is
a multiple of 8 but not 5, it is the larger
number, and we have [15|:24 =5:8.

Finally, the ratio of the triangles that


100. The ratio of the triangles that touch touch the bottom-right square is
the square on the left is 1:2. Zoe
If 2 is the larger number, we have 105. The ratio of the triangles that touch
[4 |:2 = 1:2. If 2 is the smaller number,
the top-left square is 1:3. If 12 is the
then we have 2:[4]= 1:2. So, the larger number, we have [4}:12 = 1:3.
center triangle is 1 or 4. If 12 is the smaller number, we have
The ratio of the triangles that touch the 12[36]= 1:3. So, the top triangle is
right square is 2:3. There is no integer 4 or 36.
that has a 2:3 ratio with 1. So, the The ratio in the top-right square tells us
center triangle is 4. that the smaller number in the triangles
that touch it is a multiple of 7.
There is no integer smaller than 4
that has a 2:3 ratio with 4. However, Neither 4 nor 36 is a multiple of 7, but
if 4 is the smaller number in the 2:3 the other triangle that touches this
ratio, then we have 4:6 |=2:3. So, the square contains 21, which is a multiple
upper-right triangle is 6. of 7. So, 21 is the smaller number in
the triangles that touch the top-right
We use the strategies discussed in the previous solutions square, and 36 is the larger number.
to solve the Ratiotile puzzles that follow.
The ratio of the triangles that touch the
101. top-right square is 21:36 = 712].

The ratio of the triangles that touch the


bottom-right square is 3:7. If 21 is the
larger number, we have [9}21 =3:7.
103.

es If 21 is the smaller number, we have


2149]= 3:7. So, the bottom triangle is
9 or 49.

However, if 12 and 49 are the two


triangles that touch the bottom-left
104. The ratio of the triangles that touch square, their ratio cannot simplify to
the top-right square 2:3. 3:[_], with an integer in the blank.

If 12 is the larger number, we have Therefore, the bottom triangle is 9, and


[8|:12 =2:3. If 12 is the smaller the ratio of the triangles that touch the
bottom-left square is 9:12 = 314 |.
number, we have 12:[18]= 2:3. So, the
top-center triangle is 8 or 18. . The ratio in the left square is either 1:5, 2:5, 3:5, or 4:5.
Since 16 is not a multiple of 5, we know 16 is the smaller
Suppose we place 8 in the top-center number in the ratio. Also, since 16 is not a multiple of 3,
triangle. The ratio of the triangles that the ratio of the triangles that touch the left square cannot
touch the top-left square is 3:4. Since be 3:5.
8 is a multiple of 4 but not 3, it is the
larger number, and we have [6|8=3:4. We consider the remaining possibilities.
1:5: Since 16{80]= 1:5, the bottom
However, the 6 triangle also touches
triangle can be 80.
the bottom-left square. We cannot YY

2:5: Since 16[40]= 2:5, the bottom ua

make a ratio with 6 and another integer


20 or 40 or 80
that simplifies to 5:8. sx triangle can be 40.
4:5: Since 16[20]= 4:5, the bottom
triangle can be 20.

Ratios & Rates Chapter 8 Solutions Beast Academy Practice 5C


[20}:30= 2:3,but30[40]=3:4and Finally, [24]:32:40 is the only
3080]= 3:8.Only20 can fillthe ratio of three integers with 32
bottomtriangleso thatthe ratiointhe and 40 that simplifies to 3:4:5.
rightsquaresimplifiesto[_|3. 20or 48orBE So, the bottom-left triangle is 24.
So, the center triangle is 20. Then,
the ratio of the triangles that touch
110. The ratio of the triangles that
the left square is 16:20 = 4:5, and
touch the bottom-left square is
the ratio of the triangles that touch
2:3:5. Since 9 is not a multiple
the right square is 20:30 = 2:3.
of 2 or 5, the middle value of
We use the strategies discussed in the previous solutions this ratio is 9. Then, we have
to solve the Ratiotile puzzles that follow. [6]:9:[15] = 2:3:5, and the empty
107. triangles that touch the bottom-left 6 and 15
square are 6 and 15.

The ratio of the triangles that touch


the bottom-right square is 2:2:7.
109. The ratio of the triangles that 12 and 32 Since 15 is not a multiple of 2 or 7, 7
touch the top-left square is we cannot place 15 in the bottom
3:5:8. Since 20 is not a multiple triangle.
of 3 or 8, we know 20 is the So, we place the 6 and 15 as shown.
middle value of this ratio. So, we
have [12|:20[32]= 3:5:8, and the The ratio of the triangles that touch
empty triangles that touch the the bottom-right square is 2:2:7.
Since 66}{21]=2:2:7, the two
top-left square are 12 and 32.
other triangles that touch this
The ratio of the triangles that
square are 6 and 21.
touch the top-right square is 5:6:7.
6 and 21
Since 32 is not a multiple of 5, 6,
or 7, we cannot place 32 in the 42 and 84
top triangle. Suppose we place 21 in the
middle-right triangle as shown.
So, we place the 12 and 32 as shown.
Then, the only ratio of three
The ratio of the triangles that touch integers with 21 that simplifies
the top-right square is 5:6:7. Since to1:2:4is21[42}(84]
=1:2:4.
12 is not a multiple of 5 or 7, it
is the middle value of this ratio.
Then, we have [10]:12[14]=5:6:7, Xe
However, the top triangle also
and the empty triangles that touch touches the top-left square. We
the top-right square are 10 and 14. cannot make a ratio of integers
with 9 and either 42 or 84 that
The shaded empty triangle to the simplifies to 1:3:3. x
right also touches the bottom-right
square. There is no ratio of three Therefore, we place the 6 and 21
integers with 14 and 50 that in the triangles around the
simplifies to 1:4:5. bottom-right square as shown.
Therefore, we place the 10 and 14
as shown.

The ratio in the top-right square is (12 and 24)


1:2:4. Since 6 is a multiple of both or (3 and 12)
1 and 2 (but not 4), we have
10{40]:50 is the only ratio of
6[12]/24]=1:2:4,or
three integers with 10 and 50
that simplifies to 1:4:5. So, the
[3}:6[12]
= 1:2:4.
bottom triangle is 40. So,the twoothertrianglesthat
touch this square are (12 and 24)
or (3 and 12).

Beast Academy Practice 5C Ratios & Rates Chapter 8 Solutions [151]


There is no ratio of three integers (12 and 24) At the rate of 5 baby dragons per day, 5-12 = 60 baby
with 9 and either 12 or 24 that or (3 and 12) dragons will be born in the next 12 days on Beast Island.
simplifies to 1:3:3.
So, only 3 can be in the triangle
117. iy ee S==
apples perpie=6applesperpie.
that touches both the top-left and
top-right squares. bayseeouices
Ores = ounces
ofsugarperpie
8pies
=3ounces
ofsugarperpie.
Therefore, we place the 3 and
12 as shown. 18
Cc.faderdollars
ec =dollars
perpound
ofapples.
- =14and==799SO14dollars
perpound
ofapples
is 1.20 abies per pound of apples, or $1.20 per
pound of apples.
Finally, 3:99] is the only ratio of
three integers with 3 and 9 that d. 15ounce es 5 pounds
ofapples
perpie
simplifies to 1:3:3. So, the far-left = 12 pounds of apples per pie.
triangle is 9.
e, 24 ounces of
48 applessugar _ 24
= Gg ounces of sugar per apple

= 3 ounces of sugar per apple.

111. Since there are 85 sheets per roll,


2 rolls have 2:85 =170 sheets, f. a15pounds
_15pounds
ee =73 perapple
5 rolls have 5:85 = 425 sheets, =3 pounds
perapple.
10 rolls have 10-85 = 850 sheets, and
55 rolls have 55-85 = 4,675 sheets. 118. He receives 300Torts=
75 Beastbucks
Torts
perB eastbuck
112. Bess can travel 160 miles using 5 gallons. So, she =rs#39perBeastbuck.
travels
=ar == miles
pergallon
=32miles
pergallon. 119. Since he will receive 4 Torts per Beastbuck, he will
So, her car will go receive 4-100 = 400 Torts for 100 Beastbucks.
1-32 =32 miles with 1 gallon of gas,
3°32 =96 miles with 3 gallons of gas, 120. : 75
HereceivesBeastbucks
300 Toris.300Beastbucks
perTort
10-32 = 320 miles with 10 gallons of gas, and
=F+Beastbucks
perTort.
55°32 = 1,760 miles with 55 gallons of gas.
121.
113. Grogg buys 6 pens for $2.10 = 210 cents. So, he spends Since a ltJones r eceives+Beastbucks
receive Z2 - 480 = 120 Beastbucks
per T
for 480 Torts.
ort,
hewill
210cents _210
“6pens ~~6 cents per pen = 35 cents per pen.
At this rate, he can buy
Since 4 Torts are worth 1 Beastbuck, he will receive
1 pen for 1-35 =35 cents ($0.35),
480+4 = 120 Beastbucks for 480 Torts.
5 pens for 5-35 = 175 cents ($1.75),
10 pens for 10-35 = 350 cents ($3.50), and
55 pens for 55-35 = 1,925 cents ($19.25).
114. The 16-pound bag from Arthur's Grocery costs 122. Theblimp
Peleg90lee in3deredsoitsaverage
12 dollars _ 12 dollars per pound = dollars per pound,
16 pounds 16 speed
was
which is 75 cents per pound. 90 miles _ 30 miles _ ‘
atoms o BAGH Ek30 miles per hour.

The i eee bagf rom


8dollars _ =dollars
B etty's
oe costs 123. One day is 24 hours. So, in one hour, kudzu grows
10pounds— perpound==~
=dollars
perpound, 15in__
15in _Sin _5
which is80oe perpound. 1day 24hr 8hr 8
inches per hour.

145 ft
So, the 16-pound bag from Arthur's Grocery costs 124. The speed of the ball was
24 sec)
less per pound.
To write the ball's speed as a number of feet per second,
115. Benji can type 1,500 words in 30 minutes, which is a rate
we first write this speed as the relationship between a
of 1,500+30 =50 words per minute.
whole number of feet and a whole number of seconds.
At the rate of 50 words per minute, Benji needs
5,000+50 = 100 minutes to type his 5,000-word essay.
4145 2
ft_~
ft _™290 ft
s5ec
116. Forty-five baby dragons were born in the last 9 days, 24sec
which is a rate of 45+9 =5 baby dragons born per day.

[152
| Ratios
&Rates
Chapter
8Solutions Beast Academy Practice 5C
So, the average speed of the ball was
290 ft 58 ft
= 58 feet per second.
5sec 1sec 5 mowers can mow 4 fields in 60 minutes. It will ae
132. a.
1 mower five times as long to mow the same number
Since the ball traveled 145 feet in 2 seconds, it can of fields.
travel 145+2> feet in 1 second. So, 1 mower can mow 4 fields in 5:60 = 300 minutes.

b. 1 mower can mow 4 fields in 300 minutes. It will take


So, the average speed of the ball was 58 feet per
second.
tas longtomow
1 field.
125. Robbie's average speed
is 6——is So, 1mower can mow1 field in +-300=75 minutes.
2 hr
3 C. 1 mower can mow 1 field in 75 minutes. It will take 3
To write this speed as a number of miles per hour, we i as long to mow 1 field together.
mowers 3
first write this speed as the relationship between a whole
number of miles and a whole number of hours. So, 3 mowers can mow1 field in 4-75 = 25 minutes.

°3 d. 3 mowers can mow 1 field in 25 minutes. It will take


TSI 0:1 45M them 5 times as long to mow5 fields.
1$hr —» Shr So, 3 mowers can mow 5 fields in 5-25=125
minutes.
So, Robbie'saverage speed is 133 a. 6 hoses can fill 5 buckets in 15 minutes. It will take
45
- mm=<—
9 > =9 miles per hour. them = as long to fill 1 bucket.
So, 6 hoses can fill 1 bucket in i. 15=3 minutes.

Since Robbie ran 15 miles in 1é hours, he can run b. 6 hoses can fill 1 bucket in 3 minutes. So, in
24 = 8:3 minutes, they can fill 8 times as many
15+13 miles in 1 hour.
buckets.
1521$=1523=15-2=9,
So, 6 hoses can fill 8: 1=8 buckets in 24 minutes.
So, Robbie's average speed is 9 miles per hour.
: 1 ; C. 6 hoses can fill 8 buckets in 24 minutes. Three hoses
126. Erica can run 45 meters per second. =) in 45 seconds,
will fill half as many buckets in the same time.
shecanrun45-aie 45;g= =2024meters.
127.
Theboatcould
travel
25Ne inoneoi atmaximum So, 3 hoses can fill 4-8 = 4 buckets in 24 minutes.

134. a. 3 gazellephants can eat 3 buckets in 3 minutes.


piped-13okanhour,
theboatcanonlygo31asfar:
3-25=2=84miles. 3 gazellephants can eat 6 buckets in twice the time it

128. pos hikes


aan of8+8=16
kilometers
in3+2=5 takes them to eat 3 buckets. So, 3 gazellephants can
eat 6 buckets in 2-3=6 minutes.
hours. So, Grogg's average speed for the entire trip is
16km_
a 16 eat
km per hour = 35km per hour. 6 gazellephants can eat 6 buckets in one-half the
time it takes 3 gazellephants. So, 6 gazellephants
129. Allison's team are a total of 5+3 =8 miles in can eat 6 buckets of peanuts in 7'6 =3 minutes.
a pra
Fis ii Cae f=15 hours.
So, their average ies was b. In part (a), we determined that 6 gazellephants eat 6
buckets in 3 minutes.
Simio.
ise 24mij2 6mi 4
Vg ageesA =6 miles per hour. 6 gazellephants will eat twice as many buckets in
3 twice the time. So, 6 gazellephants can eat 2-6 =12
130. Four hours is 240 minutes, so Annalise walks 10 miles
buckets in 6 minutes.
in 240 minutes. Therefore, she walks each mile in
240+10=24 minutes. 135. a. 12 paint-bots can il houses in 9 hours.
Painting
5 housesis a=
=~ me
1 theworkofpainting
20
It takes Annalise 4 hours to walk 10 miles. She will travel houses. So, Z
=: as many bots are needed to paint 5
one tenth of the distance in one tenth of the time. So, it houses as ae need to paint 20 houses. Therefore,
will take her = =< hours to walk = =1 mile.
4.12 =3 paint-botscan paint5 houses in9 hours.
Onehouris60minutes,
so2 hours
is2.60=24minutes.
. One hour is 60 minutes, so Urlich can paddle 5 miles
in 60 minutes. Therefore, he paddles each mile in
60+5=12 minutes.

So,his44-mile
tripwill
take44-12=2-12=54
minutes.
Da
Beast Academy Practice 5C Ratios
&Rates
Chapter
8Solutions
| 153
|
b. 3 paint-bots can paint 5 houses in 9 hours. 137. a. 6 lumberjacks can chop 5 logs in 40 minutes.
9 paint-bots can paint 5 houses in one-third the time 24 lumberjacks can chop 5 logs in one fourth the time
it takes 3 paint-bots. Therefore, 9 paint-bots can it takes 6 lumberjacks to chop the logs. Therefore, 24
paint 5 houses in 4.9 = 3 hours. lumberjacks can chop 5 logs in +.40 = 10 minutes.
9 paint-bots can paint twice as many houses in 6 So, 24 lumberjacks will chop 3 times as much wood
hours as they can in 3 hours. So, 9 paint-bots can in 30 minutes as they can in 10 minutes. Therefore,
paint 5-2 = 10 houses in 6 hours. 24 lumberjacks can chop 3-5 = 15 logs in 30 minutes.

b. We begin by considering how long it will take for 1


12 paint-bots can Ree 20 houses in 9 hours. lumberjack to chop 1 log.
= as many houses as 12
9 paint-botscan paint =
-Saws 6 lumberjacks can chop 5 logs in 40 minutes.
paint-bots.
1 lumberjack takes six times as long as 6
In 6 hours, only ¢=43 as many houses can be lumberjacks to chop 5 logs. So, 1 lumberjack can
painted as can be painted in 9 hours. chop 5 logs in 6-40 = 240 minutes.
So, 9 paint-bots can paint $.2=5 as many houses 1 lumberjack can chop 1 log in one fifth the time it
in 6 hours as 12 paint bots in 9 hours.
takes him or her to chop 5 logs. So, 1 lumberjack can
Therefore, 9 paint-bots can paint 4-20 = 10 houses chop 1 log in 4-240 = 48 minutes.
in 6 hours. Now, we use our 1-lumberjack-1-log time to find how
many lumberjacks can chop 6 logs in 72 minutes.
136.a. Building
10carpenters take G ae
12 sheds | is 12
tobuild 8sheds.
- =15 times the work of 1 lumberjack takes six times as long to chop 6 logs
building 8 sheds. So, it takes 10 carpenters 15 times as it takes him or her to chop 1 log. So, 1 lumberjack
as long to build 12 sheds as it takes them to build 8 can chop 6 logs in 6-48 = 288 minutes.
sheds. 72 minutes is one fourth as long as 288 minutes. So,
Therefore, 10 carpenters take 18- 135 27 days to it will take 4 times as many lumberjacks to chop 6
build 12 sheds. logs in 72 minutes as it takes to chop the logs in 288
minutes. Therefore, it takes 4-1 =4 lumberjacks to
b. 10 carpenters take 18 days to build 8 sheds.
chop 6 logs in 72 minutes.
1 carpenter takes 10 times as long to build 8 sheds
as it takes 10 carpenters. Therefore, 1 carpenter
takes 18-10 = 180 days to build 8 sheds. 6 lumberjacks can Cre 5 logs in 40 minutes.
3 carpenters take one third as long to build 8 sheds 72 minutes is Za? 5= of 40 minutes. So, 6 lumberjacks
oeittakes1carpenter.Therefore,
3carpenterstake can chop 5 as many logs in 72 minutes as they can
=°180=60pe tobuild8 sheds. chop in 40 minutes. Therefore, 6 lumberjacks can
15
pees igus
P= ~aslongas60days.So,3 chop BP =9 a e in 72 minutes.
6 logs is :==F2 of 9 logs. So, ittakes 32 as many
carpenters
building
for15dayscanonlybuild+as
manyshedsastheycanin60days. lumberjacksto chop6 logs as ittakes to chop9 logs.
Therefore,=.6 = 4 |lumberjackscan chop6 logs in
eee 3carpenters
working
for15dayscan
build
8:t=2 sheds. 72 minutes.

10 carpenters take 18 oe to build 8 sheds.


3 carpenters can build +9 as many sheds as 10 138. ae are3:5,280=15,840feat|ins mies.
carpenters can. 139. Twenty yards is 20:3=60 feet, and 60 feet is
In 15 days, only a2 as many sheds can be built 60-12 =720 inches.
as can be built in 18 days. 140. Thirty kilometers is 30- 1,000 = 30,000 meters, and
So,3carpenters
canbuild
70

5.3=-
~ asmany
sheds 30,000 meters is 30,000- 100= 3,000,000 centimeters.
in 15 days as 10 carpenters can build in 18 days. 141. There are 7 days in 1 week, 24 hours in 1 day, and 60
Therefore, 3 carpenters working for 15 days can minutes in 1 hour.
build 8-7 =2 sheds.
‘One week is 7 days, and 7 days is 7-24 = 168 hours, and
168 hours is 168-60 = 10,080 minutes.

142. There are 60 minutes in an hour and 60 seconds in a


minute, so one hour is 60:60 = 3,600 seconds.

So, a person jogging 2 meters per second will jog


2°3,600 = 7,200 meters in an hour.

Beast Academy Practice 5C


Ratios &Rates Chapter 8 Solutions
Since there are 1,000 meters in a kilometer, jogging 154. To convert from calories per gram to calories per
7,200 meters in an sete is equal to jogging kilogram, we use the conversion factor oe so that the
7,200_ 72
7.000=707=7.2or7 kilometers
inanhour. grams units cancel.

143. The cups units pe and 15-8=120, so we have 5 calories per g=


Scalories 1,000 g _= 5,000 calories per kg
pee dige
g 1kg
8 fluid ounces
15 cups =15 cups: 1cup = 120 fluid ounces. 155. To convert from feet per minute to miles per minute, we
use the conversion factor mi so that the feet units
144. Theounces
unitscancel,
anda == =33,sowehave cancel.
5,280 ft

60 ounces = 60ounees:a 324 Pounds. We show how these units cancel when we use this
conversion factor in Problem 162.
145. The minutes units cancel, and 10
—= = 2t, so we have
156. To convert from cents per meter to dollars per meter,
150feet _ 1minote
_ we use the conversion factor i aoter so that the cents
150 ft per min = minute ©seconds
60 =24ftpersec. ‘ 100 cents
units cancel.
146. To convert from nanoseconds to shakes, we use el idollar_ 1
: 1 shake 10 cents per meter = Tneier AO <=F0 dollars per meter
the conversion factor aenEnesscoada 5° that the

nanoseconds
units
cancel. 157. To convert from grains per kilogram to oe per
6nanoseconds
=6
nanesecords
-ie ==shakes gram, we use the conversion factor Tes so that the
kilograms units cancel.
147.
Topalo — fathoms
tofeet,weusetheconversion We show how these units cancel when we use this
factor Tita so that the fathoms units cancel.
conversion factor in Problem 159.

30fathoms
=30fathems
ee
Figinem
= 180feet 158. To convert from gallons per foot to gallons per mile, we
use the conversion factor <i so that the feet units
148. To convert from nautical miles to meters, we use the cancel.
;
ConwersiOn 1,852
m eters
factor4 nautical Z
mile2°thatthenauticalmiles We show how these units cancel when we use this
unitscancel. conversion factor in problem 160.
_
meters
1,852
2 nautical miles = 2 nauticatimnies+nauticatmite
7
_ =3,704m Below is the correct matching for #153-158.

149. 153. 4 dollars per meter into 1,000 g


To convert from inches to cubits, we use the conversion
cents per meter. 1kg
factorW 1cubit
inches
B : :
°° thattheinchesunitscancel. 154. 5 caloriesper graminto 5,280ft
calories per kilogram. 1 mile
_1cubit_=5 cubits
90inches=90inches- 78inches 155. 66 feet per minute into 1 mile
miles per minute. 5,280 ft
150. To convert from eat to kilocalories, we use the
14,184
kilocalorie 156. 10centspermeterinto 100cents
conversion factor joules so that the joules units dollars per meter. 1 dollar

cancel. 157. 50,000 grains per kilogram _1


dollar_
1 kilocalorie into grains per gram. 100 cents
10joules 7184joules~2,= kilocalories
=10joutes- 1 ; 1 kg
158. S gallons per foot into 7,000 g
151. To convert from drams per fortnight to pines per gallons per mile.
7
fortnight, we use the conversion: 75ounce
factor Grams S° that
the drams units cancel.
159. We use the conversion factor TH so that the
1 ounce 3
et
6dramsperfortnight CeanisJ
=1fortnight 0unce
16d 2g OZperfortnight kilograms units cancel.

152. To convert from bytes per jiffy to bytes per second, we 50,000grainsper kg =ouSa ae 50 grainsper g
use the conversion factor 100jiffies .” that the jiffies units

cancel. 1second 160. We use the conversion factor 3:=


————
tt <0 that the feet
units cancel.
200
bytes
perjiffy
=a 100HES
-20,000
bytes
persec 5}
1 gallons per foot =piel 20H = 2,640 gallons per mile

153. To convert from dollars per ests to cents per meter, we


use the conversion factor econ so that the dollars 161. :
We use the conversion factor eee t so that the
units cancel. dollars units cancel.
We show how these units cancel when we use this _ 4dotters 100 cents _
4 dollars per meter = Scier ester 400 cents per meter
conversion factor in Problem 161.

Beast Academy Practice 5C Ratios


&Rates
Chapter
8Solutions
| 155
|
162. We use the conversion factor SoaNE so that the feet 169. To convert from pounds per cup to ounces per teaspoon,
units cancel. wemustcancel thepounds andcupsunits.
Dy 66 1 miish 7 pounds per cup = aoe
66feetperminute=Tmin 5,280.8~80 milesperminute So, we choose the conversion aa ih "pounds" in

163. The gallons and minutes units cancel. We can also the denominator and "cups" in the numerator:
cancel some factors in the numerator and denominator.
ie 48 tsp. 1 Ib
30 get 128floz mi
1 48tsp ee 16oz
30 gallons in 8 minutes = 8min 1gaf 60sec
We show how the units cancel when we use these two
at30-428
f oz conversion factors in the next problem.
~ 8-60sec 170. We use the two conversion factors chosen in the
_ 30-16fl oz
~ 568sec previous problem to compute
=8 fluidounces per second 7 pounds per Cup =F" Fa ts5 1 er
164. The cups and batches units cancel. We can also cancel
_7:36oz
some factors in the numerator and denominator.
‘ny
448tsp
a 120grams_11 batch_ a ouncesperteaspoon
3 cups for2 batches = Voas cookies
1B
=220 ounces per teaspoon.
iu3-426grams
~ 2-42cookies 171. To convert from miles per hour to feet per minute, we
= 15 grams per cookie must cancel the mile and hour units.
3 3 mi
165. To convert from feet per hour to inches per minute, we 3 miles per hour= cane

must cancel the feet and hour units. 5,280 feet equals 1 mile. We use the conversion factor
200ft
200 feet per hour=Shen with"miles"
inthedenominator:
22a
So, we choose the conversion factors with "feet" in the 60 minutes equals 1 hour. We use the conversion factor
denominator and "hours" in the numerator: 1 hr
with "hours" in the numerator:
60 min’
1 ft 60 min Therefore, we have
12 in 1 hr
_ Smt 5,280ft 1hf_
3 miles per hour = Tht Tet “60min
We show how the units cancel when we use these two
88
conversion factors in the next problem. 3:5,280 ft
68 min
166. We use the two conversion factors chosen in the -
= 264 feet per minute.
previous problem to compute
200 feet per hour = 200 12in 1hf 172. To convert from cents per inch to dollars per yard, we
Thr 14 60min
must cancel the cents and inch units.
_ _200-3Zin 15
cents
be25 min 15 cents per inch =
1in
40 inches per minute. 100 cents equals 1 dollar. We use the conversion factor
Pe 1 dollar
with "cents" in the denominator: TOOGenie:
167. To convert from gallons per day into fluid ounces per
hour, we must cancel the gallons and day units. 36 inches equals 1 yard. We use the conversion factor
with "inches" in the numerator: ee
ne
450gallons
perday=Te
So, we choose the conversion factors with "gallons" in Therefore, we have
the denominator and "days" in the numerator: 15 cents per inch = LS

3
malagaliny
Tee
128
irozcefl.oz
Ozce1 24h
1dayr —16:36 dollars
n9 188 yd
9
We show how the units cancel when we use these two _ 3:36 dollars
conversion factors in the next problem. 520yd
168. We use the two conversion factors chosen in the =“t dollarsper yard
previous problem to compute
=52 dollarsper yard.
450gallons
perday=e. OR oe
52_= Foo’
40 8° 52 dollars per yard is more commonly written
_450-1261
OZ as $5.40 per ome ,
x gethr
= 2,400 fluid ounces per hour.

Ratios &Rates Chapter 8 Solutions Beast Academy Practice 5C


173. To convert from meters per second to kilometers per Therefore, we have
hour, we must cancel the meters and seconds units.

3 meters per second =


3m 45gallons
perday
=>. eae oe,bf5ae
1 sec’
16
1,000 meters equals 1 kilometer. We use the conversion —45-128flOz
factor with "meters" in the denominator: 1km 37460min
1,000 m°
= 4 fluid ounces per minute.
One minute is 60 seconds, and one hour is 60 minutes,
so 60-60 = 3,600 seconds equals 1 hour. We use the 175. To convert from grams per serving to ounces per box, we
conversion factor with "seconds" in the numerator: must cancel the grams and serving units.
3,600 sec
14
1 hr 14 grams per serving =
1 serving"
Therefore, we have
One box is 8 servings, and we use the conversion factor
3 metersper second =——>-=———_
.-=—== with "servings" in the numerator: 2S¢™"9s
Shor a:

2Aba We estimate that 28 grams is 1 ounce. We use the


rh
&51960
hr 289°
Therefore, we have
== kmperhour ste Sag eS Loz,
14 grams per serving 1seming 1box 28g
=104kmperhour.
_ F-8 oz
"28 box
= 4 ounces per box.
To convert from seconds to hours, we could first convert
from seconds to minutes, then from minutes to hours. To 176. To convert from centimeters per day to feet per week, we
do this, we must choose two conversion factors so that must cancel the centimeters and days units.
60 sec 60 min
all necessary units cancel: Tain d Phe 762 cm
762 centimeters per day = 1 day *
Therefore,
wehave 7 days equals 1 week. We use the conversion factor with
3meters
persecond 3mf001km_6tan
= ie 0sec6Ae
0min days" in the numerator: 7 week
To convert centimeters to feet, we first convert from
_360-60km centimeters to inches, then from inches to feet.
521900hr 254 centimeters equals 100 inches. We use the
6
_ 3:66°3km conversion factor with "centimeters" in the denominator:
5-450. hr
100 in
nee 254 cm*
=2 kmperhour 12 inches equals 1 foot, and we need to cancel the
=102kmperhour "inches" unit of the conversion factor chosen above.
So, we use the conversion factor with "inches" in the
174. To convert from gallons per day to fluid ounces per denominator: ———
minute, we must cancel the gallons and day units. 42in*
Therefore,wehave
45gallons
perday
=aa 762.ce17days|
762cmper day=—7a9 week Aes
1ft
12h
128 fluid ounces equals 1 gallon. We #20 the conversion
factor with "gallons" in the denominator: ie _027-3001
3
One day is 24 hours, and one hour is 60 minutes,
22e4-¥2week
so 24-60 = 1,440 minutes equals 1 day. We use the =175feetperweek.
:
conversion . ""day"uein the numerator: i =735d 4.
factor with

Therefore, we have

45gallons
perday
=san.N28:fliozs
oateiot 177. 2 bears can catch 30 salmon in 54 minutes.

One bear can catch half as many salmon in 54 minutes


_95-128
fl oz
gp PAOmin as 2 bears can. So, 1 bear can catch 5-30 = 15 salmon
in 54 minutes.
=4 fluidounces per minute.
One bear can catch one salmon in one fifteenth the
amount of time it takes to catch 15 salmon. So, 1 bear

To convert from days to minutes, we could first convert cancatch1salmon


in+ 54== 32minutes.
from days to hours, then from hours to minutes. To do singed
minute
is60hichire
33minutes
is
this, we must choose two conversion aoe so that all
necessary units cancel: day
77, and So min’
32-60
=216
seconds.

Beast Academy Practice 5C Ratios & Rates Chapter 8 Solutions


Since 1 minute is 60 seconds, 54 minutes. is Crawling 70 meters at 5 meters per second takes
54-60 = 3,240 seconds. 70+5= 14 seconds.

So, 2 bears can catch 30 salmon in 3,240 seconds. 181. The ratio of Jorble's height to Yorble's height is 5:9.

One bear can catch half as many salmon in 3,240 So, for some value of x, Jorble is 5x inches tall and
seconds as 2 bears can. So, 1 bear can catch 5-30 =o Yorble is 9x inches tall. The difference between their
salmon in 3,240 seconds. heights is 9x-5x = 4x inches.

One bear can catch one salmon in one fifteenth the


amount of time it takes to catch 15 salmon. So, 1 bear
WearetoldthatJorble is10inches shorter thanYorble,
can catch 1 salmon in —'3, 240 = 216 seconds.
sowehave4x=10.Solving forx,sg getx=oa=2.
Therefore, eas is9- =
Jorbie is 5:oie = =124 5 re
@
= =224
“=
inches tall, and
178. The ratio of Sarah's fruits to Jeremy's fruits isme5, and 5 =e —_—
together they have shes fruits. So, Sarah has @°135=60
=
fruits and Jeremy has alo
2 O47 ONTUILS: Check: The ratio of Jorble's height to Yorble's height is
124:224 =25:45=5:9. V
Sarah's ratio of rats to limes is 2:3, and she has 60
182. The ratio of the height of the rectangle to its perimeter
fruits. So, Sarah has = 60 = 24 lemons and 2. 60 = 36
limes. is 2:19. So, for some unit of length x, the rectangle's
perimeter is 19x units, and its height is 2x units.
Jeremy'sratioofpons tolimesis1:2,andnehas75
fruits.
So,Jeremy has=+75=25lemons and=.75=50 2
limes. Perimeter = 19x
When they combine their fruits into the same basket,
there are 24+25 = 49 lemons and 36+50 = 86 limes. Since the perimeter of a rectangle is the sum of its
Therefore, the ratio of SERS to limes is 49:86. four side lengths, the sum of the two unknown sides is
179. The ratio tells us Meta of
+ the perimeter comes from 19x-2x-2x = 15x units.
2° 4
theshortest
side,-4=+oftheperimeter
comes
3ze from
the3 Therefore, the width of the rectangle is Px units.

medium side, and 5 of the perimeter comes from the


longest side.

The perimeteris 60 inches, so the lengths . the three


sides of the isangle are +:60 = 15 inches, =3°60= 20 So, the ratio of the rectangle's height to its width is
inches, and 79°60=25
= inches.
2x:2x = 4x:15x. We divide both quantities by x to
The legs of a aa triangle are its two shortest sides, and simplify this to 4:15.
the area of a right triangle is equal to half the product of 183. We start by considering a simple example. Since the
the lengths of its legs. Therefore, the area of the triangle
ratio of Alyssa's biking speed to her running speed is 5:2,
is P20 = 150 sq in.
we imagine that Alyssa bikes 5 mph and runs 2 mph.
—or— If Alyssa's school is 20 miles away, it will take her
The ratio of the lengths of the three legs is 3:4:5. So, for 20+5=4 hours to bike to school, and 20+2 = 10 hours to
some number of inches x, the shortest side is 3x inches run to school. So, the ratio of her biking time to running
long, the medium side is 4x inches long, and the longest time is 4:10 = 2:5.
side is 5x inches long. There is nothing special about the numbers or units that
Since the perimeter of the triangle is 60 inches, we have we used in our example! Alyssa can bike any distance 5
times in the same amount of time it takes her to run the
3x+4x+5x =60
same distance 2 times.
12x =60.
If x is the time it takes for Alyssa to bike to school 5 times
So, x= 2s = 5. The lengths of the triangle sides are
or run to school 2 times, then the time it takes Alyssa to
3x =3-5=15 inches, 4x =4-5=20 inches, and bike to schoo! once is de, and the time it takes Alyssa to
5x =5:5=25 inches.
runtoschool
onceis3x.
The legs of a right triangle are its two shortest sides, and
So, the ratio of the time it takes her to bike to school to
the area of a right triangle is equal to half the product of the time it takes her to run to school is 4x to $x.
the lengths of its legs. Therefore, the area of the triangle
Dividing both quantities by x gives tx:dx= = . i.
iss 2:20 = 150 sq in.
Multiplying both numbers by the coe common
180. Since the tarantulemurs are crawling directly towards multiple
oftheirdenominators,
10,givesu5e4 =2:5.
each other, the tarantulemurs move 2+3=5 meters
closer to each other each second. When they meet, they
will have crawled a combined distance of 70 meters. The faster Alyssa goes, the less time it will take her to
get to school.

Ratios & Rates Chapter 8 Solutions Beast Academy Practice 5C


For example, if Alyssa travels twice as fast when biking
Since _ of all the seats are empty, the ratio of teachers to
than when running, it will take her half as long to get to
empty seats in the auditorium is <4 = 2:3.
school biking than it will running.
In other words, if the ratio of her biking speed to her Uy

running speed is 2:1, then the ratio of the time it takes to We can create a diagram to represent the seats in the

bike to school to the time it takes to run to school is 1:2. auditorium.

sitailarty, if Alyssa bikes 2 as fast as she runs, it will take The ratio of students to teachers is 5:1. So, we can make

her 5 as long to get to school. rows of 6 filled seats, each with 5 students and 1 teacher.
We use S for students, and T for teachers.
In other words, if the ratio of her biking speed to her
running speed is 5:2, then the ratio of the time it takes to $.S*S SeSsh
bike to school to the time it takes to run to school is 2:5. The ratio of filled seats to empty seats is 4:1. So, we
184. When 15 blue cars leave the lot, the ratio of blue cars to can make groups of 5 rows, each with 4 filled rows and
red cars changes from 2:3 to 1:2. 1 empty row. We use E in our diagram to represent an
empty seat.
Since the number of red cars in the parking lot does not
change, we write our ratios using the same number of S SrSes Smt
red cars. Since 6 is a multiple of 3 and of 2, we can write S:S$ 3 ST
each ratio as a ratio of blue cars to 6 red cars. Soo oo E
Before cars left: 2:3=4:6
5,525,0505°1
EEE BEE
After cars left: 1:2=3:6
So, we can make groups of 30 auditorium seats, each
When 15 blue cars leave, the ratio of blue cars to red with 20 filled by students, 4 filled by teachers, and 6
cars changes from 4:6 to 3:6. This tells us that, for each empty. Therefore, the ratio of teachers to empty seats is
blue car that leaves the parking lot, there are 6 red cars 4:6 = 2:3.
in the lot. Fifteen blue cars left, so there are 15-6 =90 186. It would be very complicated to think about how far
red Cars.
Brody travels before each time he changes direction.
However, we know his flying speed. If we know how long
The original ratio of the number of blue cars to red cars in he was flying, then we can use his speed to find the total
the lot is 2:3. So, for some number c, there were 2c blue distance he flew.
cars and 3c red cars in the lot. Since each skateboard is traveling toward the other, the
skateboards move 5+5 = 10 feet closer to each other
After 15 blue cars exit the lot, 2c-—15 blue cars and 3c
every second.
red cars remain. Since the ratio of blue cars to red cars
at that time is 1:2, we know there are twice as many red When they meet, they will have rolled a combined
cars as blue cars remaining in the lot. So, 3c is twice as distance of 100 feet.
much as 2c-—15. We write an equation to solve for c. Rolling 100 feet at 10 feet per second takes 100+10=10
3c =2:-(2c-15) seconds.

3c =4c-30 Since Brody flies 7 feet per second for 10 seconds before
the skateboards collide, he travels a total of 7-10 =70
30 =4c- 3c
feet.
30=Cc

So, there are 3c = 3-30 =90 red cars in the lot.

Check: Since there are 90 red cars and 60:90 = 2:3,


there were originally 60 blue cars in the lot. After 15 blue
cars leave the lot, 60-15 =45 remain, and the ratio of
blue cars to red cars is 45:90=1:2. Ww

185. The ratio of filled seats to empty seats is 4:1. So, - of the
seats in the auditorium are filled, and = of the seats are
empty.
The ratio of students to teachers is 5:1. So, 2 of the
filled seats hold students, and = of the filled seats hold
teachers.
2 of the seats in the auditorium are full, and teachers
have } 2
of these filled seats. So, -4 = 75 of the
auditorium seats are filled by teachers.

Beast Academy Practice 5C Ratios & Rates Chapter 8 Solutions


Ratios & Rates Chapter 8 Solutions Bpasicauom serene
We can write any number of thousandths as a decimal
eae with three digits to the right of the decimal point.
ues
1. 0.9 has aQ in the tenths place. So, 0.9 = =. 809 0.809.
So, 7,000=
2. 0.327 has a 3 in the tenths place, a 2 in the hundredths 10. We begin by writingthe fractionas amixed number.
place, and a 7 in the thousandaths place. So, 857_ g57
100 ~100°
De ae,
S l
0.327=+65+709+To00 Then, we can write any number of hundredths as a
28000 (20 7 decimal with two digits to the right of the decimal point.
1,000 " 1,000" 1,000
Mm a27 So>57
100
—0.57.
T herefore
GSTS
> 100
6 2657:
100
~ 7,000"

dh We have COPA
7.000 Pilger= 2.021.
000 = 27000
000

| We can write any decimal with three digits to the right of 12. We have 23" = 23.0071.
10,000
the decimal point as a number of thousandths.
_ 327 13. 1.43 is between 1.4 and 1.5. Since 1.43 is closer to 1.4
So, 0.327 = 7,000"
than to 1.5, we round 1.43 down to 1.4.
3. We can write any decimal with two digits to the right of
14. 0.368 is between 0.3 and 0.4. Since 0.368 is closer to 0.4
the decimal point as a number of hundredths.
than to 0.3, we round 0.368 up to 0.4.
So, 0.46 =
a 100 =50°
15. 77.7777 is between 77.77 and 77.78. Since 77.7777 is
4. We-can write any decimal with three digits to the right of closer to 77.78 than to 77.77, we round 77.7777 up to
the decimal point as a number of thousandths. 77.78.
ig es,
So, 6.128 =67 099= 125° 16. 1.245 is exactly halfway between 1.24 and 1.25.
Numbers that are exactly in the middle are rounded up.
5. Wehave27.014=27—4=27——
1,000 500° So, we round 1.245 up to 1.25.
zs _ 505". _101_ 17. 0.00317 is between 0.003 and 0.004. Since 0.00317 is
6. We have 505.0505 = 505 70,000 > 505 2,000
closer to 0.003 than to 0.004, we round 0.00317 down to
ie = is 3 tenths. So, to write = as a decimal, we write a 3 0.003.
in the tenths place: = =0.3.
18. 8.2497 is between 8.249 and 8.250. Since 8.2497 is
8. We have closer to 8.250 than to 8.249, we round 8.2497 up to

Dee00K. 3. 8.250.
700 ~ 100 * 100
19. For each number, we either place the decimal point
rat: gees
= 40 * 700" e two spaces to the left of a 6 that will remain a 6 when
the number is rounded down, or
| So, to write = as a decimal, we write 5 in the tenths
e two spaces to the left of a 5 that will become a 6 when
| place and 3 in the hundredths place: 23. = 0.53.
the number is rounded up.

Below, we show the only way to place each decimal


We can write any number of hundredths as a decimal point between two digits so that the hundredths digit of
with two digits to the right of the decimal point.
each rounded number is 6.
53 7 0:93:
So, 100 19.659 5.6565 4565.464
9. We have
Check: We round each answer to the nearest hundredth.
| g09 _ 800 ,_0 9
19.659 rounds to 19.66. YW
1,000 1,000 1,000. 1,000
| tt 85 gst ppt 5.6565 rounds to5.66“
= 40 + 400*7,000° 4565.464 rounds to 4565.46. V

809 ; : min 20. Let x represent the result Grogg gets after rounding his
So, to write 7000 25 2 decimal, we write 8 in the
decimal to the nearest hundredth.
tenths place, 0 in the hundredths place, and 9 in the
» 1809s
_ When Grogg rounds x to the nearest tenth, he gets 0.4.
thousandths place: 7,000 = 0.809.
The smallest number that rounds to 0.4 when rounded to
the nearest tenth is 0.35.

Beast Academy Practice 5C Decimals Chapter 9 Solutions


So, the smallest result Grogg can get after rounding his Alice ran 6.35-4.4 = 1.95 miles farther than Ben did on
decimal to the nearest hundredth is 0.35. The smallest the way out. Alice also ran 1.95 miles farther than Ben on
number that rounds to 0.35 when rounded to the nearest the way back.
hundredth is 0.345. So, Alice ran 1.95+1.95 =3.9 more miles than Ben.

Check: To the nearest hundredth, 0.345 rounds to 0.35. 27. We get the greatest possible difference by subtracting
To the nearest tenth, 0.35 rounds to 0.4. Any number the smallest number from the largest number. It is easier
smaller than 0.345 rounds to 0.34 or less when rounded
to order these decimals if we write them vertically with
to the nearest hundredth, which cannot round to 0.4. ¥
their place values aligned.
21. We stack the numbers vertically so that matching place 13/4
The largest number in this list is 13.1. 12.85
values are lined up. Then, we add the digits in each
The smallest number in this list is 0.078. 3.54
place value from right to left.
1 1 So, the greatest possible difference that 1.0065
2.35 PaGh5) 2.36
can be made by subtracting two of these 0.32
+ 6.91 + 6.91 + 6.91 numbers is 13.1-0.078 = 13.022. 0.23
6 26 9.26

22. We stack the numbers vertically so that matching place


values are lined up, filling the empty hundredths place of
12.4 with 0. Then, we add the digits in each place value 28. 0.8is8aie or=So,0.
8x10=2x«10=
ao
from right to left.
1 11 11
12.40 12.40 12.40 12.40
For any two adjacent place values, the place value
+ 9.73 + 9.73 + 9.73 + 9.73
on the left is ten times the place value to its right. So,
3 13 23 22.13
multiplying a number by 10 moves each digit to the
23. After stacking the numbers vertically with the place next-larger place value to its left.
values aligned, we see that we cannot take 5 hundredths
from 0 hundredths. So, we take 1 tenth from the tenths
place of 5.90 and break it into 10 hundredths. This gives
us 5 ones, 8 tenths, and 10 hundredths to subtract from.
8 10
5.98 This is the same as moving the decimal point one place
= ZS to the right.

0). 8 x10
Then, we subtract the digits ineach place value Hom
right to left. = 2028 |
8 10 8 10 8 10 Gy’
5.98 5.98 5.90
So, 0.8x10=8.
=12.00 = 2,00 = 2.00
29. 0.064 is 6hundredthsand 4 thousandaths,or 1,
——-~
See
5 25 3.25
To multiply this sum by 10, we distribute the 10 as shown
24. We stack the numbers vertically and subtract as shown.
below.
010 ow
B10 ow
#10 owi0 owt
10.720 10.460 10.728 10.408 10.728 0.064x
10=(B+aa) 10 ule 4

= 0008 . = "0,008Gi w0:008=. u20.0084) -—0,008


2 92 092 0.092 10.092
=(765%
y/o 10)+(35510)
a
25. To compute the perimeter, we add the three side lengths.
bs (Oita
=107700
1
4.30
=0.64.
6.45 So, 0.06410 = 0.64.
+ 9.90
20.65
Multiplying a number by 10 moves each digit to the
So, the perimeter of the triangle is 20.65 cm. next-larger place value to its left.
26. Alice ran 6.35 miles out, then another 6.35 miles back.
All together, she ran 6.35+6.35 = 12.7 miles.
Ben ran 4.4 miles out, then another 4.4 miles back.
All together, he ran 4.4+4.4 =8.8 miles.

So, Alice ran 12.7-8.8 =3.9 more miles than Ben.

Decimals Chapter 9 Solutions Beast Academy Practice 5C


This is the same as moving the decimal point one place 41. When we multiply the missing number by 10, the decimal
to the right. point moves one place to the right, giving 0.3. So, to find
the missing number, we move the decimal point in 0.3
one place to the /eft. This gives 0.03.
So, 0.03 10=0.3.
42. When we multiply the missing number by 100, the
So, 0.064 10 = 0.64.
decimal point moves two places to the right, giving 71.3.
30. Multiplying a number by 10 moves the number's decimal So, to find the missing number, we move the decimal
point one place to the right. So, 10x3.7 =37. point in 71.3 two places to the /eft. This gives 0.713.
31. Multiplying a number by 10 moves the number’s decimal So, 100x0.713 = 71.3.
point one place to the right.
43. When we multiply the missing number by 10,000, the
So, 0.901 10x10=9.01x10=90.1. decimal point moves four places to the right, giving 345.
So, to find the missing number, we move the decimal
point in 345 four places to the /eft. This gives 0.0345.
Multiplying a number by 10 once moves the number’s
decimal point one place to the right. So, multiplying a So, 10,000 x0.0345 = 345.
number by 10 twice moves the number's decimal point 44. To go from 0.02689 to 268.9, we move the decimal point
two places to the right. four places to the right. Multiplying a number by four 10’s
So, 0.901 x10x10=90.1. moves the number’s decimal point four places to the
32 right. Multiplying by four 10’sis the same as multiplying
Since multiplication is associative and commutative,
by 10x10x10x10= 10%.
10x21.032x 10 = 21.032 10x10. Multiplying a number
by 10 twice moves the number's decimal point two So, 0.02689 x 10* = 268.9. Therefore, n = 4.
places to the right. 45. To compute this quotient, we consider the relationship
So, 10x21.032x 10=21.0382x10x10=2,103.2. between multiplication and division.
33. 103.001 x10 =3.001x10x10=300.1. If0.043+10=[_|, then[_]x10=0.043.
34. Since 100 = 10x10, multiplying a number by 100 moves Since 0.0043 x 10=0.043, we have 0.043+10=0.0043.
the number’s decimal point two places to the right. Notice that dividing by 10 moves the decimal point one
Moving the decimal point in 12.3 = 12.30 two places to place to the left!
the right gives 1,230. 46. Multiplying 0.002 by 1,000 moves the decimal point in
So, 12.3x 100 = 1,230. 0.002 three places to the right, giving the integer 2:
35. Since 1,000 = 101010, multiplying a number by 1,000
0.002 x 1,000 = 2.
moves the number's decimal point three places to the
right. So, 1,000 x0.03405 = 34.05. The only positive integer smaller than 2 is 1. Since 1 is
half of 2, we can multiply 0.002 by half of 1,000 to get 1.
36. Multiplying a number by 1,000 moves the number's
decimal point three places to the right. So, Half of 1,000 is 500, so
0.00002x 1,000=0.02. 0.002500 = 1.
37. 10° is the product of five 10’s. So, multiplying a number
Since 1 is the smallest positive integer, 500 is the
by 10° moves the number's decimal point five places to
smallest positive integer we can multiply by 0.002 to get
the right. Therefore, 0.040608 x 10° = 4,060.8.
an integer result.
38. To go from 1.821 to 18.21, we move the decimal point
47. Multiplying 0.0025 by 10,000 moves the decimal point in
one place to the right. Multiplying a number by 10 moves
0.0025 four places to the right, giving the integer 25:
the number's decimal point one place to the right.
So, 1.821x10= 18.21. 0.0025 x 10,000 = 25.
39. To go from 0.007 to 7, we move the decimal point Both 10,000 and 25 are divisible by 25. So, multiplying
three places to the right. Multiplying a number by 0.0025 by x of 10,000 will give a result that is x of 25.
10x10x10= 1,000 moves the number's decimal point 1 -<10,000
_ ie eS
35of10,000is“55 =400,and56of25is55=1.So,
three places to the right.
So, 0.0071,000 = 7.
0.0025x400 = 1.
40. To go from 0.065 to 6.5, we move the decimal point two Since 1 is the smallest positive integer, 400 is the
places to the right. Multiplying a number by 10x10 = 100 smallest positive integer we can multiply by 0.0025 to get
moves the number’s decimal point two places to the right. an integer result.

So, 100x0.065 = 6.5.

Beast Academy Practice 5C Decimals Chapter 9 Solutions


52. Multiplying a number by 0.1 moves the number's decimal
point one place to the left. So, 12.34x0.1 = 1.234.
48. 5 ore
1.5 is 1+55; and 0.1 is 10° We use these fractions to 53. Multiplying a number by 0.1 moves the number’s decimal
compute 1.5x0.1 as shown below: point one place to the left.
So, 6.5x0.1x0.1 =0.65x0.1 = 0.065.

Multiplying a number by 0.1 once moves the number’s


decimal point one place to the left. So, multiplying a
number by 0.1 twice moves the number’s decimal point
two places to the left.
So, 1.5x0.1=0.15.
So, 6.5x0.1x0.1 =0.065.
54. Since multiplication is associative and commutative,
For any two adjacent place values, the place value 0.1x0.046x0.1 =0.046x0.1 0.1. Multiplying a number
on the right is one tenth the place value to its left. So, by 0.1 twice moves the number’s decimal point two
multiplying a number by 0.1 moves each digit to the places to the left.
next-smaller place value to its right.
So, 0.1x0.046x0.1 =0.046x0.1x0.1 =0.00046.
oer 0tects 100"
55. Multiplying 831 by 0.1 three times moves the decimal
x 0.1
point in 831 three places to the left.
So, 831 x0.1x0.1x0.1 =0.831.

This is the same as moving the decimal point one place 56. Dividing by a number is the same as multiplying by that
to the Jeff. number's reciprocal. The reciprocal of 10 is a 0.1, so
dividing by 10 is the same as multiplying by 0.1.
oe x 0.1 So, 0.0345+10=0.0345x0.1 = 0.00345.
57. Dividing by a number is the same as multiplying by that
number's reciprocal. Since 0.1 =4. the reciprocal of 0.1
So, 1.5x0.1 =0.15. is 10. So, dividing by 0.1 is the same as multiplying by 10.
49. 0.09 is aa and 0.1 is +. We use these fractions to e
So, 0.0067 +0.1 = 0.0067 x 10 = 0.067.
compute 0.10.09 as shown below: 58. (0.1)? =0.1x0.1. To compute this product, we move the
Ballenger
Oe decimal point in 0.1 one place to the left.
0.1x0.09=75x755
So, (0.1)? = 0.01.
59. (0.1)? =0.1 0.10.1. To compute this product, we move
the decimal point in 0.1 two places to the left.
So, 0.10.09 = 0.009. So, (0.1)? = 0.001.
60. We notice a pattern in the powers of 0.1.
Multiplying a number by 0.1 moves each digit to the (0.1)'=0.1 has 1 digit right of the decimal point.
next-smaller place value to its right. (0.1)? =0.01 has 2 digits right of the decimal point.
(0.1)? = 0.001 has 3 digits right of the decimal point.
OMNO):Seok Og Since each copy of 0.1 shifts the decimal point one place
~~ ~~OA to the left, we know this pattern continues. So, (0.1)®has
=O 009 6 digits right of the decimal point: 5 zeros, followed by a1.
Therefore, (0.1)®= 0.000001.
This is the same as moving the decimal point one place
to the /eft. In general, (0.1)” has n digits after the decimal point:
(n-1) zeros, followed by a 1.
OL09 x0.d 61. 2.3x(0.1)? is the product of 2.3 and two copies of 0.1.
= 0 009 So, we move the decimal point in 2.3 two places to the
left. 2.3x(0.1)? = 0.023.
So, 0.10.09 = 0.009. 62. Since 0.01 = 0.10.1, multiplying a number by 0.01
50. Multiplying a number by 0.1 moves the number's decimal moves that number's decimal point two places to the left.
point one place to the left. So, 0.1x8=0.8. So, 0.30.01 = 0.003.
51. 63. (0.1)?x 404 is the product of 404 and three copies of 0.1.
Multiplying a number by 0.1 moves the number's decimal
point one place to the left. So, 3400.1 =34.0=34. So, we move the decimal point in 404 three places to the
left: (0.1)°x 404 = 0.404.

Decimals Chapter 9 Solutions Beast Academy Practice 5C


64. Since 0.001 =0.1x0.1x0.1, multiplying a number by 74. We have
0.001 moves that number's decimal point three places to
the left. So, 6.7x0.001 = 0.0067.
65. Multiplying a number by (0.1)? moves that number's
decimal point two places to the left.
So, (0.1)?x6.05 = 0.0605.
66. Since 0.0001 =(0.1)*, multiplying a number by 0.0001 0.4x0.8 = (40.1) x(8x0.1)
moves that number's decimal point four places to the left. = (4x8)x(0.1x0.1)
So, 8,500x0.0001 = 0.8500 = 0.85. = 32x0.01
67. Multiplying a number by (0.1)®moves that number’s = 0.32.
decimal point six places to the left. 75. We have
So, (0.1)®x
4,000,000 = 4.000000 = 4.
68. Since 0.00001 = (0.1)5, multiplying a number by 0.00001
moves that number’s decimal point five places to the left.
So, 75.25x0.00001 = 0.0007525.
69. To go from 1.2 to 0.012, we move the decimal point two
places to the left. Multiplying a number by 0.01 moves
0.06x0.11 =(6x0.01)x (110.01)
the number’s decimal point two places to the left.
= (6x11)x(0.01x0.01)
So, 1.2x0.01 = 0.012. = 660.0001
70. To go from 312.5 to 0.3125, we move the decimal point = 0.0066.
three places to the left. Multiplying a number by 0.001 76. We have
moves the number's decimal point three places to the
left.
So, 0.001 x312.5 = 0.3125.
71. When we multiply the missing number by 0.001, the
decimal point moves three places to the /eft, giving
0.03456. So, to find the missing number, we move the
decimal point in 0.03456 three places to the right. This 0.05x0.3 =(5x0.01)x(3x0.1)
gives 34.56. = (5x3)x(0.01 x0.1)
So, 0.001 x34.56 = 0.03456. = 15x0.001
= 0.015.
72. When we multiply the missing number by 0.0001, the
77. We have
decimal point moves four places to the /eft, giving 0.25.
So, to find the missing number, we move the decimal =_6_ 12
point in 0.25 four places to the right. This gives 2,500.
0.006x0.12 =7495*799
wi tee
~ 100,000
So, 2,500x0.0001=0.25.
= 0.00072.

73. We write each number as a fraction, multiply, then 0.006x0.12 = (6x0.001)x(120.01)


convert back to decimal form:
= (6x 12)x(0.001 x0.01)
=72x0.00001
= 0.00072.

78. We have

0.07x0.80.002 aot
= 755*35*7900
yy oe
2 Se
Writing 0.2 as 2x0.1 and 0.3 as 3x0.1, we have ~ 4,000,000

= 0.000112.
0.2x0.3 = (2x0.1)x(3x0.1)
= (2x3)x(0.1 0.1) —Or—
=6x0.01
0.07 x0.8x0.002 = (7 x0.01) x(8x0.1) x(2x0.001)
= 0.06.
= (7x8x2)x(0.01 x0.1 0.001)
= 112x0.000001
= 0.000112.

Beast Academy Practice 5C Decimals Chapter 9 Solutions


Since 10.1 is a little more than 10, and 1.001 is a little
79. Writing
0.0049asa fraction,
wehave75ao:
oooSince more than 1, we expect 10.1 1.001 to be a little more
49 =7x7, and 10,000 = 100x100, we have
than 10x1=10. So, we place the decimal point in the only
0.0049
=a a place that gives a number that is a little more than 10.
ST,
~ 400x100 10.1101
i POT
=700
“00 83. 33.34and 333.4havea totalof 33.34 x333.4
=(5)
i 2- 2+1=3 digitsto the rightofthe 2 1
decimalpoint.
So,thesquare
ofa is0.0049.
Asadecimal,
os 0.07. So, we place the decimal point in 11115.556
11115556 so that there are 3 digits 3
We begin by writing 0.0049 as 49x0.0001. Since to the right of the decimal point.
49 =7x7 and 0.0001 =0.01 x0.01, we have ih Agipey
eves)
0.0049 = 49x0.0001

= (7X7)x(0.01 x0.01)
84. 1,428.57and 0.007 havea total 1,428.57 x 0.007
of 2+3=5 digitsto the rightof 2 3
= (7x0.01)x(7x0.01)
=0.07x0.07 the decimalpoint.
= (0.07). So, we place the decimal point in 9.99999
999999 so that there are 5 digits 5
So, the square of 0.07 is 0.0049. As a fraction, 0.07 = a. to the right of the decimal point.
80. The perimeter of the rectangle is 1 in, so 2(0.A+0.B) = 1.
Therefore, 0.A+0.B= 5 =0.5. 9.99999

Since A and B are digits, A+B must be 5. This gives the 85. 0.03 and 0.8 have a total of 2+1=3digits 0,03x0.8
following cee to the right of the decimal point. 2 1

¢ A=1,B=4 (orA=4, B=1): So, we move the decimal point in 0.024


In this case, the areasi the rectangle is 0.1x0.4=0.04 3x8 = 24 so that there are 3 digits to the 3
square inches. right of the decimal point.
° A=2,B=3 (orA=3, B=2): So, 0.03x0.8 = 0.024.
In this case, the area of the rectangle is 0.2x0.3 =0.06 86. 0.012 and 9 have atotal of 3+0 =3 digits 0.012x9
square inches.
to the right of the decimal point. 3
So, the largest possible area of the rectangle is 0.06
So, we move the decimal point in 0.108
square inches.
12x9=108 so that there are 3 digits to 3
81. 1.1 and 1.1 have a total of 1+ 1 =2 digits 11x11
the right of the decimal point.
to the right of the decimal point. 1 1
So, 0.012x9 =0.108.
So, we place the decimal point in 121 so do, 87. 0.002 and 0.0006 have a total of 3+4=7 digits to the
2
that there are 2 digits to the right of the right of the decimal point.
decimal point.
So, we move the decimal point in 2x6 = 12 so that there
are 7 digits to the right of the decimal point.
Since 1.1 is a little more than 1, we expect 1.11.1 to be
So, 0.002 x 0.0006 = 0.0000012.
a little more than 1x1=1. So, we place the decimal point
in the only place that gives a number that isa little more 88. 0.0404 and 0.07 have a total of 4+2=6 digits to the right
than 1. of the decimal point.
1.21 So, we move the decimal point in 4047 = 2,828 so that
there are 6 digits to the right of the decimal point.
82. 10.1 and 1.001 have a total of 10.1 x 1,001 So, 0.0404 x0.07 = 0.002828.
1+3=4 digits to the right of the 1 3 89. 0.2 and 0.5 have a total of 1+1=2 digits 0.2x0.5
decimal point. to the right of the decimal point. 1 1
So, we place the decimal point in 10.1101 So, we move the decimal point in 0.10
101101 so that there are 4 digits 4 2x5=10 so that there are 2 digits to the 2
to the right of the decimal point. right of the decimal point, including the
trailing zero. This gives 0.10.
After we have placed the decimal point, we can remove
the trailing zero. So, 0.2x0.5=0.10=0.1.

Decimals Chapter 9 Solutions Beast Academy Practice 5C


90. 0.06 and 0.25 have a total of 2+2=4 0.06 x 0.25 gives (15x 1)+(15x2) = 15+30=45 digits to the right of
digits to the right of the decimal point. 2 2 the decimal point, including trailing zeros.

So, we move the decimal point in 0.0150 To count the number of trailing zeros that are removed,
rYwrAY

6x25 = 150 so that there are 4 digits to 4 we count the zeros at the end of 6'°x5'°. The product
the right of the decimal point, including the of 15 copies of 6 and 15 copies of 5 is the same as the
trailing zero. This gives 0.0150. product of 15 copies of 6x5. Since 6x5 =30 has 1 zero,
the product of 15 copies of 6x5 has 15 zeros.
After we have placed the decimal point, we can remove
the trailing zero. So, 0.06x0.25 = 0.0150 = 0.015. Therefore, (0.6)'®x(0.05)'> has 45-15 =30 digits to the
right of the decimal point.
91. 0.075 and 0.8 have a total of 3+ 1 =4 digits to the right of
the decimal point.
(0.6)'®x (0.05)'* is the product of 15 copies of 0.6 and 15
So, we move the decimal point in 75x8 = 600 so that
copies of 0.05. We can pair each 0.6 with a 0.05 to get
there are 4 digits to the right of the decimal point,
the product of 15 copies of 0.6x0.05 = 0.030 = 0.03.
including the trailing zeros. This gives 0.0600.
So, (0.6)'®x(0.05)'® = (0.03)"°.
After placing the decimal! point, we can remove the
trailing zeros: 0.075 x0.8 = 0.0600 = 0.06. Since 0.03 has 2 digits to the right of the decimal point
92. with no trailing zeros, the product of 15 copies of 0.03
0.00125 and 0.032 have a total of 5+3=8 digits to the
has 15x2 = 30 digits to the right of the decimal point with
right of the decimal point.
no trailing zeros.
So, we move the decimal point in 125x32 = 4,000 so
Therefore, (0.6)'*x(0.05)'* = (0.03)'> has 30 digits to the
that there are 8 digits to the right of the decimal point,
right of the decimal point.
including the trailing zeros. This gives 0.00004000.
96. Since 0.7 and 0._] have a total of 2 digits to the right
After placing the decimal point, we can remove the
of the decimal point, and their product 0.5[_] also has 2
trailing zeros: 0.00125
x0.032 = 0.00004000 = 0.00004.
digits to the right of the decimal point, we can ignore the
93. The numbers in 0.9x0.8x0.7x0.6x0.5x0.4x0.3x0.2x0.1 decimal points and consider the following equation:
have a total of 9 digits to right of the decimal point. So,
7xH=5i.
there are 9 digits to the right of the decimal point in the
product. However, some of those digits may be trailing 8 is the only number we can multiply by 7 to get a
zeros, which we do not count. product in the 50’s.

The number of trailing zeros is equal to the number of Since 7x8 =56, we have
zeros at the end of 9!=9x8x7x6x5x4x3x2x1. 0.7x0{8]= 0.5/6].
The number of zeros at the end of 9! is given by the 97. Since 10.L] and 0.L] have a total of 2 digits to the right
largest power of 10 that is a factor of (9!). We can pair of the decimal point, and their product 3._]6 also has 2
a2anda5in9x8x7x6x5x4x3x2x1 to make a 10. digits to the right of the decimal point, we can ignore the
Since there are no more 5’s, we cannot make any more decimal points and consider the following equation:
10’s. So, 10' is the largest power of 10 that is a factor of 10L]xLJ= 36.
(9!). Therefore, there is 1 zero at the end of (9!).
3 is the only digit we can multiply 10[_] by to get a product
So, the product 0.9x0.8x0.7x0.6x0.5x0.4x0.3x0.2x0.1 in the 300’s. So, we have 10[_]x([3]=3J6.
has 9-1 =8 digits to the right of the decimal point.
Then, the missing units digit in 10[_] must be 2 in order to
In fact, 9! = 362,880, and get a units digit of 6 in 3L]6. This gives 10[2]x(3] = 3[0]6.
0.90.8 0.7x0.6x0.5x0.4x0.2x 0.1
= 0.00036288.
Therefore, we have
94. 0.3 has 1 digit to the right of the decimal point, and 0.07
10[2]x0[3] = 3.[0]6.
has 2 digits to the right of the decimal point.
98. All together, 0.[_]x0.2 has 1 more digit to the right of
So, multiplying 15 copies of 0.3 and 15 copies of 0.07
the decimal point than the product 0.L]. To make the
gives (15x 1)+(15x2) = 15+30 = 45 digits to the right of
number of digits to the right of the decimal point on both
the decimal point, including any trailing zeros.
sides of the equation match, we can write the equation
Since neither 3 nor 7 has factors of 2 or 5, no product as 0.L]x0.2=0.Jo.
of 3’s and 7’s will have trailing zeros. So, there are no
Then, we can temporarily ignore the decimal points and
trailing zeros to remove after the decimal point.
consider the equation []x2 =[]o.
Therefore, (0.3)'°x(0.07)'® has 45 digits to the right of the
5 is the only digit that can fillthe blank in _]x2 to give a
decimal point. product that ends in zero. Since [5]x2 =[1]0, we have
95. 0.6 has 1 digit to the right of the decimal point, and 0.05
0:5x0/2=0:10=0:1-
has 2 digits to the right of the decimal point.
So, multiplying 15 copies of 0.6 and 15 copies of 0.05
We fillin the blanks as shown:
0{5]x0.2 = 0[1].

Beast Academy Practice 5C Decimals Chapter 9 Solutions


Then, we can temporarily ignore the decimal points and
99: All together, 0.6x2._] has 1 more digit to the right of
consider the equation ]5x[_]= 300.
the decimal point than the product L].L]. To make the
number of digits to the right of the decimal point on both The only factor pairs of 300 that include a number with
sides of the equation match, we can write the equation units digit 5are 5x60, 15x20, 25x12, and 75x4. Of
as 0.6x2._J=L1.L0. these, only 754 can be used to fill in the missing digits.

Then, we can temporarily ignore the decimal points and Since [7]5x[4]= 300, we have
consider the equation 6x2[_]=[ | Jo. 0.75x0.04 = 0.0300 = 0.03.
For the product of 6 and another integer to end in zero, We fillin the blanks as shown.
the integer must be a multiple of 5, and therefore ends in
0.[7]5x0.0/4] =0.03.
0 or 5. However, numbers in this problem cannot have
trailing zeros, so the integer we multiply by 6 must end . All together, 0.3[.]x0.0L] has 1 more digit to the right of
in 5. Since 6x2[5]=[1]510, we have the decimal point than the product 0.007. To make the
number of digits to the right of the decimal point on both
0.6x2.5=1.50=1.5.
sides of the equation match, we can write the equation
We fill in the blanks as shown. as 0.3L_]x0.0[_]= 0.0070.
0.6 x 2.[5]=(1].5). Then, we can temporarily ignore the decimal points and
100. All together, 0..]x4.L] has the same number of digits to consider the equation 3L]xL]=70.
the right of the decimal point as the product 0._]3. So, we The only factor pair of 70 that includes a number with
can temporarily ignore the decimal points and consider
the following equation:
LIx4LJ=Lis.

If the digit in the first blank is 3 or greater, then the


tens digit 3 is 35x2. Since [3]5x[2]= 70, we have
0.35x0.02 = 0.0070 = 0.007.
We fillin the blanks as shown.
0.3[5] x0.0[2]=0.007.
ae
product will be 3x40 = 120 or greater. However, the
product is a two-digit number. So, the digit in the first . Alltogether, 0] [ ]x0.J has 3 more digits to the right
blank is 1 or 2. of the decimal point than the product 0.7. To make the
number of digits to the right of the decimal point on both
If the digit in the first blank is 2, then the product will
sides of the equation match, we can write the equation
be even. However, [_]3 has units digit 3, so it is odd.
as 0.LT | ]x0.LJ=0.7000.
Therefore, the digit in the first blank is 1.
Then, we can temporarily ignore the decimal points
(x4. J=L13. and consider the equation L[_| ]xL]=7,000. Since
Then, the digit in the second blank must be 3, and the 1,000 x7 = 7,000, the one-digit number in the equation
digit in the final blank must be 4. LIT _]x[]=7,000 must be greater than 7 in order for the
[1]x4/3]
=[4]s. other missing number to have fewer than four digits. So,
the one-digit number is either 8 or 9. Since 9 is nota
We fill in the blanks as shown.
factor of 7,000, the one-digit number must be 8.
0[1)x4[3]=0[4/3.
So, we have [ | | ]x[8]=7,000. Therefore, the missing
101. All together, 0.0L_]x0.0L_]has 1 more digit to the right of
three-digit number is 7,000+8 = 875.
the decimal point than the product 0.003. To make the
Since [8[7[5]x[8]=7,000, we have
number of digits to the right of the decimal point on both
sides of the equation match, we can write the equation 0.875x0.8 =0.7000 = 0.7.
as 0.0_]x0.0[_]= 0.0030.
We fillin the blanks as shown.
Then, we can temporarily ignore the decimal points and 0.[8[715]x0 [8] = 0.7.
consider the equation [_]x[_]= 30. There is only one pair
of digits with product 30: [5]x[6]= 30. whePsa
We consider the equation _]_|_]xL]= 7,000. The prime
So, we have
factorization of 7,000 is 2°x5°x7. Since numbers in this
0.05 x0.06 = 0.0030 = 0.003. problem cannot have trailing zeros, neither of the missing
We fill in the blanks as shown. numbers in the product [[ | |x] can have a factor of 2
0.0[5]x 0.0[6] =0.003. and a factor of 5. This gives the following possibilities:
(2°x7)x(5°) or (5°x7)x(2°).
Sincemultiplication
iscommutative,youmayhave
insteadwritten0.0[6]x
0.0[5]=0.003. Only the product (5*x7) x (2°) includes a one-digit
102. All together, 0._]5x0.0L] has 2 more digits to the right number: 2°= 8.
of the decimal point than the product 0.03. To make the So, we have (5°x7)x(2°) =8758 = 7,000. Therefore,
number of digits to the right of the decimal point on both 0.875x0.8 = 0.7000 =0.7.
sides of the equation match, we can write the equation
as 0.L]5x0.0L_]= 0.0300. We fillin the blanks as shown.
0.[8[715]x0 [8]=0.7.

Decimals Chapter 9 Solutions Beast Academy Practice 5C


105. All together, 1..]x0./T [J has 2 more digits to the right Multiplying by 300 shifts the decimal point right 2 places.
of the decimal point than the product 0.04. To make the Multiplying by 0.03 shifts the decimal point left 2 places.
number of digits to the right of the decimal point on both Multiplying by 0.003 shifts the decimal point left 3 places.
sides of the equation match, we can write the equation
All together, we shift the decimal point in 27 left 3 places.
as 1...)x0.[71 ]=0.0400. So, 3000.03 x0.003 = 0.027.
Then, we can temporarily ignore the decimal points and 110. We begin by computing 4x 12x1=48. Then we
consider the equation 1L]x(_] | ]=400.The number 1[]
determine where to place the decimal point.
is at least 10 and at most 19, and is also a factor of
400. The only numbers from 10 to 19 that are factors Multiplying by 40 shifts the decimal point right 1 place.
of 400 are 10 and 16. However, 1[_]cannot be 10 since Multiplying by 0.012 shifts the decimal point left 3 places.
numbers in this problem cannot have trailing zeros. Multiplying by 1,000 shifts the decimal point right 3 places.

So, we have 1[6]x[_[ | ]=400. Therefore, the other All together, we shift the decimal point in 48 right 1 place.
So, 40x0.012x 1,000 = 480.
missing number is 400+ 16 = 25. Since 25 has only two
digits, we write a leading 0 in the hundreds place.
Since 1[6]x(0]2[5]= 400, we have We see that 0.012x 1,000 = 12.
So, 40x0.012x 1,000 = 40x 12 = 480.
1.6x0.025 = 0.0400 = 0.04.
111. We begin by computing 25x 4x2 = 200. Then we
We fill in the blanks as shown.
determine where to place the decimal point.
1 [6)x 0.[01215]= 0.04.
Multiplying by 0.025 shifts the decimal point left 3 places.
106. Since 200 = 2x 100, multiplying by 200 is the same as Multiplying by 0.04 shifts the decimal point left 2 places.
multiplying by 2, then shifting the decimal point in the Multiplying by 2,000 shifts the decimal point right 3 places.
product 2 places to the right.
All together, we shift the decimal point in 200 left 2 places.
Similarly, since 0.009 = 90.001, multiplying by 0.009 is So, 0.025x0.04x2,000 = 2.
the same as multiplying by 9, then shifting the decimal
point in the product 3 places to the /eft.
Since multiplication is commutative and associative,
So, to compute 200x0.009, we can multiply 2x9 = 18,
then shift the decimal point 2 places to the right and 3 0.025 x0.04x 2,000 = 0.04x (0.025 x2,000).
places to the left. This is the same as shifting the decimal We see that 0.025x2,000 = 50.
point 1 place to the left: So, 0.04 (0.025 x2,000) = 0.04x50=2.
1.8 112. (300)'°x(0.002)"° is the product of 10 copies of 300 and
wy
So, 200 x0.009 = 1.8. 10 copies of 0.002.
107. Since 0.04 =4x0.01, multiplying by 0.04 is the same as Multiplying by 300 shifts the decimal point right 2 places.
multiplying by 4, then shifting the decimal point in the So, multiplying by 10 copies of 300 shifts the decimal
product 2 places to the /eft. point right 2x 10= 20 places.

Similarly, since 13,000 = 13x 1,000, multiplying by Multiplying by 0.002 shifts the decimal point left 3 places.
13,000 is the same as multiplying by 13, then shifting the So, multiplying by 10 copies of 0.002 shifts the decimal
decimal point in the product 3 places to the right. point left 3x 10= 30 places.

So, to compute 0.04x 13,000, we can multiply 4x 13 = 52, All together, we shift the decimal point in the product left
then shift the decimal point 2 places to the left and 10 places. So, to compute (300)'°x(0.002)'°, we multiply
3 places to the right. This is the same as shifting the 3'°x2'°, which equals some integer that does not end in
decimal point 1 place to the right: zero. Then, we shift the decimal point 10 places to the
left, which gives a product with 10 digits to the right of the
520.
decimal point.
So, 0.04 13,000 = 520.
In fact, (300)'° x(0.002)'° = 0.00604661 76.
108. Multiplying by 0.00025 is the same as multiplying by 25,
then shifting the decimal point 5 places to the left.
(300)'°x(0.002)"° is the product of 10 copies of 300 and
Multiplying by 800 is the same as multiplying by 8, then
10 copies of 0.002. We can pair each 300 with a 0.002
shifting the decimal point 2 places to the right.
to get the product of 10 copies of 300x0.002 = 0.6. So,
So, to compute 0.00025 x 800, we multiply 25x8 = 200, (300)'°x (0.002)'° = (0.6)'°.
then shift the decimal point 3 places to the left.
Each time we multiply by 0.6, the decimal point in the
So, 0.00025x800= 0.2. product shifts 1 place to the left. So, the product of 10
109. We begin by computing 3x3x3= 27. Then we determine copies of 0.6 shifts the decimal point in the product
where to place the decimal point. 1x10=10 places to the left.

Beast Academy Practice 5C Decimals Chapter 9 Solutions


So, to compute (300)'°x (0.002)'° = (0.6)'°, we compute
6'°, which equals some integer that does not end in zero. After working for 6 hours, Timmy will earn 12.25x6
Then, we shift the decimal point left 10 places, which dollars. We use the distributive property:
gives a product with 10 digits to the right of the decimal
point. 12.25x6 =(12+0.25)x6
"In fact,(0.6)"°
=0.0060466176. = (12x6)+(0.25x6)
113. (400)'°x(0.0025)"° is the product of 10 copies of 400 and
=72+1.5
10 copies of 0.0025.
=73.5.
' Multiplying by 400 shifts the decimal point right 2 places. So, Timmy will earn $73.50.
So, multiplying by 10 copies of 400 shifts the decimal 118. We consider the computation using fractions.
point right 2x 10= 20 places. eS teat myaee RP KC
0.125 = 1,000 8’ and 0.01= 700"So, we have 3%4= 99°
Multiplying by 0.0025 shifts the decimal point left 4
places. So, multiplying by 10 copies of 0.0025 shifts the eral ae
Therefore,a= 70078"
decimal point left 4x 10= 40 places.
Dividing by a number is the same as multiplying by that
All together, we shift the decimal point in the product number's reciprocal. The reciprocal of Z is 8.
left 20 places. So, to compute (400)'°x (0.0025)'°, we
Se
Therefore, a=700% Ome
SE 100 ee
multiply 4'°x25"°, then shift the decimal point 20 places
to the left. As a fraction, we write a= 700 = 25"

Since 4x25 = 100, we can pair ten 4’s with ten 25’s to get
4'°x25'° = (4x25)'° = 100". Each copy of 100 in 100'°
We know that 125x8 = 1,000. Since 0.125 has 3 digits to
adds two trailing zeros to the product. So, 100'° equals the right of the decimal point, 0.125x8 = 1.
1 followed by 2x 10 = 20 trailing zeros:
However, the product we seek is 0.01. To get 0.01, we
100,000,000,000,000,000,000. move the decimal point in 8 two places to the left. This
Then, shifting the decimal point left 20 places gives us 1. gives 0.125x0.08 = 0.01.

So, (400)'°x(0.0025)'°= 1. So, a=0.08.


As a fraction, we write a= 8-2
100° 25°

(400)'°x(0.0025)"° is the product of 10 copies of 400 and 119. Rather than computing the areas of the two rectangles
10 copies of 0.0025. We can pair each 400 with a 0.0025 separately, then adding the results, we notice that each
to get the product of 10 copies of 400x0.0025. So, rectangle has a side with length 2.1 m. We can join the
(400)'°x(0.0025)'° = (400x0.0025)'°- two rectangles along this side to create a larger rectangle
Since 400 x0.0025 = 1, we have (400x0.0025)'°= 11° =1. with the same area as both smaller rectangles combined.

114. To compute the height of a stack of 40 bricks in which 4.473m 3.527m


each brick is 0.64 centimeters tall, we multiply 40x0.64.
Multiplying by 40 shifts the decimal point right 1 place.
Multiplying by 0.64 shifts the decimal point left 2 places.
Pafi Se et
So, to compute 400.64, we multiply 4x64 = 256, then This larger rectangle has height 2.1 m and width
shift the decimal point a total of 1 place to the left. 4.473+3.527 =8 m. So, its area is 2.1x8 = 16.8 sq m.
So, the height of the 40-brick stack is 40x0.64 = 25.6 cm.
Check: Since 0.64 is a little more than half, we expect
400.64 to be a little more than half of 40. ¥ 120.

115. contains a 1, each block in the middle row contains a 2,


The volume of a cube with 0.4-cm edges is 0.4x0.40.4.
and each block in the bottom row contains a 3. So, every
To compute this product, we multiply 4x4x4=64, then
path has a product of 1x2x3=6.
move the decimal point 3 places to the left.
Therefore, the volume of the cube is The product we seek is 0.06. To go from 6 to 0.06,
0.4x0.4x0.4 = 0.064 cubic cm. we move the decimal point two places to the left. The
direction in which we move the decimal point and the
116. Since 1 inch is 2.54 centimeters, 0.3 inches is 0.3x2.54
number of times we move it is determined by the number
centimeters.
of trailing zeros and the number of digits after the
Since 3x254 = 762, we have 0.3x2.54 = 0.762. So, a decimal point in each path.
0.3-inch tall gentlebug is 0.762 centimeters tall. We consider the placement of the decimal point for
117. After working for 6 hours, Timmy will earn 12.25x6 each path, then circle the numbers in the correct path as
dollars. Since 1,225x6 =7,350, we have 12.25x6=73.5. shown.
So, Timmy will earn $73.50.

Decimals Chapter 9 Solutions Beast Academy Practice 5C


124. Ignoring decimal points and trailing zeros, every path
in this pyramid has a product of 1x7x1113= 1,001.
To go from 1,001 to 0.1001, we move the decimal point
4 places to the left. The path below is the only path in
which the decimal point moves a total of 4 places left.

0.1001
0.1x2x3=06 0.1x2%0.3=0.06
0.06 0.06

10x0.7%1.1x0.013=0.1001
0.1x0.2x0.3 =0.006 0.1x0.2x0.03 =0.0006 125. Ignoring decimal points and trailing zeros, every path in
this pyramid has a product of 2x5x2x5=100. To go
121. Ignoring decimal points and trailing zeros, each block in
from 100 to 0.01, we move the decimal point 4 places
the pyramid contains a 2. So, every path has a product of
to the left. The path below is the only path in which the
2Xx2x2=8:
decimal point moves a total of 4 places left.
The product we seek is 0.008. To go from 8 to 0.008,
0.01
we move the decimal point 3 places to the left. The path
below is the only path in which the decimal point moves
a total of 3 places left.
0.008

2 |)
| 20fo.c2
| @) 0.2x0.5x2x0.05 =0.0100

0.2x0.02x2
=0.008 126. Ignoring decimal points and trailing zeros, every path in
122. Ignoring decimal points and trailing zeros, every path in this pyramid is a product of 2’s and 3’s, so the path must
have a product of 24. To go from 24 to 0.024, we move
this pyramid has a product of 2x3x5=30. To go from
the decimal point 3 places to the left. The path below is
30 to 0.3, we move the decimal point 2 places to the left.
the only path in which the decimal point moves a total of
The path below is the only path in which the decimal
3 places left.
point moves a total of 2 places left.
0.024
0.3

ee
ie
ce]os[6
123.
0.2x30x0.05 = 0.30 ooo]
02[CS]
0.02x0.2x2x3=0,024
Ignoring decimal points and trailing zeros, every path
in this pyramid has a product of 3x3x3x3=81. To go 127. Ignoring decimal points and trailing zeros, every block in
from 81 to 8.1, we move the decimal point 1 place to the
this pyramid contains a 1, a 3, or a 4. So, to get a product
left. The path below is the only path in which the decimal of 12, we need exactly one 3 and exactly one 4. There
point moves a total of 1 place left. are only four paths with one 3 and one 4, as shown.
8.1
0.012

30x3x0.3x0.3=8.1
Beast Academy Practice 5C Decimals Chapter 9 Solutions
Then, to go from 12 to 0.012, we move the decimal point 131. Since 0.068 is about 0.07 and 297.4 is about 300, we

3 places to the left. Among the four paths, the only one in estimate that 0.068 x297.4 is about 0.07 x300 = 21.

which the decimal point moves a total of 3 places left is Among our choices, only 20.2232 is close to 21.
shown below.
2.2032 30.2232 180.2232 202.232
0.012

132. 0.9594 is close to 1. So, we estimate the value of each


answer choice and circle the one that is close to 1.

° 2.46x0.039 ~2.5x0.04 = 0.1.

¢ 0.82x11.7=1x10=10.

¢ 0.018x5.33 ~0.02x5=0.1.
¢ 0.234x4.1=0.25x4=1.
0.1x0.03x0.1x40=0,012
Among these choices, only 0.2344.1 is close to 1.
128. Ignoring decimal points and trailing zeros, every block
in the pyramid contains a 1, a 24, or a 25. The only 2.460.089 0.82x11.7 0.018x5.33
nonzero digit in the product we seek is 6. We can only
get a product whose only nonzero digit is 6 by multiplying 133. We estimate each product.
24x25 = 600. So, we look for paths with exactly one 24 ¢ 50.03x0.0819 =50x0.08 = 4.
and exactly one 25. There are four such paths:
° 0.68x0.392 =0.7x0.4=0.28.

0.6 0.6 ¢ 0.01207 x42.9 =0.012x40 =0.48.

° 0.054x 102.29 =0.05x100=5.

We use these estimates to connect each product with its


location on the number line, as shown below.

50.03x0.0819 0.680.392 0.01207 x 42.9 0.054 x 102.29

0 1 2 3 4 5 6 iti 8 9

134. 0.9914 is less than 1, so 0.9914x0.52 is less than


Then, to go from 600 to 0.6, we move the decimal point
1x0.52=0.52.
3 places to the left. Among the paths shown above, the
only one in which the decimal point moves a total of 3
places left is shown below.
0.9914x0.52
«<)0.52
135. 3.911 is less than 4, and 0.049 is less than 0.05.
So, 3.911x0.049 is less than 4x0.05 = 0.2.

3.911x0.049
<) 0.2
136. 9.008 is greater than 9, and 0.7134 is greater than 0.7.
So, 9.008 x0.7134 is greater than 9x0.7 =6.3.

9.008x0.7134
(>)63
137. 98.05 is less than 100, so 98.05x0.00852 is less than
100 x0.00852 = 0.852.

In other words, 0.852 is greater than 98.05x0.00852.


0.852 (>) 98.5 0.00852
Fortheniblewebelow,youmayhaveisedaifarent
138. 0.029 is less than 0.03, and 0.38 is less than 0.4.
estimates
thantheonesgiven
toarrive
atthesamefinal So, 0.029 0.38 is less than 0.03x0.4 =0.012.
answers. 2.05 is greater than 2, and 0.0061 is greater than 0.006.
129. Since 0.0198 is about 0.02, and 407.6 is about 400, we
estimate that 0.0198x407.6 is about 0.02x400=8. So, 2.05x0.0061 is greater than 2x0.006 = 0.012.

So, 0.0198407.6 is closer to 8 than to 80. Since 0.029x0.38<0.012, and 2.05x0.0061>0.012, we


have
130. Since 523.9 is about 500, and 0.0031 is about 0.003, we
estimate that 523.9x0.0031 is about 500x0.003 = 1.5.
0.029x0.38
«) 2.05x0.0061.
So, 523.9x0.0031 is closer to 1.5 than to 15.

Decimals Chapter 9 Solutions Beast Academy Practice 5C


139. 3.208 is greater than 3, and 0.768 is greater than 0.7.
So, 3.208 x0.768 is greater than 3x0.7 =2.1.
4.95 is less than 5, and 0.3809 is less than 0.4.
So, 4.95x0.3809 is less than 5x0.4=2.
144. Multiplying the numerator and denominator of B 3by2
gives an equivalent fraction with denominator 100.
Since 3.208 x0.768>2.1, and 4.95 x0.3809<2, we have x2

—_—
3 26
3.208x0.768 (>) 4.95x0.3809.

140. 0.0625 is greater than 0.06, and 48 is greater than 40.


So, 0.0625 48 is greater than 0.06x40 = 2.4. SO,513_26
0=7400
=0:26.
0.0625 is less than 0.07, and 48 is less than 50. 145. Multiplying the numerator and denominator of 3 by 25
So, 0.062548 is less than 0.07x50 =3.5. gives an equivalent fraction with denominator 100.
So, 0.0625 x48>2.4 and 0.0625x48<3.5. Since the x25
ae
problem tells us that 0.0625 x48 is equal to an integer,
that integer must be 3.
Alo 400
x25
3_ 75
0.0625 is about 0.06, and 48 is about 50. So, we
So,7=100=0-75.
146. Multiplying the numerator and denominator of 2 by 2
estimate that 0.062548 is about 0.06 x50 =3. Since the
problem tells us that 0.062548 is equal to an integer, gives an equivalent fraction with denominator 10.
that integer must be 3. x2

141. 4.104 is greater than 4, and 5.724 is greater than 5.


So, 4.104x5.724 is greater than 4x5 = 20.
Also, 4.104 is less than 5, and 5.724 is less than 6.
So, 4.104x5.724 is less than 5x6 =30.

Therefore, 4.104x5.724 is between 20 and 30. So, there


are 7 multiples of 10 between 4.104*5.724 and 99: Wewrite2 as the mixedmae 15 S. Multiplying
the
30, 40, 50, 60, 70, 80, and 90.
ridim@rater
and denominatorof33 by2 givesan equivalent
fractionwithdenominator10.
142. We estimate each product.
x2
¢ 0.028x7.03~0.03x7=0.21.
hepetial:
° 0.982x0.41 =1x0.4=0.4.
5 10
¢ 0.621.571 ~0.6x1.5=0.9. x2
° 0.76x0.259 =0.8x0.25 = 0.2.
Te ce
So, F=15=155=1.6
Among these estimates, only 0.028 7.03 =0.21 and 147. Multiplying
the numeratorand denominatorof &
5 bY4,
0.76x0.259 ~ 0.2 are close. So, we circle these two Teruo s.0.28.
we have==
55-1007
products as shown below.
148. Multiplying
the numeratorand denominatorof 2+ by5;
0.028x7.03) 0.982x0.41 0.62x1.571 ©.76x0.259 1
we have=- _255_ 555.
50 100=222 =2.55.
In fact, 0.028 x 7.03 = 0.76 0.259 = 0.19684. 149. Neither 10 nor 100 are divisible by 8. However,
143. Since 0.0058 is close to 0.006, and 21.3 is close to 20, 1,000+8 = 125. So, multiplying the numerator and
denominator of 2g by 125, we have 5 b2e = 0.625.
Ralph’s answer should be close to 0.00620 =0.12.
8 1,000
The answer Ralph got was 1.2354, which is about 10 150.
times bigger than our estimate of 0.12. So, Ralph’s
Multiplying
theRROlar anddenominator
of==
73 by8,
answer is not reasonable.
wehaveaaa =~
7000
a = 0.072.
151.
In fact, 0.0058 21.3 = 0.12354. So, Ralph’s answer was Multiplying
thenumerator
anddenominator
of&5bY2a)
off by an entire place value. This could have happened wehave&
40
=—2=
1,000
90.075.
by placing a decimal point in the wrong spot, or by
forgetting one of the O’s after the decimal point in 0.0058. 152. Wewrite
= asthemixed
Lillie4hMultiplying
the
These are easy mistakes to make! This is why it’s always numerator and denominator of 34 bY125, we have
a good idea to check the reasonableness of your answer, Ao=41 =4.125.
125
even when using a calculator!

Beast Academy Practice 5C Decimals Chapter 9 Solutions


153. We write seem as an equivalent fraction with a 156. We use the following steps to write i as a decimal.
denominator that is a power of 10.

Since 2°9x5'' is the product of 99 twos and 101 fives, 0.02 0.02|0.025
0.005
we can pair each of the 99 twos with a five to get 99 tens.
_. Then, we have two 5’s left over.
40)1.00 40)1.00
26Boe 40)1.00
uneOI80
299 x 5101 = 299 x 599 x 52 0.20 0.200
-0.200
= 10%°x5?.
0
[eR et
So,io" +40=0.025.
In order to make a power of 10, we need two more 2’s Check: 40x0.025 = 1.00=1. YW
to pair with the remaining two 5’s. So, we multiply the
157. We use the following steps to write 2 as a decimal.
numerator and denominator of om by 22.
x2?
0.0 ve
13.7 ™ 13x22 07
999%.5101 9101 5101
0.8 0.8 0g [1-875
x2 1.0 1.0 1.0 1.0
8)15.0 8)15.0 8)15.0 8)15.0
In the numerator, we have 13x2?=13x4=52.
-8.0 -8.0 -8.0 -8.0
In the denominator, we have 21° x 5101 = 10101.
S 13 S852
7.0 7.0 7.0 7.0
O, 999x 5101Sr 10101" -6.4 -6.4 =6.4
6 0.60 0.60
When Ta is written as a decimal there are 101 digits to -0.56 -0.56
the right of the decimal point: 99 zeros, followed by the 0.04 0.040
digits 5 and 2. So, the sum of the digits to the right of the -0.040
decimal point is 5+2=7.
0
154. To compute <= 4=5 using long division, we 5 )4.0 So,= 15+8=1.875.
write 4 as 4.0.
Then, we can ignore the decimal point for a
moment and think of 5)4.0 as 5)40.
Wewrite2 as12thenwrite
Zasadecimal
asshown
below.
Since 5x8 = 40, we know that 5 goes into 40
8 times with 0 left over. 0.005
0.07 0.07 {0.875
Dividing 5 into 4.0 is similar. 0.8 0.8 0.8
8)7.0 8)7.0 8)7.0 0.8
: 5)4.0
Since 5x0.8=4.0, we know that 5 goes into4.0 49 -6.4 =6.4 =6.4
0.8 times with 0 left over. 0 0.6 0.60 0.60
-—0.56 -0.56
So,2=4+5=0.8. 0.04 0.040
-—0.040
Check:
5x0.8=4.0=4.
¥ 0
155. To compute 3 = 3+4 using long division, 4)3.0
we write 3 as 3.0. Then, we can ignore
Since
Z=0.875,wehave= = Z=1.875.
Check: 8x 1.875 = 15.000 =15. W
the decimal point for amoment and think
of 4)3.0 as 4)30.
158. We use the following steps to write : as a decimal.
Since 4x7 = 28, we know 4 goes into 30 a
0.003
total of 7 times with 2 left over. 0.03 0.03
Dividing2 4 into 3.0 is similar. 0.7 0.3 0.3 0.3
: 4)3.0 3)1.0 3)1.0 3)1.0 3)1.0
Since 4x0.7 =2.8, we know 4 goes into -2.8 =0.9 =0.9 =0.9
3.0 a total of 0.7 times with 0.2 left over. 0.2 0.1 0.10 0.10
Next, we divide 4 into 0.2. This is easier to think about if
-0.09 -0.09
0.01 0.010
we write 0.2 as 0.20 and consider dividing 4 into 20.
-0.009
Since 4x5 = 20, we know 4 goes into 20 a ce 0.75 0.001
total of 5 times with 0 left over. 0.7 : We notice a pattern! Each time we divide, we get a
Dividing 4 into 0.20 is similar. he
remainder that is a decimal ending in 1. When we divide
these remainders by 3, the quotient is always a decimal
Since 4x0.05=0.20, we know 4 goes into 0.20
ending in 3.
0.20 a total of 0.05 times with 0 left over. —0:20

So,3=3+4=0.75. So,i=0.333...
=0.3.
Check:
4x0.75=3.00=3.
¥Y

Decimals Chapter 9 Solutions Beast Academy Practice 5C


159. We use the following steps to write 3 as a decimal. We write 1458 as 42—,then write7pasa decimal.
Wes

0.003 0.0003
0.03 0.03 0.003 0.003
0.8 0.8 0.8 0.08 0.08 0.08
6)5.0 6)5.0 6)5.0 12)1.00 12)1.00 12)1.00
=4.8 =4.8 =4.8 -0.96 -0.96 0:96
0.2 0.20 0.20 0.04 0.040 0.040
-0.18 -0.18 - 0.036 - 0.036
0.02 0.020 0.004 0.0040
-0.018 - 0.0036
0.002 0.0004

After dividing 6 into 5.0, each remainder is a decimal Since


+ =0.08333...
= 0.083,
wehave8 =12
<5=1.083.
ending in 2. When we divide these remainders by 6, the First, we write 2 as a decimal using the long Sie
163.
quotient is a decimal ending in 3. So, every digit after the
steps shown below.
8 in our answer is 3.
0.0007
So,2= 0.8333... = 0.83.
0.07
0.002
0.07
0.002
0.07
160.fo usethefollowing
stepstowrite
4asadecimal. 0.2
11)3.0
0.2
11)3.0
0.2
11)3.0
0.2
11)3.0
0.004 =2.2 =2.2 =2.2 =2.2
0.04 0.04 0.8 0.80 0.80 0.80
0.4 0.4 0.4
9)4.0 9)4.0 9)4.0 =077 =O.07 =Oi77
9)4.0 0.03 0.030 0.030
~3.6 -3.6 -3.6 -0.022 -0.022
0.4 0.40 0.40 0.008 0.0080
-0.36 -0.36 -0.0077
0.04 0.040 0.0003
-0.036
The remainders alternate between decimals ending in
0.004
8 and 3. When we divide these remainders by 11, the
So,$=0.444...
= 0.4. co alternate between decimals ending in 2 and 7.

161. We use the following steps to write 7g


= as a decimal.
So, 7770.272727...=
= O27.
Every digit that is an even number of places to the right
0.003 of the decimal point is 7.
0.03 0.03
0.1 0.1 0. So, the 100" digit to the right of the decimal point is 7.
15)2.0 15)2.0 15)2.0 15)2.0
164. We write x as a decimal using the steps shown below.
15 00-150 =15
0.5 0.50 0.50 0.000003
-—0.45 -—0.45 0.0003 0.0003
0.05 0.050 0.03 0.03 0.03
-0.045 33) 1.00 33) 1.00 33) 1.00
0.005 -0.99 =0.99 =0.99
So, = =0. 1333... = 0.13. 0.01 0.0100 0.0100
- 0.0099 - 0.0099
162. We use the followingsteps to writes as a decimal. 0.0001 0.000100
- 0.000099
0.000001
0.0003
0.003 0.003 Adding up our quotients, we have
0.08 0.08 0.08
1.0 1.0 1.0 1.0 + = 0.03+0.0003+0.000003+ :-: = 0.030303... = 0.03.
12)13.0 12)13.0 12)13.0 12)13.0
=12.0 - 12.0 -12.0 =VLE Every digit that is an odd number of places to the right of
1.0 1.00 1.00 1.00 the decimal point is 0.
-0.96 -0.96 - 0.96
0.04 0.040 0.040 So, the 99" digit to the right of the decimal point is 0.
-—0.036 -—0.036 165. We look for fractions whose denominator can be written
0.004 0.0040
-—0.0036 as a power of 10.

0.0004
x Multiplying the numerator and denominator by 4, we
have= =700=0:04.
Aad
So,12=
a2
1.08333...
= 1.083.
x Since 30 = 3x10, and no power of 10 has a factor of
3, we Sane multiply 30 by an integer to get a power of
1OSSOna= cannot be written as a terminating decimal.

Beast Academy Practice 5C Decimals Chapter 9 Solutions


0.333...
97:since91=7%x13, andRopowerof10hasa factor
a 7or13,wecannotwrite4 asa Bese witha +0.333...
denominatorthatisa powerof10.So,*=cannotbe 0.666...
written
asa terminating
decimal. So,a 2x4=(0.333...)
+(0.333...)
=0.666...
=0.6.
jag:Multiplying
A fails thenumerator
125=a4 =0.008. anddenominator
by8,we 169. Since
4370.350.333...,
wehave
en
69a4
5g:Since55=5x11, anenopower of10hasa factorof
a wecannot write525as5fraction
witha denominator Bes
=i+3
thatisa power
of10,ea 56cannotbewritten
asa =(0.333...)
+3.
terminating
decimal. To compute (0.333...)+3, we consider easier quotients.
35:Multiplying
the numeratorand denominatorby5° We know 0.3+3=0.1, 0.33+3=0.11, 0.333+3=0.111,
_ Sees:
_ 1
west30=D8
=Dex5s
=ior SINCe
55canbewritten
asa and so on. Continuing the pattern, we have
fraction with a denominator that is a Powel of 10, it can be
(0.333...)+3=0.111...= 0.1.
writtenas a terminatingdecimal.(Infact, s= = 0.03125.) So, i= 0.1.
We circle the fractions that can be written as terminating
170. We use the fact that a= 0.1 to writeeach fractionas a
decimals, as shown below.
decimal.
ok le
30 55 £=2xi=2x0.1=2x0.111..
=0.222...=02.
166. We'retoldthat2
z canbeee asa terminating decimal. 3=3xi=3x0.1=3x0.111...=0.333...=0.3.
So,itispossibletowrite5as anequivalent
fraction
with
a denominator that is a Bowie of 10. $=4x2=4x0.1=4x0.111...=0.444...=0.4.
Every power of 10 is a product of 2’s and 5’s, and no
other prime. So, it is possible to multiply b by some
2=5xi=5x0.1=
5x0.111...=0.555...=0.5.
integer to get a product of only 2’s and 5’s. 2=6xi=6x
x0.7=6x0.111...=0.666...
=0.6.
Therefore, 2 and 5 are the only primes that can be
included in the prime factorization of b. £=7x4=7x0.1
=7x0.111...=0.777...=0.7.
Note that this is always true for fractions in simplest form.
If5 is notin simplest form and the prime factorization of b
2=8xi=8x0.1=8x0.111...
=0.888...=0.8.
includes primes other than 2 or 5, then it may be possible
Wenotice
thatforanydigit
A,thefraction
4canbe
to write . as a terminating decimal.
written
as0.A.
For example, consider ; = 3. The prime factorization of 171.
b is 2x3. However, the 3 in the prime factorization of a
InProblem169,wefoundthat4=0.7.
So,
cancels with the 3 in the prime factorization of bwhen a=txt=0.1x0.1=0.1x0.111..
90° 10°°9
we simplify 2 to i So, we can write 2 as the terminating Multiplying a number by 0.1 moves the decimal point one
decimal 0.5. place to the left. So,

1£..
ia RRM re = ‘ones
.—Conversion Strat
1 67. |
fgted Oh Then, 1oy
wehave46 Cae
— y axa9=2x0.01=0.02:
Wenoticethat a0
— 10S
eee70:1 and ca
Since 70 3 = 0.125, we have a,
30 = 0-1 x0.125.
Also,
wehave1Gabel
67a=s}ieae
6xob=6x0.01==0.06.
Multiplying a number i 0.1 moves the decimal point one So,a come statements
areaspats
place
totheleft.So, = 5X2=0.1x0.125 =0.0125. =0.1,SO
55
+=0.07.Therefore,
76
+=&=0.02,
168. Since£=0.3=0.333...,
wehaveés2x4=2x0.333... and=:=&=0.06.
172. Since x = 0.03 = 0.030303..., we have
Multiplying
(0.333...)
by2doubles allofthe3’s.
So,£=2x4=2x0.333... =0.666...
=0.6.
nOhes
Multiplying
anumber by2 isthesameasadding 2copies =(0.030303...)
+3.
ofthatnumber.
So,2x2=(0.333...)
+(0.333...).
Westack We know 0.03+3=0.01, 0.0303+3=0.0101, and
these
decimals
vertically,
andaddthem
asshown: 0.030303+3 =0.010101.

Decimals Chapter 9 Solutions Beast Academy Practice 5C


Continuingthispattern,we have 177. Multiplying the numerator and denominator of 47 by 9, we
have = = 88
(0.030303...)+3=0.010101...iI 2 —
1199°
So,95=0.01. ForanyBe-digit
number
AB,weknow
2e2=0.AB.
fae
173. We use the fact that s = 0.01 to write each fraction as a
So,q179970:63.
|.
decimal. 178. re ee the numeratorand denominatorof &
3 DY3, we
have +=12
28
5 =2x aye di=
=2x0.01= 0.02. 33°99°
Foranytwo-digit
number
AB,
weknow
22
2=0,AB.
=
& 7X =7x0.01
=0.07
07.
So, +=12=0,72.
33° 99
Ua17xs 5 = 17x0.07 =0.17, 179. ee el a
Wehave55=9%749 7
=0-70.1.
49 _ siioee ae a Multiplying a number by 0.1 moves the decimal point one
#2=49x24
=490.07= 0.49 place to the left. one 607
S =0.7x0.1 =0.07.

= 75
19 i==9xg5
=9x0.07
ATS FT=0.09. 180.
From
thepreuaus
problem,
weknow
that
0 .07
=<
_28
So,4x0.07=4xt =Sor oe
174. From the previous problem, we notice that fractions of
the form se can be written as decimals of the form 0.AB, We know that for any digit A, the fraction & can be written
where A and B are digits. as 0.A. Using this fact, we have

So, we guess that the reverse is true, and that 0.36 = 2


We have i
0.36 = 0.363636...
= 36x0.010101...
= 360.01

So,4x0.
07=£8=0,31.
&.
Wethenaii ourfraction
toget0.36=2 =41: 0.28 038 025 030 0.37

Ingeneral,
=4 =0.AB,whereAandB aredigits.
181. Multiplying
13_65
thenumerator
and denominator
of82 by5,we
175. eal3° Sede
We have <4 = T11ers. —01229+
a7 111=0.333=111.
have73=99:
To compute 0.333+111, we consider easier quotients.
We know that for any digit A, the fraction & can be written
0.333+111=0.003, and 0.333333+ 111= 0.003003.
as 0.A. Using this fact, we have
From this pattern, we see that
0.333+111 =0.003.

So
0,sas=0.003.
acaisSAal
Then,wehaveoo5=333°3=o +3=0.003+3.

Since 0.003+3=0.001 and 0.003003+3=0.001001, and


So 13. 95.
° 8 30 0.72.
so on, we see that
182. Since >2 =0.5 and i= 0.3, we add 0.5+0.3.
0.003+3 = 0.001.
We line up the decimal points and write 0’s after the 5 in
0.5. Then, we add as shown below.
So
He wapae
=SB= 27Xgpg
Then, == =27«0.001
=0.027. 0.5000...
+ 0:3333=.
So, the completed statements are as follows: 0.8383...
S_1,1_9= 0.83.
So, 62°33 93
=0.333,
so=oa=a 0.003,
and<5=0.001.
=~
|-
Therefore,= ine)
NJ
=—-=0.027. 183. Since
is 0.3,andis 0.25,weadd0.3+0.25.
We line up the decimal points and write 0’s after the 5 in
176. Writing
422
©asamixed
number,
wehave 0.25. Then, we add as shown below.

OS8S3se

0.58333...
(eh ar 0.583
So, Dyin

Beast Academy Practice 5C Decimals Chapter 9 Solutions


188. We begin by converting each fraction to a decimal.
184. Since
f=0.6,and+=0.25,weadd0.6+0.25.
We line up the decimal points and write 0’s after the 5 in
0.25. Then, we add as shown below.
3
dat-3= 5-3=.16
1 1
0.66666... 0.66666... 0.66666...
+ 0.25000... + 0.25000... +0:25000.
666... 1666... 0.91666... a= 01
diario ry
So, To= 3tq =0-916 There are only two empty squares in
the path for a= .01. So, we fill these
185. Since
é=0.6,and+=0.25,wesubtract
0.6-0.25. squares with 0 and 1, as shown.
We line up the decimal points and write 0’s after the 5 in
0.25. Then, we subtract as shown below.
Then, to complete the path for Z= 10: i
0.66666... we place a 6 in the bottom-middle
= 0.25000...
square.
0.41666...
Placing the 6 also completes the path
5-2
So, oleae 19416
= 0.416. for
Or e5== .06.

186. Since
i= 0.3,and; =0.25,wesubtract
0.3-0.25. Finally, to complete the path for
We line up the decimal points and write 0’s after the 5 35 2.6, we place a 2 in the empty
in 0.25. To subtract, we need to take a tenth from the square before the decimal point.
tenths place in 0.3 to make 13 hundredths. Then, we
subtract as shown below.
Each fraction’s path traces its decimal 3
213
0.33333... 0.33333... 213
0.33333... form, so we are done.
~025000... . —0.250004.~. 2 -0'25000.;.
0.08333...
So,+=1-1=0.083 We use the strategies discussed in the previous
al ee f problems to solve the following puzzles.

187. We begin by converting each fraction to a decimal. Since


189. Inll Oo
yoo
leading zeros to the left of the decimal point are ignored
Ol
|=
w I l |
in these puzzles, we do not write leading zeros when we
convert.

a=.25, a576, 2-5


97-2 a oO

There are only two empty squares in le


190. M wo
the path for ts .25. So, we fill these ax
squares with 2 and 5, as shown. —_
Placing the 2 also completes the path
for2=2
or Ga 2.
191.
Then, the path for 3 = .6 has only one
empty square after the decimal point.
Ml
Io
So, we fill this square with 6.
Blo
o
Il
oY
Each fraction’s path correctly traces its
decimal form, so we are done. 192. =

disettoe tad
38 eyo = Hi =
yg=2+.6+.25=2.916

Decimals Chapter 9 Solutions Beast Academy Practice 5C


193. We convert each fraction to a decimal. 195. We convert each fraction to a decimal.

4=.16 4
1-32-36 Sai 18

2 =61=6.1 ee Se 88
2=12=1.8 2-81. BT
Since
thesquares
inthebottom 4 =8 2A8
row make up the repeating part of
In the second row of the grid, there are three empty
the path for += .16, each square
squares which are part of the path of 2= .8. So, exactly
must remain empty or contain a 6.
one of these squares contains an 8.

The path of oe = .88 crosses an


The left empty square is the first empty square in the third row.
square in the path of a =3.1, So Then, it crosses the three empty
it cannot be 6. Therefore, it must squares in the second row, where
remain empty. We cross out this there can only be one 8. So, we
square as a reminder that it is must place an 8 in the third row.
empty, then place a 6 in the right
square as shown.
Then, we consider the loop that is
shared by the paths for 2 =546)
Then, we use the strategies = = .81, and = = .18. One of the
discussed in previous problems to squares in this loop contains a 1,
complete the puzzle as shown. and another contains an 8.

1 cannot go in either of the loop’s


top squares, since they are in the
194. We convert each fraction to a decimal. path of z= 8. Also, 1 cannot go

* =a =axa - 2ext =26x.1=.26 intheloop’s


bottom-right
square,
a at since this is the first empty square
3723523 inthe pathof 2 =81. So, 1 must
Paglia. go in the bottom-left square. This
TRS 9
completes the path for 507 .18, so
= =.3 we place an X to the right of the 1.

We consider the squares in the


path of 4 = .26, each of which must Then, we cannot place an 8 above
be empty or filled with a 2 or a 6. the 1, since this is the first empty
square in the path of = =¢18/'So0,
One of these squares is the first we must place the 8 as shown.
square in the path of 2 = 4.5, which
cannot have any 2’s or 6’s. ; s i
Finally, we place a 5 in the first
Another square is the first square Tairare axihe path 6t y 5 To,
in the path of = = .3, which cannot
have any 2’s or 6’s.
Zt
3p
So, we place an X in these two Q
P|
squares, then fill the remaining
empty squares in the path of
= = .26 with 2 and 6 as shown. Each fraction’s path correctly
traces its decimal form, so we are
Then, we use the strategies done.
discussed in previous problems to
complete the puzzle as shown.

Beast AcademyPractice 5C Decimals Chapter 9 Solutions | 179


196.Wehavets 25and2=2.5, Then, we complete the path for

So, the first square in the path of 2 2 = 5.5. This completes the path
must be filled with 2, and the first 13).
for55=52.
square in the path of the bottom Z
4 Finally, we place the 5 in the
_.must be empty.
path for 1_ 5 in the only square 4
that does not interfere with
Then, we consider the path of the another path.
bottomi.
Exactly one of the two shaded
198. We complete the path for
squares shown to the right must
contain a 2. f= 1.75, placing the 1 so that it is
not in the path of 8 =5.3.

If we place the 2 as shown, then


we must fill the remaining empty
squares in the bottom #8 path with
an X anda 5.
Then, we complete the path for
at = 3.375, placing the second 3
so that it is not in the path of
Also, the square to the left of the 2 a =.7, and placing the 5 so that
must be filled with a 2 to complete it is not in the path of Z= at
the path of the top t

However, this forces the first two


digits in the left 1’s path to be 2's. x

So, we must place a 2 as shown.


Wey we complete the path
Then, there is no digit that can go
for Ch ofa
beneath this 2 that will satisfy the
paths of the left i and the top i. This also completes the path
for a =.7.
So, this square must be empty.

In the shaded square on the right,


there is no digit that can satisfy the
paths of the left i and the bottom Each fraction’s path correctly
r So, this square must also be traces its decimal form, so we
empty. are done.

Finally, the last square in the path


of the bottom i must be 5, and the
last two squares in the path of the
a: U 33
40
left i must be 2 and 5. 199. The5 inthe path of 2=.5 cannot «|
be inthe path of= = .28,SOwe 2..»1/
placethe 5 as shown. s ©
Each fraction’s path correctly
traces its decimal form, so we are 66 2 2=
done.
2SoO
ON
IS
1259 } 9
Then, we complete the path for
Z = .425. This completes the
197.Wecompletethepathsforz=.2,x =.55,and2=2.20, path for the é= .2 on the right.

Completing
thepathfor: =2.25 also completes the path We place *’s in the squares at
the ends of these two paths as
for£1=5.25. shown.
PS) 9 WO.
- oy)xo
Decimals Chapter 9 Solutions Beast Academy Practice 5C
o
Then, we complete the path
for the é = .2 on the left, then
the path for x = .28. This also <2=.725 i= .16
completes the path for 8 = 825.
21=5.25 4=18
Finally, we place a 5 to complete
the path for & = 528. 2=2.25 2 =81
a= .9 = =.06
Each fraction’s path correctly t= 4.8
traces its decimal form, so we
are done. There is only one way to place these fractions and
exactly one of each digit so that each fraction’s path
correctly traces its decimal form. (For a complete
step-by-step solution, visit www.BeastAcademy.com.)

200. The empty squares in the middle


row are in the repeating part of
the paths for a a Ay 2 = .64,
4
and 5 a3
=.4.So, each of these
squares must remain empty or
contain a 4.

We consider the loop that is


shared by the following paths:
6_
Tt, a 417 mw Sz
77 = 194, 77 =-45. 202. We write 0.05+1.25 0.05
as +5s: Then, we multiply the
One of the squares in this numerator and denominator of ae by 100 to get an
loop contains a 5, and another equivalent fraction whose numerator and denominator
contains a 4. are both integers.

x100
5 cannot go in either of the loop’s ¥ * $ 0.051 (gu295
top squares, since the only digit A+= be 25 125
that can go in either square is 4. er x10
Also, 5 cannot go in the loop’s a Bie eons 1 : api
z
lower-left square, since this is :y 1 —
126 simplifies
aaa to =,
25 so we have 0.05+1.25==. 25

the first empty square in the path 19: 203. We write 0.4+2.2 as +. Then, we multiply the
of =; =-45. So, a 5 must go in the numerator and denominator of 33 by 10 to get an
loop’s lower-right square. equivalent fraction whose numerator and denominator
We cannot place a 4 above the are both integers.
5, since this is the first empty x10
square in the path of = = 54, O4nt ee
Also, we cannot place a 4 to the 2 928
left of the 5 weet this square
is in the path of 755 = .256. & simplifies to 4, so we have 0.4+2.2 =2.
So, we placea 4 in the loop’s ae
: 204. We write 3.03+0.6 as %. Then, multiplying the
upper-left square. 0.6 3.03
; F numerator and denominator of 06 by 100 gives
ae AUIS cee W7:: 3.03 _903_101_ 51
and +; = .54 are now complete. Ge an 30 ae
We finish the puzzle by 205. We write 3.5+70.707 as ==. Then, multiplying the
completing the following paths: numerator and denominator of Fea by 1,000 gives

29_
7a ae6g 32= eg 2.5 ae 3.5 _ 3,500 __ 500
70.707 70,707 10,101"

1 eee. Shen
coed 54, 9 =3.4|

Beast Academy Practice 5C Decimals Chapter 9 Solutions


206. Multiplying
the numeratorand denominatorofp0aee
22 by 100 215. The smallest positive number with three digits after the
decimal point is 0.001. So, we look for the number that
gives

e
0.35_35_7 makes the following equation true:
O05rKS.=)*
0.008x[____]=0.001.
207. Multiplying
the numeratorand denominatorofa034 by 100
lf 0.008x[___]=0.001, then 0.001+0.008=[__].
gives
We can write 0.001 +0.008 as 2:22" then multiply the
2.4 _ 240
GoaG50" 0.008’
0.001
numerator and denominator by 1,000 to get 0.008 _
—~—=8" 1
208. Multiplying
the numeratorand denominator ofeve
===by
So, the smallest positive number we can multiply by
1,000 gives
0.008 to get a decimal product with 3 digits after the
Q1092562
0.8 8001224.~
0d ass decimal point is 2= 0.125.

209. We write4.9+0.14 as a quotientof integers,then divide:


The smallest positive number with three digits after the
2 es FO
4.970.14=—"7=14=35. decimal point is 0.001.

210. We write 0.002+0.04 as a quotient of integers, then We know 0.008+8 = 0.001. Since dividing by a number is
convert the resulting fraction to a decimal: the same as multiplying by that number's reciprocal, we
have
0.002+ 008 goo
oat Se 2 jt 5h
oG 100 0.05. 0.008x4=0.001.

211. We write 0.6+4.5 as a quotient of integers, then convert So, the smallest positive number we can multiply by
the resulting fraction to a decimal: 0.008 to get a decimal product with 3 digits after the
decimal point isg10.125.
=06_ 6 _12_12,1
0.674.5=75=46 =on 6 xt =15x0.1=0.13.
.» TOmake our siciouaene easier, we write out the first
several digits of each decimal. Then, we line up our place

Since >4= 0.5, we have (3y" = (0.5)%. To find the


values, filling any empty place values with trailing zeros. eaarT
212.
rightmost digit of (0.5)*°, we temporarily ignore the
0.05=0.050505...
decimal point and consider 5°°. 0.050=0.050050...
All positive powers of 5 have units digit 5. For example, 0.05=0.050000
5'=5, 5?=25, 5°=125, and so on. So, 5% is equal to 0.05=0.055555...
some integer whose rightmost digit is 5.
Each decimal has 0 ones, 0 tenths, and 5 hundredths. In
Ja
To compute (0.5)°°, we move the decimal point in 5% the thousandths place, only 0.05 has a nonzero digit. So,
ninety-nine places to the left. Moving the decimal point 0.05 is the largest decimal.
left does not change the rightmost digit. So, the
0.05=0.05[01505...
rightmost digit of (0.5)°? is 5.
0.050=0.05[0]050...
213. 5 1 4\10
Since 5 70.2, we have (2) =(0.2)'°.
0.05=0.05[0|000
To compute (0.2)'°, we move the decimal point in
0.05=0.05[5)555...
21°= 1,024 ten places to the left.
Among the three remaining decimals, only 0.05 has a
Since 1,024 has 4 digits, moving the decimal point ten nonzero digit in the ten-thousandths place. So, 0.05 is
places to the left gives 10-4 =6 zeros to the right of the
the second-largest decimal.
decimal point before the first nonzero digit.
0.05 = 0.050[5]05...
We have
0.050 = 0.050/0]50...
(5)=(0.2)""=0,goq9001024, 0.05 = 0.050[0]00
214. Since potatoes cost $0.67 per pound, the cost of 6.375 Then, 0.050 has 5 hundred-thousandths while 0.05 has
pounds of potatoes is 0.676.375 dollars. 0 hundred-thousandths. So, 0.050 is the third-largest
Since 0.67 is less than 0.7 and 6.375 is less than 7, we decimal, and 0.05 is the smallest.
know that 0.67 x6.375 is less than 0.7x7 =4.9.
0.050 = 0.0500/5)0...
Since 0.67 is more than 0.6=5 and 6.375 is more than 6, 0.05 = 0.0500/0)0
we know that 0.67 x6.375 is more than Ex6 =4,
We list the four decimals in order from least to greatest.
So, 0.676.375 is between 4 and 4.9. Therefore, the
0.05, 0.050, 0.05, 0.05
smallest whole number of dollars needed to buy the
potatoes is 5.
In fact, 0.67x6.375 = 4.27125.

Decimals Chapter 9 Solutions Beast Academy Practice 5C


217. We begin by writing each term in the sum as a fraction. with a denominator that is a power of 10.

eu
0.1=5. gis.
2x5
eee
27x5* 5e
ae)
0.01Soto
=0.1x0.1 ne
=ox75=55. ~ 27x57
roe
0.001=0.01x0.1iene
=90%70=900° TOF
Multiplying x by any number smaller than 63 leaves a
So,0.1+0.01+0.001
=S+hte.1 prime factor other than 2 or 5 in the denominator of the
We write each fraction with a common denominator of
resulting fraction.
900, then add:
So, 63 is the euanee! positiveintegerthat can be
1 1 1 = 100, 10, 1 multipliedby a to get a resultthat can be expressed as a
terminatingdecrial
219. In order to divide, we write each decimal as a fraction.

0.03=a icine
70 30°
So,0.1+0.01+0.007
=a 0.2=6.
So,wehave
We add the decimals vertically, writing the first several 0.03
_9 9a-05=-122
Op ~0.038+0.2=55
+6.
digits to the right of each decimal point.
To divide by a number, we multiply by its reciprocal. So,
Ost ATs.
OOH... Ie od oe
30°9 30 2 60 20°
+0.00111...
0.12333... Finally,weconverta toa decimal:
So, 0.1+0.01+0.001 = 0.123. We use the steps shown
below to write 0.123 as a fraction:
B=Jo=0.15.
3 a
0.03
_
So, O35
—>=0.15.
0.123 = 12.3x0.01
220. Since 0.3 = 4 adding three copies of 0.3 is the same
as adding three copies of 4. So, Grogg’s sum is
equal to
Ti Mie ieeaiag
3°3°3 ae 1.

218. Only a fraction that can be written with a denominator We have just shown that 0.9 and 1 are the same number!
that is a power of 10 can be written as a terminating This may sound strange at first, but one way to convince
decimal. So, if a fraction can be written as a terminating ourselves that this is true is by trying to find a number
decimal, then 2’s and 5’s are the only primes in the prime between 0.9 and 1.

factorization
oftheonce If two numbers are different, then they fall on different
The denominatorof 5jis 8!,and the primefactorization places on the number line, and we can always zoom in
of 8! is and find some number in between them on the number
8!=8x7x6x5x4x3x2x1 line. However, there are infinitely many 9’s after the
= (2x2x2)x7x(2x3)x5x(2x2)x3x2 decimal point in 0.9, so there is no number we can write
I EKOOO Mie that is greater than 0.9 while also being less than 1. (Try
it!). Since there is no number between 0.9 and 1, they
In order to write the denominator oft ; as a power of 10, must be the same number!
we must cancel the factors of 3? aa 7 in (8!).
If this is still confusing to you, don’t worry too much.
The smallest number that has 3? and 7 as factors is Things involving infinity can be very tough to think about!
3?x7 =9x7=63. Multiplying x by 63 gives As you continue learning math, you'll discover other ways
to show that 0.9 and 1 are the same number.

Since 5ae 5 has only factors of 2 and 5 in the


denominator, it can be written as an equivalent fraction

Beast Academy Practice 5C Decimals Chapter 9 Solutions


20

20

20:

205

21(

211

For additional books, printables, and more, visit

BeastAcademy.com
212
Ready to move beyond Beast Academy?
‘Visit Art of Problem Solving at
AoPS.com

Books Community
Curriculumforgrade 6-12 Over200,000
members
213.
Online School Free Resources
Livecourses taughtby mathbeasts Games, videos,and lessons

214.

182
Decimals Chapter 9 Solutions Beast Academy Practice S
PRACTICE
Beast Academy is a series of illustrated textbooks and
workbooks from the math beasts at Art of Problem Solving.
Together with the companion website, BeastAcademy.com,
Beast Academy provides a rigorous, challenging, engaging,
and fun curriculum for aspiring math beasts in grades 2-5.

This is Practice 5C in a four-book series for fifth grade:

5A 5B oC 9D
Cs) Dsio)
|(ols) ORS)
telitss
(es ¢Sequences e Percents
e Integers e Factors & ‘e Ratios & Rates e Square Roots
e Expressions & Multiples *Decimals ° Exponents
Equations e Fractions :

The companion book, Guide 5C, uses an entertaining, comic book


style to teach the mathematical concepts found in this book.

Ready to move beyond Beast Academy?


Visit Art of Problem Solving at
AoPS.com

Books Community
Curriculumforgrades 6-12 Over200,000
members
Online School Free Resources
Live courses taught by math beasts Games, videos, and lessons

About the Publisher ISBN 978-1-934124-65-9


Art of Problem Solving books provide a complete 90000>
mathematics curriculum for high-performing students in
grades 6-12. The Art of Problem Solving Online School
offers challenging courses at various levels, including
middle-school and high-school mathematics, contest
and olympiad preparation, and computer science.
Learn more at www.artofproblemsolving.com 9 "781934"124659

You might also like